ALL SETS: NR 304 + NR 302 Practice questions

Ace your homework & exams now with Quizwiz!

The nurse is obtaining a health history on an 87-year-old woman. Which of the following areas of questioning would be most useful at this time? a.Obstetric history b.Childhood illnesses c.General health for the past 20 years *d. Current health promotion activities*

D It is important for the nurse to recognize positive health measures, such as what the person has been doing to help him or herself stay well and to live to an older age. The other responses are not pertinent to a patient of this age.

37. The nurse is reviewing laboratory studies on a patient who may have protein malnutrition. Which of these measurements is an early indicator of protein malnutrition?

D) Serum transferrin

1. A patient has been unable to eat solid food for 2 weeks, and is in the clinic today complaining of weakness, tiredness, and hair loss. The patient states that her usual weight is 175 pounds, but today she weighs 161 pounds. What is her recent weight change percentage? To calculate recent weight change percentage, use this formula:

ANS: 8%

The nurse has discovered decreased skin turgor in a patient and knows that this finding is expected in which condition? a. Severe obesity b. Childhood growth spurts c. Severe dehydration d. Connective tissue disorders such as scleroderma

ANS: C Decreased skin turgor is associated with severe dehydration or extreme weight loss.

During the aging process, the hair can look gray or white and begin to feel thin and fine. The nurse knows that this occurs because of a decrease in the number of functioning: a. Metrocytes. b. Fungacytes. c. Phagocytes. d. Melanocytes

ANS: D In the aging hair matrix, the number of functioning melanocytes decreases; as a result, the hair looks gray or white and feels thin and fine. The other options are not correct. DIF: Cognitive Level: Understanding (Comprehension) MSC: Client Needs: Health Promotion and Maintenance

A father brings in his 2-month-old infant to the clinic because the infant has had diarrhea for the last 24 hours. He says his baby has not been able to keep any formula down and that the diarrhea has been at least every 2 hours. The nurse suspects dehydration. The nurse should test skin mobility and turgor over the infant's: a. Sternum. b. Forehead. c. Forearms. d. Abdomen.

ANS: D Mobility and turgor are tested over the abdomen in an infant. Poor turgor, or tenting, indicates dehydration or malnutrition. The other sites are not appropriate for checking skin turgor in an infant.

When the nurse is conducting sexual history from a male adolescent, which statement would be most appropriate to use at the beginning of the interview? A) "Do you use condoms?" B) "You don't masturbate, do you?" C) "Have you had sex in the last 6 months?" D) "Often boys your age have questions about sexual activity."

ANS: D Start the interview with a permission statement. This conveys that it is normal and all right to think or feel a certain way. Avoid sounding judgmental.

During a health history, a 22-year old woman asks, "Can I get that vaccine for HPV? I have genital warts and I'd like them to go away!" What is the nurse's best response? A) "The HPV vaccine is for girls and women ages 9 to 26, so we can start that today." B) "This vaccine is only for girls who have not started to have intercourse yet." C) "Let's check with the physician to see if you are a candidate for this vaccine." D) "The vaccine cannot protect you if you already have an HPV infection."

ANS: D The HPV (human papillomavirus) vaccine is appropriate for girls and women age 9 to 26 and is given to prevent cervical cancer by preventing HPV infections before girls become sexually active. However, it cannot protect the woman if an HPV infection is already present.

The nurse educator is preparing an education module for the nursing staff on the epidermal layer of skin. Which of these statements would be included in the module? The epidermis is: a. Highly vascular. b. Thick and tough. c. Thin and nonstratified. d. Replaced every 4 weeks.

ANS: D The epidermis is thin yet tough, replaced every 4 weeks, avascular, and stratified into several zones.

A 60-year-old man has just been told he has benign prostatic hypertrophy. He has a friend who just died from cancer of the prostate, and he is concerned this will happen to him. How should the nurse respond? A) "The swelling in your prostate is only temporary and will go away." B) "We will treat you with chemotherapy so we can control the cancer." C) "It would be very unusual for a man your age to have cancer of the prostate." D) "The enlargement of your prostate is caused by hormone changes and not cancer."

ANS: D The prostate gland commonly starts to enlarge during the middle adult years. This benign prostatic hypertrophy (BPH) is present in 1 out of 10 males at the age of 40 years and increases with age. It is thought that the hypertrophy is caused by hormonal imbalance that leads to the proliferation of benign adenomas. The other responses are not appropriate.

When performing a genitourinary assessment on a 16-year-old boy, the nurse notices a swelling in the scrotum that increases with increased intra-abdominal pressure and decreases when he is lying down. The patient complains of pain when straining. The nurse knows that this description is most consistent with a(n) _____ hernia. A) femoral B) incisional C) direct inguinal D) indirect inguinal

ANS: D With indirect inguinal hernias there is pain with straining and a soft swelling that increases with increased intra-abdominal pressure, which may decrease when the patient lies down. These findings do not describe the other hernias. See Table 24-7 for descriptions of femoral and direct inguinal hernias

The nurse educator is preparing an education module for the nursing staff on the epidermal layer of skin. Which of these statements would be included in the module? The epidermis is: a. Highly vascular. b. Thick and tough. c. Thin and nonstratified. d. Replaced every 4 weeks.

ANS: D The epidermis is thin yet tough, replaced every 4 weeks, avascular, and stratified into several zones. DIF: Cognitive Level: Understanding (Comprehension) MSC: Client Needs: General

A 35-year-old man is seen in the clinic for an infection in his left foot. Which of these findings should the nurse expect to see during an assessment of this patient? a. Hard and fixed cervical nodes b. Enlarged and tender inguinal nodes c. Bilateral enlargement of the popliteal nodes d. Pelletlike nodes in the supraclavicular region

ANS: Enlarged and tender inguinal nodes The inguinal nodes in the groin drain most of the lymph of the lower extremities. With local inflammation, the nodes in that area become swollen and tender.

The nurse is explaining the mechanism of the growth of long bones to a mother of a toddler. Where does lengthening of the bones occur? A) Bursa B) Calcaneus C) Epiphyses D) Tuberosities

ANS: Epiphyses Lengthening occurs at the epiphyses, or growth plates. The other options are not correct.

A patient states that the pain medication is "not working" and rates his postoperative pain at a 10 on a 1 to 10 scale. Which of these assessment findings indicates an acute pain response to poorly controlled pain?

ANS: Increased blood pressure and pulse Responses to poorly controlled acute pain include tachycardia, elevated blood pressure, and hypoventilation. Confusion and depression are associated with poorly controlled chronic pain. See Table 10-1.

A 54-year-old man comes to the clinic with a "horrible problem." He tells the nurse that he has just discovered a lump on his breast and is fearful of cancer. The nurse knows that which statement about breast cancer in males is true? a. Breast masses in men are difficult to detect because of minimal breast tissue. b. Breast cancer in men rarely spreads to the lymph nodes. c. One percent of all breast cancers occurs in men. d. Most breast masses in men are diagnosed as gynecomastia.

ANS: One percent of all breast cancer occurs in men. One percent of all breast cancer occurs in men. Early spread to axillary lymph nodes occurs due to minimal breast tissue.

During an examination of a woman, the nurse notices that her left breast is slightly larger than her right breast. Which of these statements is true about this finding? a. Breasts should always be symmetric. b. Asymmetry of breast size and shape is probably due to breastfeeding and is nothing to worry about. c. Asymmetry is not unusual, but the nurse should verify that this change is not new. d. Asymmetry of breast size and shape is very unusual and means she may have an inflammation or growth.

ANS: c. This finding is not unusual, but the nurse should verify that this change is not new. The nurse should notice symmetry of size and shape. It is common to have a slight asymmetry in size; often the left breast is slightly larger than the right. A sudden increase in the size of one breast signifies inflammation or new growth.

An imaginary line connecting the highest point on each iliac crest would cross the _____ vertebra. A) first sacral B) fourth lumbar C) seventh cervical D) twelfth thoracic

ANS: fourth lumbar An imaginary line connecting the highest point on each iliac crest crosses the fourth lumbar vertebra

A patient is visiting the clinic for an evaluation of a swollen, painful knuckle. The nurse notices that the knuckle above his ring on the left hand is swollen and that he is unable to remove his wedding ring. This joint is called the _____ joint. A) interphalangeal B) tarsometatarsal C) metacarpophalangeal D) tibiotalar

ANS: metacarpophalangeal The joint located just above the ring on the finger is the metacarpophalangeal joint. The interphalangeal joint is located distal to the metacarpophalangeal joint. The tarsometatarsal and tibiotalar joints are found in the foot and ankle. See Figure 22-10 for a diagram of the bones and joints of the hand and fingers.

21. When the mid-upper arm circumference and triceps skinfold of an 82-year-old man are evaluated, which is important for the nurse to remember?

C) These measurements may not be accurate because of changes in skin and fat distribution.

During a staff meeting, nurses discuss the problems with accessing research studies to incorporate evidence-based clinical decision making into their practice. Which suggestion by the nurse manager would best help these problems? a. Form a committee to conduct research studies. b. Post published research studies on the units bulletin boards. c. Encourage the nurses to visit the library to review studies. d. Teach the nurses how to conduct electronic searches for research studies.

D Facilitating support for EBP would include teaching the nurses how to conduct electronic searches; time to visit the library may not be available for many nurses. Actually conducting research studies may be helpful in the long-run but not an immediate solution to reviewing existing research.

Which situation is most appropriate during which the nurse performs a focused or problem-centered history? a. Patient is admitted to a long-term care facility. b. Patient has a sudden and severe shortness of breath. c. Patient is admitted to the hospital for surgery the following day. d. Patient in an outpatient clinic has cold and influenza-like symptoms.

D In a focused or problem-centered data base, the nurse collects a mini data base, which is smaller in scope than the completed data base. This mini data base primarily concerns one problem, one cue complex, or one body system.

In the health promotion model, the focus of the health professional includes: a. Changing the patients perceptions of disease. b. Identifying biomedical model interventions. c. Identifying negative health acts of the consumer. d. Helping the consumer choose a healthier lifestyle.

D In the health promotion model, the focus of the health professional is on helping the consumer choose a healthier lifestyle.

The nurse is reviewing the structures of the ear. Which of these statements concerning the eustachian tube is true? a. The eustachian tube is responsible for the production of cerumen. b. It remains open except when swallowing or yawning. c. The eustachian tube allows passage of air between the middle and outer ear. d. It helps equalize air pressure on both sides of the tympanic membrane.

D The eustachian tube allows an equalization of air pressure on each side of the tympanic membrane so that the membrane does not rupture during, for example, altitude changes in an airplane. The tube is normally closed, but it opens with swallowing or yawning.

A 42-year-old patient of Asian descent is being seen at the clinic for an initial examination. The nurse knows that including cultural information in his health assessment is important to: a. Identify the cause of his illness. b. Make accurate disease diagnoses. c. Provide cultural health rights for the individual. d. Provide culturally sensitive and appropriate care.

D The inclusion of cultural considerations in the health assessment is of paramount importance to gathering data that are accurate and meaningful and to intervening with culturally sensitive and appropriate care.

A 90-year-old patient tells the nurse that he cannot remember the names of the medications he is taking or for what reason he is taking them. An appropriate response from the nurse would be: a."Can you tell me what they look like?" b."Don't worry about it. You are only taking two medications." c."How long have you been taking each of the pills?" d."Would you have a family member bring in your medications?"

D The person may not know the drug name or purpose. When this occurs, ask the person or a family member to bring in the drug to be identified. The other responses would not help to identify the medications.

Which statement best describes a proficient nurse? A proficient nurse is one who: a. Has little experience with a specified population and uses rules to guide performance. b. Has an intuitive grasp of a clinical situation and quickly identifies the accurate solution. c. Sees actions in the context of daily plans for patients. d. Understands a patient situation as a whole rather than a list of tasks and recognizes the long-term goals for the patient.

D The proficient nurse, with more time and experience than the novice nurse, is able to understand a patient situation as a whole rather than as a list of tasks. The proficient nurse is able to see how todays nursing actions can apply to the point the nurse wants the patient to reach at a future time.

A patient is describing his symptoms to the nurse. Which of these statements reflects a description of the setting of his symptoms? a."It is a sharp, burning pain in my stomach." b."I also have the sweats and nausea when I feel this pain." c."I think this pain is telling me that something bad is wrong with me." d."This pain happens every time I sit down to use the computer."

D The setting describes where the person is or what the person is doing when the symptom starts. Describing the pain as "sharp and burning" reflects the character or quality of the pain; stating that the pain is "telling" the patient that something bad is wrong with him reflects the patient's perception of the pain; and describing the "sweats and nausea" reflects associated factors that occur with the pain.

During a speculum inspection of the vagina, the nurse would expect to see what at the end of the vaginal canal? A) Cervix B) Uterus C) Ovaries D) Fallopian tubes

ANS: A At the end of the canal, the uterine cervix projects into the vagina

A patient has been admitted to the emergency department with a possible medical diagnosis of pulmonary embolism. The nurse expects to see which assessment findings related to this condition? a. Absent or decreased breath sounds b. Productive cough with thin, frothy sputum c. Chest pain that is worse on deep inspiration and dyspnea d. Diffuse infiltrates with areas of dullness upon percussion

ANS: Chest pain that is worse on deep inspiration, dyspnea Findings for pulmonary embolism include chest pain that is worse on deep inspiration, dyspnea, apprehension, anxiety, restlessness, PaO2 less than 80, diaphoresis, hypotension, crackles, and wheezes.

49. When checking for proper blood pressure cuff size, which guideline is correct? a. The standard cuff size is appropriate for all sizes. b. The length of the rubber bladder should equal 80% of the arm circumference. c. The width of the rubber bladder should equal 80% of the arm circumference. d. The width of the rubber bladder should equal 40% of the arm circumference.

ANS: D The width of the rubber bladder should equal 40% of the circumference of the persons arm. The length of the bladder should equal 80% of this circumference.

In performing a voice test to assess hearing, which of these actions would the nurse perform? a. Shield the lips so that the sound is muffled. b. Whisper a set of random numbers and letters, and then ask the patient to repeat them. c. Ask the patient to place his finger in his ear to occlude outside noise. d. Stand approximately 4 feet away to ensure that the patient can really hear at this distance.

B With the head 30 to 60 cm (1 to 2 feet) from the patient's ear, the examiner exhales and slowly whispers a set of random numbers and letters, such as "5, B, 6." Normally, the patient is asked to repeat each number and letter correctly after hearing the examiner say them.

In response to a question regarding the use of alcohol, a patient asks the nurse why the nurse needs to know. What is the reason for needing this information? a.This information is necessary to determine the patient's reliability. *b. Alcohol can interact with all medications and can make some diseases worse.* c.The nurse needs to be able to teach the patient about the dangers of alcohol use. d.This information is not necessary unless a drinking problem is obvious.

B Alcohol adversely interacts with all medications and is a factor in many social problems such as child or sexual abuse, automobile accidents, and assaults; alcohol also contributes to many illnesses and disease processes. Therefore, assessing for signs of hazardous alcohol use is important. The other options are not correct.

A patient with a middle ear infection asks the nurse, "What does the middle ear do?" The nurse responds by telling the patient that the middle ear functions to: a. Maintain balance. b. Interpret sounds as they enter the ear. c. Conduct vibrations of sounds to the inner ear. d. Increase amplitude of sound for the inner ear to function.

C Among its other functions, the middle ear conducts sound vibrations from the outer ear to the central hearing apparatus in the inner ear. The other responses are not functions of the middle

39. Which of these conditions is due to an inadequate intake of both protein and calories?

C) Marasmus

During an otoscopic examination, the nurse notices an area of black and white dots on the tympanic membrane and the ear canal wall. What does this finding suggest? a. Malignancy b. Viral infection c. Blood in the middle ear d. Yeast or fungal infection

D A colony of black or white dots on the drum or canal wall suggests a yeast or fungal infection (otomycosis).

What step of the nursing process includes data collection by health history, physical examination, and interview? a. Planning b. Diagnosis c. Evaluation d. Assessment

D Data collection, including performing the health history, physical examination, and interview, is the assessment step of the nursing process (see Figure 1-2).

32. The nurse is performing a nutritional assessment on an 80-year-old patient. The nurse knows that physiological changes that directly affect the nutritional status of the elderly include:

A) slowed gastrointestinal motility.

12. During a nutritional assessment, why is it important for the nurse to ask a patient what medications he or she is taking?

A) Certain drugs can affect the metabolism of nutrients.

The nurse is reviewing data collected after an assessment. Of the data listed below, which would be considered related cues that would be clustered together during data analysis? Select all that apply. a. Inspiratory wheezes noted in left lower lobes b. Hypoactive bowel sounds c. Nonproductive cough d. Edema, +2, noted on left hand e. Patient reports dyspnea upon exertion f. Rate of respirations 16 breaths per minute

A, C, E, F Clustering related cues help the nurse recognize relationships among the data. The cues related to the patients respiratory status (e.g., wheezes, cough, report of dyspnea, respiration rate and rhythm) are all related. Cues related to bowels and peripheral edema are not related to the respiratory cues.

In an interview, the nurse may find it necessary to take notes to aid his or her memory later. Which of the following statements is true regarding note-taking? 1.Taking notes may impede the nurse's observation of the patient's nonverbal behaviors. 2.Taking notes allows the patient to continue at his or her own pace as the nurse records what is said. 3.Taking notes allows the nurse to shift attention away from the patient, resulting in increased comfort level. 4.Taking notes allows the nurse to break eye contact with the patient, which may increase his or her level of comfort.

ANS: 1 Some use of history forms and note-taking may be unavoidable. But be aware that note-taking during the interview has disadvantages. It breaks eye contact too often; it shifts attention away from the patient, diminishing his or her sense of importance. It can interrupt the patient's narrative flow, and it impedes the observation of the patient's nonverbal behavior.

During an assessment of an infant, the nurse notes that the fontanels are depressed and sunken. The nurse suspects which condition? 1. Rickets 2. Dehydration 3. Mental retardation 4. Increased intracranial pressure

ANS: 2 Depressed and sunken fontanels occur with dehydration or malnutrition.

A male patient with a history of AIDS has come in for an examination and he states, "I think that I have the mumps." The nurse would begin by examining the: 1. thyroid gland. 2. parotid gland. 3. cervical lymph nodes. 4. mouth and skin for lesions.

ANS: 2 The parotid gland may become swollen with the onset of mumps, and parotid enlargement has been found with HIV.

A woman, Mrs. H., has just entered the emergency department after being battered by her husband. The nurse will need to get some information from her to begin treatment. What is the best choice for an opening with this patient? 1."Nancy, my name is Mrs. C." 2."Hello, Mrs. H., my name is Mrs. C. It sure is cold today!" 3."Mrs. H., my name is Mrs. C. You sure look like you're in pain, can I get you anything?" 4."Mrs. H., my name is Mrs. C. I'll need to ask you a few questions about what happened."

ANS: 4 Address the person, using his or her surname. Introduce yourself and state your role in the agency. Friendly small talk is not needed to build rapport.

The uterus is usually positioned tilting forward and superior to the bladder. This position is known as: A) anteverted and anteflexed. B) retroverted and anteflexed. C) retroverted and retroflexed. D) superiorverted and anteflexed.

ANS: A The uterus is freely movable, not fixed, and usually tilts forward and superior to the bladder (a position labeled as anteverted and anteflexed).

During a discussion for a men's health group, the nurse relates that the group with the highest incidence of prostate cancer is: A) Asian Americans. B) African-Americans. C) American Indians. D) Hispanics.

ANS: B According to the American Cancer Society (2010), African-American men have a higher rate of prostate cancer than other racial groups.

A 16-year-old boy is brought to the clinic for a problem that he refused to let his mother see. The nurse examines him, and finds that he has scrotal swelling on the left side. He had the mumps the previous week, and the nurse suspects that he has orchitis. Which of the assessment findings below support this diagnosis? Select all that apply. A) Swollen testis B) Mass does transilluminate C) Mass does not transilluminate D) Nontender upon palpation E) Tender upon palpation F) Scrotal skin is reddened

ANS: A, C, E, F With orchitis, the testis is swollen, with a feeling of weight, and is tender or painful. The mass does not transilluminate, and the scrotal skin is reddened. Transillumination of a mass occurs with a hydrocele, not orchitis.

A patient is complaining of severe knee pain after twisting it during a basketball game and is requesting pain medication. Which action by the nurse is appropriate?

ANS: Administer pain medication and then proceed with the assessment. According to the American Pain Society (1992), "In cases in which the cause of acute pain is uncertain, establishing a diagnosis is a priority, but symptomatic treatment of pain should be given while the investigation is proceeding. With occasional exceptions, (e.g., the initial examination of the patient with an acute condition of the abdomen), it is rarely justified to defer analgesia until a diagnosis is made. In fact, a comfortable patient is better able to cooperate with diagnostic procedures."

A 40-year-old woman reports a change in mole size, accompanied by color changes, itching, burning, and bleeding over the past month. She has a dark complexion and has no family history of skin cancer, but she has had many blistering sunburns in the past. The nurse would: a. Tell the patient to watch the lesion and report back in 2 months. b. Refer the patient because of the suggestion of melanoma on the basis of her symptoms. c. Ask additional questions regarding environmental irritants that may have caused this condition. d. Tell the patient that these signs suggest a compound nevus, which is very common in young to middle-aged adults.

ANS: B The ABCD danger signs of melanoma are asymmetry, border irregularity, color variation, and diameter. In addition, individuals may report a change in size, the development of itching, burning, and bleeding, or a new-pigmented lesion. Any one of these signs raises the suggestion of melanoma and warrants immediate referral.

During the examination portion of a patient's visit, she will be in lithotomy position. Which statement below reflects some things that the nurse can do to make this more comfortable for her? A) Ask her to place her hands and arms behind her head. B) Elevate her head and shoulders to maintain eye contact. C) Allow her to choose to have her feet in the stirrups or have them resting side by side on the edge of the table. D) Allow her to keep her buttocks about 6 inches from the edge of the table to prevent her from feeling as if she will fall off.

ANS: B The nurse should elevate her head and shoulders to maintain eye contact. The patient's arms should be placed at her sides or across the chest, not behind the head, because this position only tightens the abdominal muscles. The feet should be placed into the stirrups, knees apart, and buttocks at the edge of the examining table. Place the stirrups so the legs are not abducted too far

The nurse is assessing a patient who has liver disease for jaundice. Which of these assessment findings is indicative of true jaundice? a. Yellow patches in the outer sclera b. Yellow color of the sclera that extends up to the iris c. Skin that appears yellow when examined under low light d. Yellow deposits on the palms and soles of the feet where jaundice first appears

ANS: B The yellow sclera of jaundice extends up to the edge of the iris. Calluses on the palms and soles of the feet often appear yellow but are not classified as jaundice. Scleral jaundice should not be confused with the normal yellow subconjunctival fatty deposits that are common in the outer sclera of dark-skinned persons.

When performing the bimanual examination, the nurse notices that the cervix feels smooth and firm, is round, and is fixed in place (does not move). When cervical palpation is performed, the patient complains of some pain. The nurse's interpretation of these results should be which of these? A) These findings are all within normal limits. B) The cervical consistency should be soft and velvety—not firm. C) The cervix should move when palpated; an immobile cervix may indicate malignancy. D) Pain may occur during palpation of the cervix.

ANS: C Normally the cervix feels smooth and firm, as the consistency of the tip of the nose. It softens and feels velvety at 5 to 6 weeks of pregnancy (Goodell's sign). The cervix should be evenly rounded. With a finger on either side, the examiner should be able to move the cervix gently from side to side, and doing so should produce no pain for the patient. Hardness of the cervix may occur with malignancy. Immobility may occur with malignancy, and pain may occur with inflammation or ectopic pregnancy.

A black patient is in the intensive care unit because of impending shock after an accident. The nurse expects to find what characteristics in this patient's skin? a. Ruddy blue. b. Generalized pallor. c. Ashen, gray, or dull. d. Patchy areas of pallor.

ANS: C Pallor attributable to shock, with decreased perfusion and vasoconstriction, in black-skinned people will cause the skin to appear ashen, gray, or dull (see Table 12-2).

A woman states that 2 weeks ago she had a urinary tract infection that was treated with an antibiotic. As a part of the interview, the nurse should ask, "Have you noticed: A) a change in your urination patterns?" B) any excessive vaginal bleeding?" C) any unusual vaginal discharge or itching?" D) any changes in your desire for intercourse?"

ANS: C Several medications may increase the risk of vaginitis. Broad-spectrum antibiotics alter the balance of normal flora, which may lead to the development of vaginitis. The other questions are not correct

A patient is especially worried about an area of skin on her feet that has turned white. The health care provider has told her that her condition is vitiligo. The nurse explains to her that vitiligo is: a. Caused by an excess of melanin pigment b. Caused by an excess of apocrine glands in her feet c. Caused by the complete absence of melanin pigment d. Related to impetigo and can be treated with an ointment

ANS: C Vitiligo is the complete absence of melanin pigment in patchy areas of white or light skin on the face, neck, hands, feet, body folds, and around orifices—otherwise, the depigmented skin is normal.

When performing an assessment of a patient, the nurse notices the presence of an enlarged right epitrochlear lymph node. What should the nurse do next? a. Assess the patient's abdomen, and notice any tenderness. b. Carefully assess the cervical lymph nodes, and check for any enlargement. c. Ask additional health history questions regarding any recent ear infections or sore throats. d. Examine the patient's lower arm and hand, and check for the presence of infection or lesions.

ANS: Examine the patient's lower arm and hand, and check for the presence of infection or lesions. The epitrochlear nodes are located in the antecubital fossa and drain the hand and lower arm. The other actions are not correct for this assessment finding.

The nurse is examining a 2-month-old infant and notices asymmetry of the infant's gluteal folds. The nurse should assess for other signs of what disorder? A) Fractured clavicle B) Down syndrome C) Spina bifida D) Hip dislocation

ANS: Hip dislocation Unequal gluteal folds may accompany hip dislocation after 2 to 3 months of age, but some asymmetry may occur in healthy children. Further assessment is needed. The other responses are not correct.

During auscultation of breath sounds, the nurse should use the stethoscope correctly, in which of the following ways? a. Listening to at least one full respiration in each location b. Listening as the patient inhales and then going to the next site during exhalation c. Instructing the patient to breathe in and out rapidly while listening to the breath sounds d. If the patient is modest, listening to sounds over his or her clothing or hospital gown

ANS: Listen to at least one full respiration in each location. During auscultation of breath sounds with a stethoscope, it is important to listen to one full respiration in each location. During the examination, the nurse should monitor the breathing and offer times for the person to breathe normally to prevent possible dizziness.

The nurse is performing a well-child check on a 5-year-old boy. He has no current history that would lead the nurse to suspect illness. His medical history is unremarkable, and he received immunizations 1 week ago. Which of these findings should be considered normal in this situation? a. Enlarged, warm, and tender nodes b. Lymphadenopathy of the cervical nodes c. Palpable firm, small, shotty, mobile, and nontender lymph nodes d. Firm, rubbery, and large nodes, somewhat fixed to the underlying tissue

ANS: Palpable firm, small, shotty, mobile, nontender lymph nodes Palpable lymph nodes are often normal in children and infants. They are small, firm, shotty, mobile, and nontender. Vaccinations can produce lymphadenopathy. Enlarged, warm, tender nodes indicate current infection.

During auscultation of the lungs of an adult patient, the nurse notices the presence of bronchophony. The nurse should assess for signs of which condition? a. Airway obstruction b. Emphysema c. Pulmonary consolidation d. Asthma

ANS: Pulmonary consolidation Pathologic conditions that increase lung density, such as pulmonary consolidation, will enhance transmission of voice sounds, such as bronchophony. See Table 18-7.

Fibrous bands running directly from one bone to another that strengthen the joint and help prevent movement in undesirable directions are called: A) bursa. B) tendons. C) cartilage. D) ligaments.

ANS: ligaments. Fibrous bands running directly from one bone to another that strengthen the joint and help prevent movement in undesirable directions are called ligaments.

When performing a musculoskeletal assessment, the nurse knows that the correct approach for the examination should be: A) proximal to distal. B) distal to proximal. C) posterior to anterior. D) anterior to posterior.

ANS: proximal to distal. The musculoskeletal assessment should be done in an orderly approach, head to toe, proximal to distal, from the midline outward. The other options are not correct.

The nurse suspects that a patient has otitis media. Early signs of otitis media include which of these findings of the tympanic membrane? a. Red and bulging b. Hypomobility c. Retraction with landmarks clearly visible d. Flat, slightly pulled in at the center, and moves with insufflation

B An early sign of otitis media is hypomobility of the tympanic membrane. As pressure increases, the tympanic membrane begins to bulge.

Which critical thinking skill helps the nurse see relationships among the data? a. Validation b. Clustering related cues c. Identifying gaps in data d. Distinguishing relevant from irrelevant

B Clustering related cues helps the nurse see relationships among the data.

7. During a nutritional assessment of a 22-year-old male refugee, the nurse must remember to:

B) clarify what is meant by the term "food."

The nurse is examining a patient's ears and notices cerumen in the external canal. Which of these statements about cerumen is correct? a. Sticky honey-colored cerumen is a sign of infection. b. The presence of cerumen is indicative of poor hygiene. c. The purpose of cerumen is to protect and lubricate the ear. d. Cerumen is necessary for transmitting sound through the auditory canal.

C The ear is lined with glands that secrete cerumen, which is a yellow waxy material that lubricates and protects the ear.

A patient tells the nurse that she has had abdominal pain for the past week. What would be the nurse's best response? a. "Can you point to where it hurts?" b. "We'll talk more about that later in the interview." c. "What have you had to eat in the last 24 hours?" d. "Have you ever had any surgeries on your abdomen

*a. "Can you point to where it hurts?"* A final summary of any symptom the person has should include, along with seven other critical characteristics, "Location: specific." The person is asked to point to the location.

During assessment of infants and children, the nurse measures the head circumference and compares the measure to the chest circumference. For each finding listed below, match to the appropriate age. 1. Newborn infant 2. Toddler, age 2 years 3. Child, age 4 years

1. Head circumference equal to chest circumference. 2. Head circumference greater than chest circumference. 3. Head circumference less than chest circumference. ANS:2,1,3

Put the following patient situations in order according to the level of priority. a. A patient newly diagnosed with type 2 diabetes mellitus does not know how to check his own blood glucose levels with a glucometer. b. A teenager who was stung by a bee during a soccer match is having trouble breathing. c. An older adult with a urinary tract infection is also showing signs of confusion and agitation.

1. a = First-level priority problem 2. b = Second-level priority problem 3. c = Third-level priority problem

A patient is at the clinic to have her blood pressure checked. She has been coming to the clinic weekly since she changed medications 2 months ago. The nurse should: a. Collect a follow-up data base and then check her blood pressure. b. Ask her to read her health record and indicate any changes since her last visit. c. Check only her blood pressure because her complete health history was documented 2 months ago. d. Obtain a complete health history before checking her blood pressure because much of her history information may have changed.

A A follow-up data base is used in all settings to follow up short-term or chronic health problems. The other responses are not appropriate for the situation

In the assessment of a 1-month-old infant, the nurse notices a lack of response to noise or stimulation. The mother reports that in the last week he has been sleeping all of the time, and when he is awake all he does is cry. The nurse hears that the infant's cries are very high pitched and shrill. What should be the nurse's appropriate response to these findings? a.Refer the infant for further testing. b.Talk with the mother about eating habits. c.Do nothing; these are expected findings for an infant this age. d.Tell the mother to bring the baby back in 1 week for a recheck.

A A high-pitched, shrill cry or cat-sounding screech occurs with central nervous system damage. Lethargy, hyporeactivity, and hyperirritability, as well as the parents report of significant changes in behavior all warrant referral. The other options are not correct responses.

A man is at the clinic for a physical examination. He states that he is "very anxious" about the physical examination. What steps can the nurse take to make him more comfortable? a.Appear unhurried and confident when examining him. b.Stay in the room when he undresses in case he needs assistance. c.Ask him to change into an examining gown and to take off his undergarments. d.Defer measuring vital signs until the end of the examination, which allows him time to become comfortable.

A Anxiety can be reduced by an examiner who is confident, self-assured, considerate, and unhurried. Familiar and relatively nonthreatening actions, such as measuring the person's vital signs, will gradually accustom the person to the examination.

During an assessment of a 20-year-old patient with a 3-day history of nausea and vomiting, the nurse notices dry mucosa and deep vertical fissures in the tongue. These findings are reflective of: a.Dehydration. b.Irritation by gastric juices. c.A normal oral assessment. d.Side effects from nausea medication

A Dry mouth occurs with dehydration or fever. The tongue has deep vertical fissures.

The nurse is performing an oral assessment on a 40-year-old Black patient and notices the presence of a 1 cm, nontender, grayish-white lesion on the left buccal mucosa. Which one of these statements is true? This lesion is: a.Leukoedema and is common in dark-pigmented persons. b.The result of hyperpigmentation and is normal. c.Torus palatinus and would normally be found only in smokers. d.Indicative of cancer and should be immediately tested.

A Leukoedema, a grayish-white benign lesion occurring on the buccal mucosa, is most often observed in Blacks.

The nurse will use which technique of assessment to determine the presence of crepitus, swelling, and pulsations? a.Palpation b.Inspection c.Percussion d.Auscultation

A Palpation applies the sense of touch to assess texture, temperature, moisture, organ location and size, as well as any swelling, vibration or pulsation, rigidity or spasticity, crepitation, presence of lumps or masses, and the presence of tenderness or pain

The nurse is preparing to use a stethoscope for auscultation. Which statement is true regarding the diaphragm of the stethoscope? The diaphragm: a.Is used to listen for high-pitched sounds. b.Is used to listen for low-pitched sounds. c.Should be lightly held against the person's skin to block out low-pitched sounds. d.Should be lightly held against the person's skin to listen for extra heart sounds and murmurs.

A The diaphragm of the stethoscope is best for listening to high-pitched sounds such as breath, bowel, and normal heart sounds. It should be firmly held against the person's skin, firmly enough to leave a ring. The bell of the stethoscope is best for soft, low-pitched sounds such as extra heart sounds or murmurs.

The salivary gland that is the largest and located in the cheek in front of the ear is the _________ gland. a.Parotid b.Stensen's c.Sublingual d.Submandibular

A The mouth contains three pairs of salivary glands. The largest, the parotid gland, lies within the cheeks in front of the ear extending from the zygomatic arch down to the angle of the jaw. The Stensen's duct (not gland) drains the parotid gland onto the buccal mucosa opposite the second molar. The sublingual gland is located within the floor of the mouth under the tongue. The submandibular gland lies beneath the mandible at the angle of the jaw.

A 59-year-old patient has a herniated intervertebral disk. Which of the following findings should the nurse expect to see on physical assessment of this individual? a.Hyporeflexia b.Increased muscle tone c.Positive Babinski sign d.Presence of pathologic reflexes

A With a herniated intervertebral disk or lower motor neuron lesion, loss of tone, flaccidity, atrophy, fasciculations, and hyporeflexia or areflexia are demonstrated. No Babinski sign or pathologic reflexes would be observed. The other options reflect a lesion of upper motor neurons.

18. If a 29-year-old woman weighs 156 pounds and the nurse determines her ideal body weight to be 120 pounds, how would the nurse classify the woman's weight?

A) Obese

11. The nurse is providing care for a 68-year-old woman who is complaining of constipation. What concern exists regarding her nutritional status?

A) The absorption of nutrients may be impaired.

During a discussion about breast self-examination with a 30-year-old woman, which of these statements by the nurse is most appropriate? a. "The best time to examine your breasts is during ovulation." b. "Examine your breasts every month on the same day of the month." c. "Examine your breasts shortly after your menstrual period each month." d. "The best time to examine your breasts is immediately before menstruation."

ANS: "Examine your breasts shortly after your menstrual period each month." The best time to conduct breast self-examination is shortly after the menstrual period when the breasts are the smallest and least congested.

16. During an assessment of a 68-year-old man with a recent onset of right-sided weakness, the nurse hears a blowing, swishing sound with the bell of the stethoscope over the left carotid artery. This finding would indicate: A. A valvular disorder. B. Blood flow turbulence. C. Fluid volume overload. D. Ventricular hypertrophy.

ANS: B A bruit is a blowing, swishing sound indicating blood flow turbulence; normally none is present.

When assessing muscle strength, the nurse observes that a patient has complete range of motion against gravity with full resistance. What Grade should the nurse record using a 0 to 5 point scale? A) 2 B) 3 C) 4 D) 5

ANS: 5 Complete range of motion against gravity is normal muscle strength and is recorded as Grade 5 muscle strength.

The nurse is preparing to perform a physical assessment. Which statement is true about the physical assessment? The inspection phase: a.Usually yields little information. b.Takes time and reveals a surprising amount of information. c.May be somewhat uncomfortable for the expert practitioner. d.Requires a quick glance at the patient's body systems before proceeding with palpation.

B A focused inspection takes time and yields a surprising amount of information. Initially, the examiner may feel uncomfortable, staring at the person without also doing something. A focused assessment is significantly more than a "quick glance."

The ability that humans have to perform very skilled movements such as writing is controlled by the: a.Basal ganglia. b.Corticospinal tract. c.Spinothalamic tract. d.Extrapyramidal tract.

B Corticospinal fibers mediate voluntary movement, particularly very skilled, discrete, and purposeful movements, such as writing. The corticospinal tract, also known as the pyramidal tract, is a newer, higher motor system that humans have that permits very skilled and purposeful movements. The other responses are not related to skilled movements.

A patient has suddenly developed shortness of breath and appears to be in significant respiratory distress. After calling the physician and placing the patient on oxygen, which of these actions is the best for the nurse to take when further assessing the patient? a.Count the patient's respirations. b.Bilaterally percuss the thorax, noting any differences in percussion tones. c.Call for a chest x-ray study, and wait for the results before beginning an assessment. d.Inspect the thorax for any new masses and bleeding associated with respirations.

B Percussion is always available, portable, and offers instant feedback regarding changes in underlying tissue density, which may yield clues of the patient's physical status.

The nurse is examining an infant and prepares to elicit the Moro reflex at which time during the examination? a.When the infant is sleeping b.At the end of the examination c.Before auscultation of the thorax d.Halfway through the examination

B The Moro or startle reflex is elicited at the end of the examination because it may cause the infant to cry.

During the neurologic assessment of a "healthy" 35-year-old patient, the nurse asks him to relax his muscles completely. The nurse then moves each extremity through full range of motion. Which of these results would the nurse expect to find? a.Firm, rigid resistance to movement b.Mild, even resistance to movement c.Hypotonic muscles as a result of total relaxation d.Slight pain with some directions of movement

B Tone is the normal degree of tension (contraction) in voluntarily relaxed muscles. It shows a mild resistance to passive stretching. Normally, the nurse will notice a mild, even resistance to movement. The other responses are not correct.

44. The nurse is assessing the body weight as a percentage of ideal body weight on an adolescent patient who was admitted for suspected anorexia nervosa. The patient's usual weight was 125 pounds, but today she weighs 98 pounds. The nurse calculates the patient's ideal body weight, and reaches which conclusion?

B) She is experiencing moderate malnutrition.

17. An older patient has been diagnosed with pernicious anemia. The nurse knows that this condition could be related to: a. Increased gastric acid secretion. b. Decreased gastric acid secretion. c. Delayed gastrointestinal emptying time. d. Increased gastrointestinal emptying time.

B) decreased gastric acid secretion. Gastric acid secretion decreases with aging and may cause pernicious anemia (because it interferes with vitamin B12 absorption), iron-deficiency anemia, and malabsorption of calcium.

20. The nurse knows that during an abdominal assessment, deep palpation is used to determine: a. Bowel motility. b. Enlarged organs. c. Superficial tenderness. d. Overall impression of skin surface and superficial musculature.

B) enlarged organs. With deep palpation, the nurse should notice the location, size, consistency, and mobility of any palpable organs and the presence of any abnormal enlargement, tenderness, or masses.

A 69-year-old patient has been admitted to an adult psychiatric unit because his wife thinks he is getting more and more confused. He laughs when he is found to be forgetful, saying "I'm just getting old!" After the nurse completes a thorough neurologic assessment, which findings would be indicative of Alzheimer disease? Select all that apply. a.Occasionally forgetting names or appointments b.Difficulty performing familiar tasks, such as placing a telephone call c.Misplacing items, such as putting dish soap in the refrigerator d.Sometimes having trouble finding the right word e.Rapid mood swings, from calm to tears, for no apparent reason f.Getting lost in one's own neighborhood

B, C, E, F Difficulty performing familiar tasks, misplacing items, rapid mood swings, and getting lost in ones own neighborhood can be warning signs of Alzheimer disease. Occasionally forge

A patient has a severed spinal nerve as a result of trauma. Which statement is true in this situation? a.Because there are 31 pairs of spinal nerves, no effect results if only one nerve is severed. b.The dermatome served by this nerve will no longer experience any sensation. c.The adjacent spinal nerves will continue to carry sensations for the dermatome served by the severed nerve. d.A severed spinal nerve will only affect motor function of the patient because spinal nerves have no sensory component.

C A dermatome is a circumscribed skin area that is primarily supplied from one spinal cord segment through a particular spinal nerve. The dermatomes overlap, which is a form of biologic insurance; that is, if one nerve is severed, then most of the sensations can be transmitted by the spinal nerve above and the spinal nerve below the severed nerve.

When taking the health history on a patient with a seizure disorder, the nurse assesses whether the patient has an aura. Which of these would be the best question for obtaining this information? a."Does your muscle tone seem tense or limp?" b."After the seizure, do you spend a lot of time sleeping?" c."Do you have any warning sign before your seizure starts?" d."Do you experience any color change or incontinence during the seizure?"

C Aura is a subjective sensation that precedes a seizure; it could be auditory, visual, or motor. The other questions do not solicit information about an aura.

Immediately after birth, the nurse is unable to suction the nares of a newborn. An attempt is made to pass a catheter through both nasal cavities with no success. What should the nurse do next? a.Attempt to suction again with a bulb syringe. b.Wait a few minutes, and try again once the infant stops crying. c.Recognize that this situation requires immediate intervention. d.Contact the physician to schedule an appointment for the infant at his or her next hospital visit.

C Determining the patency of the nares in the immediate newborn period is essential because most newborns are obligate nose breathers. Nares blocked with amniotic fluid are gently suctioned with a bulb syringe. If obstruction is suspected, then a small lumen (5 to 10 Fr) catheter is passed down each naris to confirm patency. The inability to pass a catheter through the nasal cavity indicates choanal atresia, which requires immediate intervention.

Which of these statements represents subjective data the nurse obtained from the patient regarding the patient's skin? a.Skin appears dry. b.No lesions are obvious. *c.Patient denies any color change.* d.Lesion is noted on the lateral aspect of the right arm.

C The history should be limited to patient statements or subjective data—factors that the person says were or were not present.

The nurse is taking a family history. Important diseases or problems about which the patient should be specifically asked include: a.Emphysema. b.Head trauma. *c.Mental illness.* d.Fractured bones.

C Questions concerning any family history of heart disease, high blood pressure, stroke, diabetes, obesity, blood disorders, breast and ovarian cancers, colon cancer, sickle cell anemia, arthritis, allergies, alcohol or drug addiction, mental illness, suicide, seizure disorder, kidney disease, and tuberculosis should be asked.

A patient is unable to perform rapid alternating movements such as rapidly patting her knees. The nurse should document this inability as: a.Ataxia. b.Astereognosis. c.Presence of dysdiadochokinesia. d.Loss of kinesthesia.

C Slow clumsy movements and the inability to perform rapid alternating movements occur with cerebellar disease. The condition is termed dysdiadochokinesia. Ataxia is an uncoordinated or unsteady gait. Astereognosis is the inability to identify an object by feeling it. Kinesthesia is the persons ability to perceive passive movement of the extremities or the loss of position sense.

During an oral assessment of a 30-year-old Black patient, the nurse notices bluish lips and a dark line along the gingival margin. What action would the nurse perform in response to this finding? a.Check the patient's hemoglobin for anemia. b.Assess for other signs of insufficient oxygen supply. c.Proceed with the assessment, knowing that this appearance is a normal finding. d.Ask if he has been exposed to an excessive amount of carbon monoxide.

C Some Blacks may have bluish lips and a dark line on the gingival margin; this appearance is a normal finding.

The projections in the nasal cavity that increase the surface area are called the: a.Meatus. b.Septum. c.Turbinates. d.Kiesselbach plexus.

C The lateral walls of each nasal cavity contain three parallel bony projections: the superior, middle, and inferior turbinates. These increase the surface area, making more blood vessels and mucous membrane available to warm, humidify, and filter the inhaled air.

The most important step that the nurse can take to prevent the transmission of microorganisms in the hospital setting is to: a.Wear protective eye wear at all times. b.Wear gloves during any and all contact with patients. c.Wash hands before and after contact with each patient. d.Clean the stethoscope with an alcohol swab between patients.

C The most important step to decrease the risk of microorganism transmission is to wash hands promptly and thoroughly before and after physical contact with each patient. Stethoscopes should also be cleansed with an alcohol swab before and after each patient contact. The best routine is to combine stethoscope rubbing with hand hygiene each time hand hygiene is performed

When the nurse is testing the triceps reflex, what is the expected response? a.Flexion of the hand b.Pronation of the hand c.Extension of the forearm d.Flexion of the forearm

C The normal response of the triceps reflex is extension of the forearm. The normal response of the biceps reflex causes flexion of the forearm. The other responses are incorrect.

26. The nurse is reviewing the assessment of an aortic aneurysm. Which of these statements is true regarding an aortic aneurysm? a. A bruit is absent. b. Femoral pulses are increased. c. A pulsating mass is usually present. d. Most are located below the umbilicus.

C) A pulsating mass is usually present. Most aortic aneurysms are palpable during routine examination and feel like a pulsating mass. A bruit will be audible, and femoral pulses are present but decreased. Such aneurysms are located in the upper abdomen just to the left of midline.

22. During an abdominal assessment, the nurse elicits tenderness on light palpation in the right lower quadrant. The nurse interprets that this finding could indicate a disorder of which of these structures? a. Spleen b. Sigmoid c. Appendix d. Gallbladder

C) Appendix The appendix is located in the right lower quadrant. When the iliopsoas muscle is inflamed, which occurs with an inflamed or perforated appendix, pain is felt in the right lower quadrant.

22. The nurse is concerned about the skeletal protein reserves of a patient who has been hospitalized frequently for chronic lung disease. Which of these measurements would be necessary to include in the assessment?

C) Mid-arm muscle area

6. The nurse is assessing a 30-year-old unemployed immigrant from an underdeveloped country who has been in the United States for 1 month. Which of these problems related to his nutritional status might the nurse expect to find?

C) Osteomalacia

40. A pregnant woman who is HIV positive is asking the nurse about breastfeeding her infant. Which of these statements is true?

C) Women who are HIV positive should not breastfeed because HIV can be transmitted through breast milk.

While the nurse is taking the history of a 68-year-old patient who sustained a head injury 3 days earlier, he tells the nurse that he is on a cruise ship and is 30 years old. The nurse knows that this finding is indicative of a(n): a.Great sense of humor. b.Uncooperative behavior. c.Inability to understand questions. d.Decreased level of consciousness.

D A change in consciousness may be subtle. The nurse should notice any decreasing level of consciousness, disorientation, memory loss, uncooperative behavior, or even complacency in a previously combative person. The other responses are incorrect.

Which of these statements about the peripheral nervous system is correct? a.The CNs enter the brain through the spinal cord. b.Efferent fibers carry sensory input to the central nervous system through the spinal cord. c.The peripheral nerves are inside the central nervous system and carry impulses through their motor fibers. d.The peripheral nerves carry input to the central nervous system by afferent fibers and away from the central nervous system by efferent fibers.

D A nerve is a bundle of fibers outside of the central nervous system. The peripheral nerves carry input to the central nervous system by their sensory afferent fibers and deliver output from the central nervous system by their efferent fibers. The other responses are not related to the peripheral nervous system.

The nurse is preparing to assess a hospitalized patient who is experiencing significant shortness of breath. How should the nurse proceed with the assessment? a.The patient should lie down to obtain an accurate cardiac, respiratory, and abdominal assessment. b.A thorough history and physical assessment information should be obtained from the patient's family member. c.A complete history and physical assessment should be immediately performed to obtain baseline information. d.Body areas appropriate to the problem should be examined and then the assessment completed after the problem has resolved.

D Both altering the position of the patient during the examination and collecting a mini database by examining the body areas appropriate to the problem may be necessary in this situation. An assessment may be completed later after the distress is resolved.

When examining a 16-year-old male teenager, the nurse should: a.Discuss health teaching with the parent because the teen is unlikely to be interested in promoting wellness. b.Ask his parent to stay in the room during the history and physical examination to answer any questions and to alleviate his anxiety. c.Talk to him the same manner as one would talk to a younger child because a teen's level of understanding may not match his or her speech. d.Provide feedback that his body is developing normally, and discuss the wide variation among teenagers on the rate of growth and development.

D During the examination, the adolescent needs feedback that his or her body is healthy and developing normally. The adolescent has a keen awareness of body image and often compares him or herself with peers. Apprise the adolescent of the wide variation among teenagers on the rate of growth and development.

A 32-year-old woman is at the clinic for "little white bumps in my mouth." During the assessment, the nurse notes that she has a 0.5 cm white, nontender papule under her tongue and one on the mucosa of her right cheek. What would the nurse tell the patient? a."These spots indicate an infection such as strep throat." b."These bumps could be indicative of a serious lesion, so I will refer you to a specialist." c."This condition is called leukoplakia and can be caused by chronic irritation such as with smoking." d."These bumps are Fordyce granules, which are sebaceous cysts and are not a serious condition."

D Fordyce granules are small, isolated white or yellow papules on the mucosa of the cheek, tongue, and lips. These little sebaceous cysts are painless and are not significant. Chalky, white raised patches would indicate leukoplakia. In strep throat, the examiner would see tonsils that are bright red, swollen, and may have exudates or white spots.

A patient tells the nurse that he is allergic to penicillin. What would be the nurse's best response to this information? a."Are you allergic to any other drugs?" b."How often have you received penicillin?" c."I'll write your allergy on your chart so you won't receive any penicillin." *d."Describe what happens to you when you take penicillin*

D Note both the allergen (medication, food, or contact agent, such as fabric or environmental agent) and the reaction (rash, itching, runny nose, watery eyes, or difficulty breathing). With a drug, this symptom should not be a side effect but a true allergic reaction.

The tissue that connects the tongue to the floor of the mouth is the: a.Uvula. b.Palate. c.Papillae. d.Frenulum.

D The frenulum is a midline fold of tissue that connects the tongue to the floor of the mouth. The uvula is the free projection hanging down from the middle of the soft palate. The palate is the arching roof of the mouth. Papillae are the rough, bumpy elevations on the tongue's dorsal surface.

During an assessment of a 32-year-old patient with a recent head injury, the nurse notices that the patient responds to pain by extending, adducting, and internally rotating his arms. His palms pronate, and his lower extremities extend with plantar flexion. Which statement concerning these findings is most accurate? This patient's response: a.Indicates a lesion of the cerebral cortex. b.Indicates a completely nonfunctional brainstem. c.Is normal and will go away in 24 to 48 hours. d.Is a very ominous sign and may indicate brainstem injury.

D These findings are all indicative of decerebrate rigidity, which is a very ominous condition and may indicate a brainstem injury.

In obtaining a review of systems on a "healthy" 7-year-old girl, the health care provider knows that it would be important to include the: a.Last glaucoma examination. b.Frequency of breast self-examinations. c.Date of her last electrocardiogram. *d.Limitations related to her involvement in sports activities.*

D When reviewing the cardiovascular system, the health care provider should ask whether any activity is limited or whether the child can keep up with her peers. The other items are not appropriate for a child this age.

23. The nurse is measuring a patient's frame size. Which of these statements best describes the correct technique for measuring frame size?

D) With the right arm extended forward and the elbow bent, use the calipers to measure the distance between the condyles of the humerus.

5. The nurse is aware that one change that may occur in the gastrointestinal system of an aging adult is: a. Increased salivation. b. Increased liver size. c. Increased esophageal emptying. d. Decreased gastric acid secretion.

D) decreased gastric acid secretion. Gastric acid secretion decreases with aging. As one ages, salivation decreases, esophageal emptying is delayed, and liver size decreases

After completing an initial assessment of a patient, the nurse has charted that his respirations are eupneic and his pulse is 58 beats per minute. These types of data would be: a. Objective. b. Reflective. c. Subjective. d. Introspective.

A Objective data are what the health professional observes by inspecting, percussing, palpating, and auscultating during the physical examination. Subjective data is what the person says about him or herself during history taking. The terms reflective and introspective are not used to describe data

A 59-year-old patient tells the nurse that he has ulcerative colitis. He has been having "black stools" for the last 24 hours. How would the nurse best document his reason for seeking care? a. J.M. is a 59-year-old man seeking treatment for ulcerative colitis. b. J.M. came into the clinic complaining of having black stools for the past 24 hours. c. J.M. is a 59-year-old man who states that he has ulcerative colitis and wants it checked. d. J.M. is a 59-year-old man who states that he has been having "black stools" for the past 24 hours.

*d. J.M. is a 59-year-old man who states that he has been having "black stools" for the past 24 hours.* The reason for seeking care is a brief spontaneous statement in the person's own words that describes the reason for the visit. It states one (possibly two) signs or symptoms and their duration. It is enclosed in quotation marks to indicate the person's exact words.

A patient has been shown to have a sensorineural hearing loss. During the assessment, it would be important for the nurse to: a. Speak loudly so the patient can hear the questions. b. Assess for middle ear infection as a possible cause. c. Ask the patient what medications he is currently taking. d. Look for the source of the obstruction in the external ear.

C A simple increase in amplitude may not enable the person to understand spoken words. Sensorineural hearing loss may be caused by presbycusis, which is a gradual nerve degeneration that occurs with aging and by ototoxic drugs, which affect the hair cells in the

A 42-year-old woman complains that she has noticed several small, slightly raised, bright red dots on her chest. On examination, the nurse expects that the spots are probably: a. Anasarca. b. Scleroderma. c. Senile angiomas. d. Latent myeloma.

ANS: C Cherry (senile) angiomas are small, smooth, slightly raised bright red dots that commonly appear on the trunk of adults over 30 years old.

When assessing the tongue of an adult, the nurse knows that an abnormal finding would be: a.Smooth glossy dorsal surface. b.Thin white coating over the tongue. c.Raised papillae on the dorsal surface. d.Visible venous patterns on the ventral surface.

A The dorsal surface of the tongue is normally roughened from papillae. A thin white coating may be present. The ventral surface may show veins. Smooth, glossy areas may indicate atrophic glossitis (see Table 16-5).

The wife of a 65-year-old man tells the nurse that she is concerned because she has noticed a change in her husband's personality and ability to understand. He also cries very easily and becomes angry. The nurse recalls that the cerebral lobe responsible for these behaviors is the __________ lobe. a.Frontal b.Parietal c.Occipital d.Temporal

A The frontal lobe has areas responsible for personality, behavior, emotions, and intellectual function. The parietal lobe has areas responsible for sensation; the occipital lobe is responsible for visual reception; and the temporal lobe is responsible for hearing, taste, and smell.

The nurse is performing an ear examination of an 80-year-old patient. Which of these findings would be considered normal? a. High-tone frequency loss b. Increased elasticity of the pinna c. Thin, translucent membrane d. Shiny, pink tympanic membrane

A A high-tone frequency hearing loss is apparent for those affected with presbycusis, the hearing loss that occurs with aging. The pinna loses elasticity, causing earlobes to be pendulous. The eardrum may be whiter in color and more opaque and duller in the older person than in the younger adult.

14. The nurse is performing a nutritional assessment on a 15-year-old girl, who tells the nurse that she is "so fat." Assessment reveals that she is 5 feet 4 inches and weighs 110 pounds. The nurse's appropriate response would be:

A) "How much do you think you should weigh?"

While performing an assessment of a 65-year-old man with a history of hypertension and coronary artery disease, the nurse notices the presence of bilateral pitting edema in the lower legs. The skin is puffy and tight but normal in color. No increased redness or tenderness is observed over his lower legs, and the peripheral pulses are equal and strong. In this situation, the nurse suspects that the likely cause of the edema is which condition? a. Heart failure b. Venous thrombosis c. Local inflammation d. Blockage of lymphatic drainage

ANS: A Bilateral edema or edema that is generalized over the entire body is caused by a central problem such as heart failure or kidney failure. Unilateral edema usually has a local or peripheral cause.

The nurse just noted from the medical record that the patient has a lesion that is confluent in nature. On examination, the nurse expects to find: a. Lesions that run together. b. Annular lesions that have grown together. c. Lesions arranged in a line along a nerve route. d. Lesions that are grouped or clustered together.

ANS: A Confluent lesions (as with urticaria [hives]) run together. Grouped lesions are clustered together. Annular lesions are circular in nature. Zosteriform lesions are arranged along a nerve route.

A 59-year-old patient has been diagnosed with prostatitis and is being seen at the clinic for complaints of burning and pain during urination. He is experiencing: A) dysuria. B) nocturia. C) polyuria. D) hematuria.

ANS: A Dysuria or burning with urination is common with acute cystitis, prostatitis, and urethritis. Nocturia is voiding during the night. Polyuria is voiding in excessive quantities. Hematuria is voiding with blood in the urine

A male patient with possible fertility problems asks the nurse where sperm is produced. The nurse knows that sperm production occurs in the: A) testes. B) prostate. C) epididymis. D) vas deferens.

ANS: A Sperm production occurs in the testes, not in the other structures listed.

A 62-year-old man is experiencing fever, chills, malaise, urinary frequency, and urgency. He also reports urethral discharge and a dull aching pain in the perineal and rectal area. These symptoms are most consistent with which of the following? A) Prostatitis B) A polyp C) Carcinoma of the prostate D) Benign prostatic hypertrophy (BPH)

ANS: A The common presenting symptoms of prostatitis are fever, chills, malaise, and urinary frequency and urgency. The individual may also have dysuria, urethral discharge, and a dull aching pain in the perineal and rectal area. See Table 25-3 for descriptions of carcinoma of the prostate and BPH. These are not the symptoms of a polyp.

The nurse is examining a patient who tells the nurse, "I sure sweat a lot, especially on my face and feet but it doesn't have an odor." The nurse knows that this condition could be related to: a. Eccrine glands. b. Apocrine glands. c. Disorder of the stratum corneum. d. Disorder of the stratum germinativum.

ANS: A The eccrine glands are coiled tubules that directly open onto the skin surface and produce a dilute saline solution called sweat. Apocrine glands are primarily located in the axillae, anogenital area, nipples, and naval area and mix with bacterial flora to produce the characteristic musky body odor. The patient's statement is not related to disorders of the stratum corneum or the stratum germinativum.

Which of these statements is most appropriate when the nurse is obtaining a genitourinary history from an elderly man? A) "Do you need to get up at night to urinate?" B) "Do you experience nocturnal emissions, or 'wet dreams'?" C) "Do you know how to perform a testicular self-examination?" D) "Has anyone ever touched your genitals when you did not want them to?"

ANS: A The elderly male patient should be asked about the presence of nocturia. This may be due to diuretic medication, fluid retention from mild heart failure or varicose veins, or fluid ingestion 3 hours before bedtime, especially coffee and alcohol. The other questions are more appropriate for younger males.

The nurse just noted from the medical record that the patient has a lesion that is confluent in nature. On examination, the nurse expects to find: a. Lesions that run together. b. Annular lesions that have grown together. c. Lesions arranged in a line along a nerve route. d. Lesions that are grouped or clustered together.

ANS: A Confluent lesions (as with urticaria [hives]) run together. Grouped lesions are clustered together. Annular lesions are circular in nature. Zosteriform lesions are arranged along a nerve route. DIF: Cognitive Level: Understanding (Comprehension) MSC: Client Needs: Physiologic Integrity: Physiologic Adaptation

. A 45-year-old farmer comes in for a skin evaluation and complains of hair loss on his head. His hair seems to be breaking off in patches, and he notices some scaling on his head. The nurse begins the examination suspecting: a. Tinea capitis. b. Folliculitis. c. Toxic alopecia. d. Seborrheic dermatitis.

ANS: A Tinea capitis is rounded patchy hair loss on the scalp, leaving broken-off hairs, pustules, and scales on the skin, and is caused by a fungal infection. Lesions are fluorescent under a Wood light and are usually observed in children and farmers; tinea capitis is highly contagious. DIF: Cognitive Level: Analyzing (Analysis) MSC: Client Needs: Physiologic Integrity: Physiologic Adaptation

1. While measuring a patients blood pressure, the nurse uses the proper technique to obtain an accurate reading. Which of these situations will result in a falsely high blood pressure reading? Select all that apply. a. The person supports his or her own arm during the blood pressure reading. b. The blood pressure cuff is too narrow for the extremity. c. The arm is held above level of the heart. d. The cuff is loosely wrapped around the arm. e. The person is sitting with his or her legs crossed. f. The nurse does not inflate the cuff high enough.

ANS: A, B, D, E Several factors can result in blood pressure readings that are too high or too low. Having the patients arm held above the level of the heart is one part of the correct technique. (Refer to Table 9-5, Common Errors in Blood Pressure Measurement.)

The nurse is performing a digital examination of a patient's prostate gland and notices that characteristics of a normal prostate gland include which of the following? Select all that apply. A) The gland protruding 1 cm into the rectum B) Heart-shaped with a palpable central groove C) Flat with no groove palpable D) Boggy and soft consistency E) Smooth surface, elastic, or rubbery consistency F) Fixed mobility

ANS: A, B, E The size should be 2.5 cm long by 4 cm wide, and it should not protrude more than 1 cm into the rectum. The prostate should be heart-shaped, with a palpable central groove, a smooth surface, and elastic, rubbery consistency. Abnormal findings include a flat shape with no palpable groove, boggy with a soft consistency, and fixed mobility.

A 55-year-old man is in the clinic for a yearly check-up. He is worried because his father died of prostate cancer. The nurse knows that which tests should be done at this time? Select all that apply. A) Blood test for prostate-specific antigen B) Urinalysis C) Transrectal ultrasound D) Digital rectal examination E) Prostate biopsy

ANS: A, D Prostate cancer is typically detected by testing the blood for prostate-specific antigen (PSA) or by a digital rectal exam (DRE). It is recommended that both PSA and DRE be offered to men yearly, beginning at age 50 years. If the PSA is elevated, then further lab work or a transrectal ultrasound (TRUS) and biopsy may be recommended.

When performing a genital examination on a 25-year-old man, the nurse notices deeply pigmented, wrinkled scrotal skin with large sebaceous follicles. On the basis of this information the nurse would: A) squeeze the glans to check for the presence of discharge. B) consider this a normal finding and proceed with the examination. C) assess the testicles for the presence of masses or painless lumps. D) obtain a more detailed history focusing on any scrotal abnormalities the patient has noticed.

ANS: B After adolescence, the scrotal skin is deeply pigmented and has large sebaceous follicles. The scrotal skin looks corrugated.

A 52-year-old woman has a papule on her nose that has rounded, pearly borders and a central red ulcer. She said she first noticed it several months ago and that it has slowly grown larger. The nurse suspects which condition? a. Acne b. Basal cell carcinoma c. Melanoma d. Squamous cell carcinoma

ANS: B Basal cell carcinoma usually starts as a skin-colored papule that develops rounded, pearly borders with a central red ulcer. It is the most common form of skin cancer and grows slowly. This description does not fit acne lesions. (See Table 12-11 for descriptions of melanoma and squamous cell carcinoma.)

After completing an assessment of a 60-year-old man with a family history of colon cancer, the nurse discusses with him early detection measures for colon cancer. The nurse should mention the need for a(n): A) annual proctoscopy. B) colonoscopy every 10 years. C) fecal test for blood every 6 months. D) digital rectal examinations every 2 years.

ANS: B Early detection measures for colon cancer include a digital rectal examination performed annually after age 50 years, a fecal occult blood test annually after age 50 years, sigmoidoscopy every 5 years or colonoscopy every 10 years after age 50 years; and a PSA blood test annually for men over 50 years old, except black men beginning at age 45 years (American Cancer Society, 2006).

A mother has noticed that her son, who has been to a new babysitter, has some blisters and scabs on his face and buttocks. On examination, the nurse notices moist, thin-roofed vesicles with a thin erythematous base and suspects: a. Eczema. b. Impetigo. c. Herpes zoster. d. Diaper dermatitis.

ANS: B Impetigo is moist, thin-roofed vesicles with a thin erythematous base and is a contagious bacterial infection of the skin and most common in infants and children. Eczema is characterized by erythematous papules and vesicles with weeping, oozing, and crusts. Herpes zoster (i.e., chickenpox or varicella) is characterized by small, tight vesicles that are shiny with an erythematous base. Diaper dermatitis is characterized by red, moist maculopapular patches with poorly defined borders.

The structure that secretes a thin, milky alkaline fluid to enhance the viability of sperm is the: A) Cowper's gland. B) prostate gland. C) median sulcus. D) bulbourethral gland.

ANS: B In men, the prostate gland secretes a thin milky alkaline fluid that enhances sperm viability. The Cowper's glands (also known as bulbourethral glands) secrete a clear, viscid mucus. The median sulcus is a groove dividing the lobes of the prostate gland and does not secrete fluid.

A nurse is assessing a patient's risk of contracting a sexually transmitted infection (STI). An appropriate question to ask would be: A) "You know that it's important to use condoms for protection, right?" B) "Do you use a condom with each episode of sexual intercourse?" C) "Do you have a sexually transmitted infection?" D) "You are aware of the dangers of unprotected sex, aren't you?"

ANS: B In reviewing a patient's risk for sexually transmitted infections, the nurse should ask, in a nonconfrontational manner, whether condoms are used at each episode of sexual intercourse. Asking a person whether he or she has an infection does not address the risk.

An older adult woman is brought to the emergency department after being found lying on the kitchen floor for 2 days; she is extremely dehydrated. What would the nurse expect to see during the examination? a. Smooth mucous membranes and lips b. Dry mucous membranes and cracked lips c. Pale mucous membranes d. White patches on the mucous membranes

ANS: B With dehydration, mucous membranes appear dry and the lips look parched and cracked. The other responses are not found in dehydration.

The mother of a 10-year-old boy asks the nurse to discuss the recognition of puberty. The nurse should reply by saying: A) "Puberty usually begins about age fifteen." B) "The first sign of puberty is enlargement of the testes." C) "Penis size does not increase until about the age of sixteen." D) "The development of pubic hair precedes testicular or penis enlargement."

ANS: B Puberty begins sometime between ages 9 1/2 and 13 1/2 years. The first sign is enlargement of the testes. Next, pubic hair appears and then penis size increases.

When assessing the scrotum of a male patient, the nurse notices the presence of multiple firm, nontender, yellow 1-cm nodules. The nurse knows that these nodules are most likely: A) from urethritis. B) sebaceous cysts. C) subcutaneous plaques. D) from inflammation of the epididymis.

ANS: B Sebaceous cysts are commonly found on the scrotum. These are yellowish 1-cm nodules and are firm, nontender, and often multiple. The other options are not correct.

Which of these statements about the sphincters is correct? A) The internal sphincter is under voluntary control. B) The external sphincter is under voluntary control. C) Both sphincters remain slightly relaxed at all times. D) The internal sphincter surrounds the external sphincter.

ANS: B The external sphincter surrounds the internal sphincter but also has a small section overriding the tip of the internal sphincter at the opening. The external sphincter is under voluntary control. Except for the passing of feces and gas, the sphincters keep the anal canal tightly closed.

The nurse is palpating the prostate gland through the rectum and notices an abnormal finding if which of these is present? A) Palpable central groove B) Tenderness to palpation C) Heart shape D) Elastic and rubbery consistency

ANS: B The normal prostate gland should feel smooth, elastic, and rubbery; should be slightly movable; should be heart-shaped with a palpable central groove; and should not be tender to palpation.

The nurse keeps in mind that a thorough skin assessment is extremely important because the skin holds information about a person's: a. Support systems. b. Circulatory status. c. Socioeconomic status. d. Psychological wellness.

ANS: B The skin holds information about the body's circulation, nutritional status, and signs of systemic diseases, as well as topical data on the integumentary system itself.

A 52-year-old woman has a papule on her nose that has rounded, pearly borders and a central red ulcer. She said she first noticed it several months ago and that it has slowly grown larger. The nurse suspects which condition? a. Acne b. Basal cell carcinoma c. Melanoma d. Squamous cell carcinoma

ANS: B Basal cell carcinoma usually starts as a skin-colored papule that develops rounded, pearly borders with a central red ulcer. It is the most common form of skin cancer and grows slowly. This description does not fit acne lesions.

The nurse has just completed an inspection of a nulliparous woman's external genitalia. Which of these would be a description of a finding within normal limits? A) Redness of the labia majora B) Multiple nontender sebaceous cysts C) Discharge that is sticky and yellow-green D) Gaping and slightly shriveled labia majora

ANS: B There should be no lesions, except for occasional sebaceous cysts. These are yellowish 1-cm nodules that are firm, nontender, and often multiple. The labia majora are dark pink, moist, and symmetrical; redness indicates inflammation or lesions. Discharge that is sticky and yellow-green may indicate infection. In the nulliparous woman, the labia majora meet in the midline, are symmetric and plump.

During an internal examination of a woman's genitalia, the nurse will use which technique for proper insertion of the speculum? A) Instruct the woman to bear down, open the speculum blades, and apply in a swift, upward movement. B) Insert the blades of the speculum on a horizontal plane, turning them to a 30-degree angle while continuing to insert them. Ask the woman to bear down after the speculum is inserted. C) Instruct the woman to bear down, turn the width of the blades horizontally, and insert the speculum at a 45-degree angle downward toward the small of the woman's back. D) Lock the blades open by turning the thumbscrew. Once the blades are open, apply pressure to the introitus and insert the blades at a 45-degree angle downward to bring the cervix into view.

ANS: C The examiner should instruct the woman to bear down, turn the width of the blades horizontally, and insert the speculum at a 45-degree angle downward toward the small of the woman's back. See the text under "Speculum Examination" for more detail.

During an examination, the nurse notices that a male patient has a red, round, superficial ulcer with a yellowish serous discharge on his penis. On palpation, the nurse finds a nontender base that feels like a small button between the thumb and fingers. At this point the nurse suspects that this patient has: A) genital warts. B) a herpes infection. C) a syphilitic chancre. D) a carcinoma lesion.

ANS: C This lesion indicates syphilitic chancre, which begins within 2 to 4 weeks of infection. See Table 24-4 for descriptions of the other options.

During an assessment of the newborn, the nurse expects to see which finding when the anal area is slightly stroked? A) A jerking of the legs B) Flexion of the knees C) A quick contraction of the sphincter D) Relaxation of the external sphincter

ANS: C To assess sphincter tone, the nurse should check the anal reflex by gently stroking the anal area and noticing a quick contraction of the sphincter. The other responses are not correct.

The nurse is performing an assessment on an adult. The adult's vital signs are normal and capillary refill time is 5 seconds. What should the nurse do next? a. Ask the patient about a history of frostbite. b. Suspect that the patient has venous insufficiency. c. Consider this a delayed capillary refill time, and investigate further. d. Consider this a normal capillary refill time that requires no further assessment.

ANS: Consider this a delayed capillary refill time and investigate further. Normal capillary refill time is less than 1 to 2 seconds. The following conditions can skew the findings: a cool room, decreased body temperature, cigarette smoking, peripheral edema, and anemia.

3. The nurse is reviewing anatomy and physiology of the heart. Which statement best describes what is meant by atrial kick? A. The atria contract during systole and attempt to push against closed valves. B. The contraction of the atria at the beginning of diastole can be felt as a palpitation. C. This is the pressure exerted against the atria as the ventricles contract during systole. D. The atria contract toward the end of diastole and push the remaining blood into theventricles

ANS: D Toward the end of diastole, the atria contract and push the last amount of blood (about 25% of stroke volume) into the ventricles. This active filling phase is called pre systole, or atrial systole, or sometimes the "atrial kick."

The nurse notices that a patient has a solid, elevated, circumscribed lesion that is less than 1 cm in diameter. When documenting this finding, the nurse reports this as a: a. Bulla. b. Wheal. c. Nodule. d. Papule.

ANS: D A papule is something one can feel, is solid, elevated, circumscribed, less than 1 cm in diameter, and is due to superficial thickening in the epidermis. A bulla is larger than 1 cm, superficial, and thin walled. A wheal is superficial, raised, transient, erythematous, and irregular in shape attributable to edema. A nodule is solid, elevated, hard or soft, and larger than 1 cm.

Which of these statements is true regarding the penis? A) The urethral meatus is located on the ventral side of the penis. B) The prepuce is the fold of foreskin covering the shaft of the penis. C) The penis is composed of two cylindrical columns of erectile tissue. D) The corpus spongiosum expands into a cone of erectile tissue called the glans.

ANS: D At the distal end of the shaft, the corpus spongiosum expands into a cone of erectile tissue, the glans. The penis is composed of three cylindrical columns of erectile tissue. The prepuce is skin that covers the glans of the penis. The urethral meatus forms at the tip of the glans.

A 35-year-old woman is at the clinic for a gynecologic examination. During the examination, she asks the nurse, "How often do I need to have this Pap test done?" Which reply by the nurse is correct? A) "It depends. Do you smoke?" B) "This will need to be done annually until you are 65." C) "If you have 2 consecutive normal Pap tests, then you can wait 5 years between tests." D) "After age 30, if you have 3 consecutive normal Pap tests, then you may be screened every 2 to 3 years."

ANS: D Cervical cancer screening with the Pap test continues annually until age 30. After age 30, if the woman has 3 consecutive normal Pap tests, then women may be screened every 2 to 3 years.

A woman is leaving on a trip to Hawaii and has come in for a checkup. During the examination the nurse learns that she has diabetes and takes oral hypoglycemic agents. The patient needs to be concerned about which possible effect of her medications? a. Increased possibility of bruising b. Skin sensitivity as a result of exposure to salt water c. Lack of availability of glucose-monitoring supplies d. Importance of sunscreen and avoiding direct sunlight

ANS: D Drugs that may increase sunlight sensitivity and give a burn response include sulfonamides, thiazide diuretics, oral hypoglycemic agents, and tetracycline.

A patient comes to the clinic and states that he has noticed that his skin is redder than normal. The nurse understands that this condition is due to hyperemia and knows that it can be caused by: a. Decreased amounts of bilirubin in the blood b. Excess blood in the underlying blood vessels c. Decreased perfusion to the surrounding tissues d. Excess blood in the dilated superficial capillaries

ANS: D Erythema is an intense redness of the skin caused by excess blood (hyperemia) in the dilated superficial capillaries.

A 22-year-old woman is being seen at the clinic for problems with vulvar pain, dysuria, and fever. On physical examination, the nurse notices clusters of small, shallow vesicles with surrounding erythema on the labia. There is also inguinal lymphadenopathy present. The most likely cause of these lesions is: A) pediculosis pubis. B) contact dermatitis. C) human papillomavirus. D) herpes simplex virus type 2.

ANS: D Herpes simplex virus type 2 presents with clusters of small, shallow vesicles with surrounding erythema that erupt on the genital areas. There is also the presence of inguinal lymphadenopathy. The individual reports local pain, dysuria, and fever. See Table 26-2 for more information and descriptions of the other conditions.

31. A 21-year-old woman has been on a low-protein liquid diet for the past 2 months. She has had adequate calories and appears well nourished. In further assessing her, what would the nurse expect to find?

B) Decreased serum albumin

The nurse should use which test to check for large amounts of fluid around the patella? A) Ballottement B) Tinel sign C) Phalen's test D) McMurray's test

The nurse should use which test to check for large amounts of fluid around the patella? Ballottement of the patella is reliable when large amounts of fluid are present. 2. The Tinel sign and the Phalen test are used to check for carpal tunnel syndrome. 3. The McMurray test is used to test the knee for a torn meniscus.

A female patient tells the nurse that she has had six pregnancies, with four live births at term and two spontaneous abortions. Her four children are still living. How would the nurse record this information? a.P-6, B-4, (S)Ab-2 b.Grav 6, Term 4, (S)Ab-2, Living 4 c.Patient has had four living babies. d.Patient has been pregnant six times.

*b. Grav 6, Term 4, (S)Ab-2, Living 4* Obstetric history includes: - the # of pregnancies (gravidity) - number of deliveries in which the fetus reached term (term) - number of preterm pregnancies (preterm) - number of incomplete pregnancies (abortions) - number of children living (living). This is recorded: Grav _____ Term _____ Preterm _____ Ab _____ Living _____. For any incomplete pregnancies, the duration is recorded and whether the pregnancy resulted in a spontaneous (S) or an induced (I) abortion.

A 29-year-old woman tells the nurse that she has "excruciating pain" in her back. Which would be the nurse's appropriate response to the woman's statement? a."How does your family react to your pain?" b."The pain must be terrible. You probably pinched a nerve." c."I've had back pain myself, and it can be excruciating." d."How would you say the pain affects your ability to do your daily activities?"

*d."How would you say the pain affects your ability to do your daily activities?"* The symptom of pain is difficult to quantify because of individual interpretation. With pain, adjectives should be avoided and the patient should be asked how the pain affects his or her daily activities. The other responses are not appropriate.

29. Just before going home, a new mother asks the nurse about the infant's umbilical cord. Which of these statements is correct? a. "It should fall off in 10 to 14 days." b. "It will soften before it falls off." c. "It contains two veins and one artery." d. "Skin will cover the area within 1 week."

A) "It should fall off by 10 to 14 days." At birth, the umbilical cord is white and contains two umbilical arteries and one vein inside the Wharton jelly. The umbilical stump dries within a week, hardens, and falls off in 10 to 14 days. Skin will cover the area in 3 to 4 weeks.

1. The nurse recognizes that which of these persons is at greatest risk for undernutrition?

A) 5-month-old infant

13. When evaluating the temperature of older adults, the nurse should remember which aspect about an older adults body temperature? a. The body temperature of the older adult is lower than that of a younger adult. b. An older adults body temperature is approximately the same as that of a young child. c. Body temperature depends on the type of thermometer used. d. In the older adult, the body temperature varies widely because of less effective heat control mechanisms.

ANS: A In older adults, the body temperature is usually lower than in other age groups, with a mean temperature of 36.2 C.

During an examination, the nurse would expect the cervical os of a woman who has never had children to appear: A) stellate. B) small and round. C) as a horizontal irregular slit. D) everted.

ANS: B The cervical os in a nulliparous woman is small and round. In the parous woman, it is a horizontal, irregular slit that also may show healed lacerations on the sides. See Figure 26-13.

12. When measuring a patients body temperature, the nurse keeps in mind that body temperature is influenced by: a. Constipation. b. Patients emotional state. c. Diurnal cycle. d. Nocturnal cycle.

ANS: C Normal temperature is influenced by the diurnal cycle, exercise, and age. The other responses do not influence body temperature.

A patient comes to the clinic and tells the nurse that he has been confined to his recliner chair for approximately 3 days with his feet down and he asks the nurse to evaluate his feet. During the assessment, the nurse might expect to find: a. Pallor b. Coolness c. Distended veins d. Prolonged capillary filling time

ANS: C Keeping the feet in a dependent position causes venous pooling, resulting in redness, warmth, and distended veins. Prolonged elevation would cause pallor and coolness. Immobilization or prolonged inactivity would cause prolonged capillary filling time (see Table 12-1).

A 70-year-old woman who loves to garden has small, flat, brown macules over her arms and hands. She asks, "What causes these liver spots?" The nurse tells her, "They are: a. "Signs of decreased hematocrit related to anemia." b. "Due to the destruction of melanin in your skin from exposure to the sun." c. "Clusters of melanocytes that appear after extensive sun exposure." d. "Areas of hyperpigmentation related to decreased perfusion and vasoconstriction."

ANS: C Liver spots, or senile lentigines, are clusters of melanocytes that appear on the forearms and dorsa of the hands after extensive sun exposure. The other responses are not correct.

A patient has been admitted for severe psoriasis. The nurse expects to see what finding in the patients fingernails? a. Splinter hemorrhages b. Paronychia c. Pitting d. Beau lines

ANS: C Sharply defined pitting and crumbling of the nails, each with distal detachment characterize pitting nails and are associated with psoriasis. DIF: Cognitive Level: Applying (Application) MSC: Client Needs: Physiologic Integrity: Physiologic Adaptation

During an examination, the nurse asks the patient to perform the Valsalva maneuver and notices that the patient has a moist, red, doughnut-shaped protrusion from the anus. The nurse knows that this would be consistent with: A) a rectal polyp. B) hemorrhoids. C) a rectal fissure. D) rectal prolapse.

ANS: D In rectal prolapse, the rectal mucous membrane protrudes through the anus, appearing as a moist red doughnut with radiating lines. It occurs following a Valsalva maneuver, such as straining at stool, or with exercise. See Table 25-1. For a description of rectal polyps, see Table 25-2. See Table 25-1 for descriptions of rectal fissure and hemorrhoids.

During the aging process, the hair can look gray or white and begin to feel thin and fine. The nurse knows that this occurs because of a decrease in the number of functioning: a. Metrocytes. b. Fungacytes. c. Phagocytes. d. Melanocytes.

ANS: D In the aging hair matrix, the number of functioning melanocytes decreases; as a result, the hair looks gray or white and feels thin and fine. The other options are not correct.

A patient has had a "terrible itch" for several months that he has been continuously scratching. On examination, the nurse might expect to find: a. A keloid. b. A fissure. c. Keratosis. d. Lichenification.

ANS: D Lichenification results from prolonged, intense scratching that eventually thickens the skin and produces tightly packed sets of papules. A keloid is a hypertrophic scar. A fissure is a linear crack with abrupt edges, which extends into the dermis; it can be dry or moist. Keratoses are lesions that are raised, thickened areas of pigmentation that appear crusted, scaly, and warty.

The nurse educator is preparing an education module for the nursing staff on the dermis layer of skin. Which of these statements would be included in the module? The dermis: a. Contains mostly fat cells. b. Consists mostly of keratin. c. Is replaced every 4 weeks. d. Contains sensory receptors.

ANS: D The dermis consists mostly of collagen, has resilient elastic tissue that allows the skin to stretch, and contains nerves, sensory receptors, blood vessels, and lymphatic vessels. It is not replaced every 4 weeks.

When assessing a patient's pain, the nurse knows that an example of visceral pain would be:

ANS: cholecystitis. Visceral pain originates from the larger interior organs, such as the gallbladder, liver, or kidneys.

Which of these statements is true regarding the vertebra prominens? The vertebra prominens is: a. The spinous process of C7. b. Usually nonpalpable in most individuals. c. Opposite the interior border of the scapula. d. Located next to the manubrium of the sternum.

ANS: the spinous process of C7. The spinous process of C7 is the vertebra prominens. It is the most prominent bony spur protruding at the base of the neck. Counting ribs and intercostal spaces on the posterior thorax is difficult because of the muscles and soft tissue. The vertebra prominens is easier to identify and is used as a starting point in counting thoracic processes and identifying landmarks on the posterior chest.

During an interview the patient states, "I can feel this bump on the top of both of my shoulders—it doesn't hurt but I am curious about what it might be." The nurse should tell the patient, "That is: A) your subacromial bursa." B) your acromion process." C) your glenohumeral joint." D) the greater tubercle of your humerus."

ANS: your acromion process." The bump of the scapula's acromion process is felt at the very top of the shoulder. The other options are not correct.

27. The nurse needs to perform anthropometric measures of an 80-year-old man who is confined to a wheelchair. Which of the following is true in this situation?

B) Height measurements may not be accurate because of changes in bone.

19. How should the nurse perform a triceps skinfold assessment?

C) After applying the calipers, wait 3 seconds before taking a reading. Repeat the procedure three times.

During the assessment of deep tendon reflexes, the nurse finds that a patient's responses are bilaterally normal. What number is used to indicate normal deep tendon reflexes when the documenting this finding? ____+

Responses to assessment of deep tendon reflexes are graded on a 4-point scale. A rating of 2+ indicates normal or average response. A rating of 0 indicates no response, and a rating of 4+ indicates very brisk, hyperactive response with clonus, which is indicative of disease.

In recording the childhood illnesses of a patient who denies having had any, which note by the nurse would be most accurate? a.Patient denies usual childhood illnesses. b.Patient states he was a "very healthy" child. c.Patient states his sister had measles, but he didn't. d.Patient denies measles, mumps, rubella, chickenpox, pertussis, and strep throat.

*d. Patient denies measles, mumps, rubella, chickenpox, pertussis, and strep throat.* Childhood illnesses include measles, mumps, rubella, chickenpox, pertussis, and strep throat. Avoid recording "usual childhood illnesses" because an illness common in the person's childhood may be unusual today (e.g., measles).

The nurse is preparing to conduct a health history. Which of these statements best describes the purpose of a health history? a.To provide an opportunity for interaction between the patient and the nurse b.To provide a form for obtaining the patient's biographic information c.To document the normal and abnormal findings of a physical assessment d.To provide a database of subjective information about the patient's past and current health

*d. To provide a database of subjective information about the patient's past and current health* The purpose of the health history is to collect subjective data—what the person says about him or herself. The other options are not correct.

The nurse is reviewing a patient's medical record and notes that he is in a coma. Using the Glasgow Coma Scale, which number indicates that the patient is in a coma? a.6 b.12 c.15 d.24

A A fully alert, normal person has a score of 15, whereas a score of 7 or less reflects coma on the Glasgow Coma Scale

Expert nurses learn to attend to a pattern of assessment data and act without consciously labeling it. These responses are referred to as: a. Intuition. b. The nursing process. c. Clinical knowledge. d. Diagnostic reasoning.

A Intuition is characterized by pattern recognition expert nurses learn to attend to a pattern of assessment data and act without consciously labeling it. The other options are not correct.

34. The nurse is reviewing statistics for lactose intolerance. In the United States, the incidence of lactose intolerance is higher in adults of which ethnic group? a. Blacks b. Hispanics c. Whites d. Asians

A) Blacks A recent study found estimates of lactose-intolerance prevalence as follows: 19.5% for Blacks, 10% for Hispanics, and 7.72% for Whites.

1. The nurse is percussing the seventh right intercostal space at the midclavicular line over the liver. Which sound should the nurse expect to hear? a. Dullness b. Tympany c. Resonance d. Hyperresonance

A) Dullness The liver is located in the right upper quadrant and would elicit a dull percussion note.

30. Which of these percussion findings would the nurse expect to find in a patient with a large amount of ascites? a. Dullness across the abdomen b. Flatness in the right upper quadrant c. Hyperresonance in the left upper quadrant d. Tympany in the right and left lower quadrants

A) Dullness across the abdomen A large amount of ascitic fluid produces a dull sound to percussion.

When assessing the intensity of a patient's pain, which question by the nurse is appropriate?

ANS: "How much pain do you have now?" Asking the patient "how much pain do you have?" is an assessment of the intensity of a patient's pain; various intensity scales can be used. Asking what makes one's pain better or worse assesses alleviating or aggravating factors. Asking if pain limits one's activities assesses the degree of impairment and quality of life. Asking "what does your pain feel like" assesses the quality of pain.

Which statement indicates that the nurse understands the pain experienced by an elderly person?

ANS: "Pain indicates pathology or injury and is not a normal process of aging." Pain indicates pathology or injury and should never be considered something that an elderly person should expect or tolerate. Pain is not a normal process of aging, and there is no evidence that pain perception is reduced with aging.

When assessing the quality of a patient's pain, the nurse should ask which question?

ANS: "What does your pain feel like?" To assess the quality of a person's pain, have the patient describe the pain in his or her own words.

The nurse is interviewing a recent male immigrant from Mexico. During the course of the interview, he leans forward and then finally moves his chair close enough that his knees are nearly touching. The nurse begins to feel uncomfortable with his proximity. Which statement most closely reflects what the nurse should do next? 1.Try to relax—these behaviors are culturally appropriate for this person. 2.Discreetly move one's chair back until the distance is more comfortable and then continue with the interview. 3.These behaviors are indicative of sexual aggression, and the nurse should confront this person about them. 4.The nurse should laugh but tell him that he or she is uncomfortable with his proximity and ask him to move away.

ANS: 1 Both the patient's and one's own sense of spatial distance are significant throughout the interview and physical examination, with culturally appropriate distance zones varying widely. For example, you may find yourself backing away from people of Mexican-American, Indian, or Middle Eastern origins who invade your personal space with regularity in an attempt to bring you closer into the space that is comfort- able to them.

A 75-year-old woman is at the office for a preoperative interview. The nurse is aware that the interview may take longer than interviews with younger persons. What is the reason for this? 1.An aged person has a longer story to tell. 2.An aged person is usually lonely and likes to have someone to talk to. 3.Aged persons lose much of their mental abilities and require greater time to complete an interview. 4.As a person ages, they are unable to hear and thus interviewers usually need to repeat much of what is said.

ANS: 1 The interview usually takes longer with older adults because they have a longer story to tell.

The muscles in the neck that are innervated by CN XI are the: 1. sternomastoid and trapezius. 2. spinal accessory and omohyoid. 3. trapezius and sternomandibular. 4. sternomandibular and spinal accessory.

ANS: 1 The major neck muscles are the sternomastoid and the trapezius. They are innervated by CN XI, the spinal accessory.

A patient's laboratory data reveal an elevated thyroxine level. The nurse would proceed with an examination of the: 1. thyroid gland. 2. parotid gland. 3. adrenal gland. 4. thyroxine gland.

ANS: 1 The thyroid gland is a highly vascular endocrine gland that secretes thyroxine (T4) and tri-iodothyronine (T3).

During a prenatal check, a patient begins to cry as the nurse asks her about previous pregnancies. She states that she is remembering her last pregnancy, which ended in miscarriage. The nurse's best response to her crying would be: 1."I'm so sorry for making you cry!" 2."I can see that you are sad remembering this. It is all right to cry." 3."Why don't I step out for a few minutes until you're feeling better?" 4."I can see that you feel sad about this; why don't we talk about something else?"

ANS: 2 A beginning examiner usually feels horrified when the patient starts crying. When you say something that "makes the person cry," do not think you have hurt the person. You have just hit on a topic that is important. Do not go on to a new topic. Just let the person cry and express his or her feelings fully. You can offer a tissue and wait until the crying subsides to talk.

A mother brings her 28-month-old daughter into the clinic for a well-child visit. At the beginning of the visit, the nurse focuses attention away from the toddler, but as the interview progresses, the toddler begins to "warm up" and is smiling shyly at the nurse. The nurse will be most successful in interacting with the toddler if which is done next? 1.Tickle the toddler and get her to laugh. 2.Stoop down to her level and ask her about the toy she is holding. 3.Continue to ignore her until it is time for the physical examination. 4.Ask the mother. to leave during the examination of the toddler because toddlers often fuss less if their parents aren't in view.

ANS: 2 Although most of the communication is with the parent, do not ignore the child completely. Making contact to will assist in easing into the physical examination later. Begin by asking about the toys the child is playing with or about a special doll or teddy bear brought from home: "Does your doll have a name?" or "What can your truck do?" Stoop down to meet the child at his or her eye level.

A 17-year-old single mother is describing how difficult it is to raise a 2-year-old by herself. During the course of the interview she states, "I can't believe my boyfriend left me to do this by myself! What a terrible thing to do to me!" Which of the following responses by the nurse uses empathy? 1."You feel alone?" 2."You can't believe he left you alone." 3."It must be so hard to face this all alone." 4."I would be angry, too; raising a child alone is no picnic."

ANS: 3 An empathetic response recognizes the feeling and puts it into words. It names the feeling and allows the expression of it. It strengthens rapport. Other empathetic responses are, "This must be very hard for you," "I understand," or just placing your hand on the person's arm.

A patient presents with excruciating headache pain on one side of his head, especially around his eye, forehead, and cheek that lasts about 1/2 to 2 hours, occurring once or twice each day. The nurse suspects: 1. hypertension. 2. cluster headaches. 3. tension headaches. 4. migraine headaches.

ANS: 2 Cluster headaches produce pain around the eye, temple, forehead, and cheek and are unilateral and always on the same side of the head. They are excruciating and occur once or twice per day and last 1/2 to 2 hours each.

The nurse is conducting an interview with a woman who has recently learned that she is pregnant and has come to the clinic today to begin prenatal care. The woman states that she and her husband are excited about the pregnancy but have a few questions. She looks nervously at her hands during the interview and sighs loudly. Considering the concept of communication, the nurse knows that which statement is most accurate? The woman: 1. is excited about her pregnancy but nervous about labor. 2. is exhibiting verbal and nonverbal behavior that does not match. 3. is excited about her pregnancy but her husband is not and this is upsetting to her. 4. is not excited about her pregnancy but believes the nurse will respond negatively to her if she states this.

ANS: 2 Communication is all behavior, conscious and unconscious, verbal and nonverbal. All behavior has meaning.

A patient is unable to differentiate between sharp and dull stimulation to both sides of her face. The nurse suspects: 1. Bell's palsy. 2. damage to the trigeminal nerve. 3. frostbite with resultant paresthesia to the cheeks. 4. scleroderma with a pronounced proliferation of connective tissue in the face and cheeks.

ANS: 2 Facial sensations of pain or touch are mediated by CN V, the trigeminal nerve.

The nurse has used interpretation regarding a patient's statement or actions. After using this technique, it would be best for the nurse to: 1.apologize because this can be demeaning for the patient. 2.allow the patient time to confirm or correct the inference. 3.continue with the interview as though nothing had happened. 4.immediately restate the nurse's conclusion on the basis of the patient's nonverbal response.

ANS: 2 Interpretation is not based on direct observation as is confrontation, but it is based on one's inference or conclusion. You do run a risk of making the wrong inference. If this is the case, the person will correct it. But even if the inference is corrected, interpretation helps to prompt further discussion of the topic.

During an examination, the nurse finds that a patient's left temporal artery is more tortuous and feels hardened and tender compared with the right temporal artery. The nurse suspects which condition? 1. Crepitation 2. Mastoiditis 3. Temporal arteritis 4. Bell's palsy

ANS: 3 The artery looks more tortuous and feels hardened and tender with temporal arteritis.

34. During a cardiovascular assessment, the nurse knows that a "thrill" is: A. A vibration that is palpable. B. Palpated in the right epigastric area. C. Associated with ventricular hypertrophy. D. A murmur auscultated at the third intercostal space

ANS: A A thrill is a palpable vibration. It signifies turbulent blood flow and accompanies loud murmurs. The absence of a thrill does not rule out the presence of a murmur.

The nurse has just completed a lymph assessment on a 60-year-old healthy female patient. The nurse knows that most lymph nodes in healthy adults are normally: 1. shotty. 2. not palpable. 3. large, firm, and fixed to the tissue. 4. rubbery, discrete, and mobile.

ANS: 2 Most lymph nodes are not palpable in adults. The palpability of lymph nodes decreases with age.

A pregnant woman states, "I just know labor will be so painful that I won't be able to stand it. I know it sounds awful, but I really dread going into labor." The nurse responds by stating, "Oh, don't worry about labor so much. I have been through it and although it is painful there are many good medications to decrease the pain." Which statement is true regarding this response? 1.It was a therapeutic response. By sharing something personal, the nurse gives hope to this woman. 2.It was a nontherapeutic response. By providing false reassurance, the nurse actually cut off further discussion of the woman's fears. 3.It was a therapeutic response. By providing information about the medications available, the nurse is giving information to the woman. 4.It was a nontherapeutic response. The nurse is essentially giving the message to the woman that labor cannot be tolerated without medication.

ANS: 2 Providing false assurance or reassurance, this "courage builder," relieves one's anxiety and gives you the false sense of having provided comfort. But for the woman, it actually closes off communication. It trivializes her anxiety and effectively denies any further talk of it.

A 16-year-old boy has just been admitted to the unit for overnight observation after being in an automobile accident. What is the nurse's best approach to communicating with him? 1. Use periods of silence to communicate respect for him. 2. Be totally honest with him, even if the information is unpleasant. 3. Tell him that everything that is discussed will be kept totally confidential. 4. Use slang language when possible to help him open up.

ANS: 2 Successful communication (with adolescents) is possible and rewarding. The guidelines are simple. The first consideration is one's attitude, which must be one of respect. Second, communication must be totally honest. Adolescents' intuition is highly tuned and can detect phoniness or withholding of information. Always give them the truth.

A patient says that she has recently noticed a lump in the front of her neck below her "Adam's apple" that seems to be getting bigger. During the assessment, the finding that reassures the nurse that this may not be a cancerous thyroid nodule is that the lump (nodule): 1. is tender. 2. is mobile and not hard. 3. disappears when the patient smiles. 4. is hard and fixed to the surrounding structures.

ANS: 2 Suspect any painless, rapidly growing nodule, especially the appearance of a single nodule in a young person. Cancerous nodules tend to be hard and are fixed to surrounding structures.

A patient has come in for an examination and states, "I have this spot in front of my ear lobe here on my cheek that seems to be getting bigger and is real tender. What do you think it is?" The nurse notes swelling below the angle of the jaw and suspects that it could be an inflammation of his: 1. thyroid gland. 2. parotid gland. 3. occipital lymph node. 4. submental lymph node.

ANS: 2 Swelling with the parotid gland occurs below the angle of the jaw and is most visible when the head is extended. Painful inflammation occurs with mumps, and swelling also occurs with abscesses or tumors. Swelling occurs anterior to the lower ear lobe.

The physician reports that a patient has a tracheal shift. The nurse is aware that this means that the patient's trachea is: 1. pulled to the affected side with systole. 2. pushed to the unaffected side with a tumor. 3. pulled to the unaffected side with plural adhesions. 4. pushed to the affected side with thyroid enlargement.

ANS: 2 The trachea is pushed to the unaffected side with an aortic aneurysm, a tumor, unilateral thyroid lobe enlargement, and pneumothorax.

Which of the following is appropriate for the nurse to say near the end of the interview? 1."Did we forget something?" 2."Is there anything else you would like to mention?" 3."I need to go on to the next patient. I'll be back." 4."While I'm here, let's talk about your upcoming surgery."

ANS: 2 This question gives the person the final opportunity for self-expression. No new topic should be introduced.

A woman is discussing the problems she is having with her 2-year-old son. "He won't go to sleep at night and during the day he has several fits. I get so upset when that happens." The nurse's best verbal response would be: 1."Go on, I'm listening." 2."Fits? Tell me what you mean by this." 3."Yes, it can be upsetting when a child has a fit." 4."Don't be upset when he has a fit; every 2-year-old has fits."

ANS: 2 Use clarification when the person's word choice is ambiguous or confusing (e.g., "Tell me what you mean by 'tired blood.'") Clarification is also used to summarize the person's words, simplify the words to make them clearer, and then ask if you are on the right track.

During a clinic visit, a patient states, "The doctor just told me he thought I ought to stop smoking. He doesn't understand how hard I've tried. I just don't know the best way to do it. What should I do?" The nurse's most appropriate response in this case would be: 1."I'd quit. The doctor really knows what he is talking about." 2."Would you like some information about the different ways a person can quit smoking?" 3."Stopping your dependence on cigarettes can be very difficult. I understand how you feel." 4."Why are you confused? Didn't the doctor give you the information about the smoking cessation program we offer?"

ANS: 2 Use clarification when the person's word choice is ambiguous or confusing. Clarifi- cation is also used to summarize the person's words, simplify the words to make them clearer, and then ask if you are on the right track.

During an admission assessment, the nurse notices that a male patient has an enlarged and rather thick skull. The nurse suspects acromegaly and would further assess for: 1. exophthalmos. 2. bowed long bones. 3. coarse facial features. 4. an acorn-shaped cranium.

ANS: 3 Acromegaly is excessive secretion of growth hormone that creates an enlarged skull and thickened cranial bones. Patients will have elongated heads, massive faces, prominent noses and lower jaws, heavy eyebrow ridges, and coarse facial features.

During an examination of a female patient, the nurse notes lymphadenopathy and suspects an acute infection. Acutely infected lymph nodes would be: 1. clumped. 2. unilateral. 3. firm but freely movable. 4. hard and nontender.

ANS: 3 Acutely infected lymph nodes are bilateral, enlarged, warm, tender, and firm but freely movable.

A visitor from Poland who does not speak English seems to be somewhat apprehensive about the nurse examining his neck. He would probably be most comfortable with the nurse examining his thyroid: 1. from behind with the nurse's hands placed firmly around his neck. 2. from the side with the nurse's eyes averted toward the ceiling and thumbs on his neck. 3. from the front with the nurse's thumbs placed on either side of his trachea and his head tilted forward. 4. from the front with the nurse's thumbs placed on either side of his trachea and his head tilted backward.

ANS: 3 Examining this patient's thyroid from the back may be unsettling for him. It would be best to examine his thyroid using the anterior approach, asking him to tip his head forward and to the right and then the left.

The nurse notices that a patient's palpebral fissures are not symmetrical. On examination, the nurse may find that there has been damage to:1. 1.CN III. 2.CN V. 3.CN VII. 4.CN VIII.

ANS: 3 Facial muscles are mediated by CN VII; asymmetry of palpebral fissures may be due to CN VII damage.

A woman comes to the clinic and states, "My eyes have gotten so puffy, and my eyebrows and hair have become coarse and dry." The nurse suspects: 1. cachexia. 2. cretinism. 3. myxedema. 4. scleroderma.

ANS: 3 Myxedema (hypothyroidism) is a deficiency of thyroid hormone that, when severe, causes a nonpitting edema or myxedema. The patient will have a puffy edematous face especially around eyes (periorbital edema), coarse facial features, dry skin, and dry, coarse hair and eyebrows

A patient, an 85-year-old woman, is complaining about the fact that the bones in her face have become more noticeable. What explanation should the nurse give to her? 1. Diets low in protein and high in carbohydrates may cause enhanced facial bones. 2. It is probably because she doesn't use a dermatologically approved moisturizer. 3. It is probably due to a combination of factors such as decreased elasticity, subcutaneous fat, and moisture in her skin. 4. Facial skin becomes more elastic with age. This increased elasticity causes the skin to be more taught, drawing attention to the facial bones.

ANS: 3 The facial bones and orbits appear more prominent in the aging adult, and the facial skin sags owing to decreased elasticity, decreased subcutaneous fat, and decreased moisture in the skin.

Receiving is a part of the communication process. Which receiver is most likely to misinterpret a message sent by a healthcare professional? 1.A well-adjusted adolescent in for a sports physical 2.A recovering alcoholic in for a basic physical examination 3.A man whose wife has just been diagnosed with lung cancer 4.A hearing-impaired man who uses sign language to communicate and has an interpreter with him

ANS: 3 The receiver attaches meaning determined by his or her past experiences, culture, self-concept, and current physical and emotional states.

An American Indian woman has come to the clinic for diabetic follow-up teaching. During the interview, the nurse notices that she never makes eye contact and speaks mostly to the floor. Which statement is true regarding this situation? 1. She is nervous and embarrassed. 2. She has something to hide and is ashamed. 3. She is showing inconsistent verbal and nonverbal behaviors. 4. She is showing that she is listening carefully to what the nurse is saying.

ANS: 4 Eye contact is perhaps among the most culturally variable nonverbal behaviors. Asian, American Indian, Indochinese, Arab, and Appalachian people may consider direct eye contact impolite or aggressive, and they may avert their own eyes during the interview. American Indians often stare at the floor during the interview, a culturally appropriate behavior indicating that the listener is paying close attention to the speaker.

The nurse is aware that the four areas in the body where lymph nodes are accessible are the: 1. head, breasts, groin, and abdomen. 2. arms, breasts, inguinal area, and legs. 3. head and neck, arms, breasts, and axillae. 4. head and neck, arms, inguinal area, and axillae.

ANS: 4 Nodes are located throughout the body, but are accessible to examination only in four areas: head and neck, arms, axillae, and inguinal region.

23. During an examination of a 3-year-old child, the nurse will need to take her blood pressure. What might the nurse do to try to gain the child's full cooperation? 1.Tell the child that the blood pressure cuff is going to give her arm a big hug. 2.Tell the child that the blood pressure cuff is asleep and cannot wake up. 3.Give the blood pressure cuff a name and refer to it by this name during the assessment. 4.Tell the child that by using the blood pressure cuff, we can see how strong her muscles are.

ANS: 4 Use short, simple sentences with a concrete explanation. Take time to give a short, simple explanation for any unfamiliar equipment that will be used on the child. Preschoolers are animistic; they imagine inanimate objects can come alive and have human characteristics. Thus a blood pressure cuff can wake up and bite or pinch.

The following statement could be found at which phase of the interview? "Mr. S., I would like to ask you some questions about your health and your usual daily activities so that we can better plan your stay here." 1. During the summary 2. Closing the interview 3. During the body of the interview 4. Opening/introducing the interview

ANS: 4 When gathering a complete history, give the reason for the interview during the opening or introduction of the interview.

Of the 33 vertebrae in the spinal column, there are: A) 5 lumbar. B) 5 thoracic. C) 7 sacral. D) 12 cervical.

ANS: 5 lumbar. There are 7 cervical, 12 thoracic, 5 lumbar, 5 sacral, and 3 to 4 coccygeal vertebrae.

39. The nurse is preparing for a class on risk factors for hypertension, and reviews recent statistics. Which racial group has the highest prevalence of hypertension in the world? A. African-Americans B. Whites C. American Indians D. Hispanic

ANS: A According to the American Heart Association, the prevalence of hypertension is higher among African-Americans than in other racial groups.

4. During an examination of a child, the nurse considers that physical growth is the best index of a childs: a. General health. b. Genetic makeup. c. Nutritional status. d. Activity and exercise patterns.

ANS: A Physical growth is the best index of a childs general health; recording the childs height and weight helps determine normal growth patterns.

9. The nurse knows that one advantage of the tympanic membrane thermometer (TMT) is that: a. Rapid measurement is useful for uncooperative younger children. b. Using the TMT is the most accurate method for measuring body temperature in newborn infants. c. Measuring temperature using the TMT is inexpensive. d. Studies strongly support the use of the TMT in children under the age 6 years.

ANS: A The TMT is useful for young children who may not cooperate for oral temperatures and fear rectal temperatures. However, the use a TMT with newborn infants and young children is conflicting.

1. The sac that surrounds and protects the heart is called the: A. Pericardium. B. Myocardium. C. Endocardium. D. Pleural space

ANS: A The pericardium is a tough fibrous double-walled sac that surrounds and protects the heart. It has two layers that contain a few milliliters of serous pericardial fluid.

20. When assessing a patients pulse, the nurse should also notice which of these characteristics? a. Force b. Pallor c. Capillary refill time d. Timing in the cardiac cycle

ANS: A The pulse is assessed for rate, rhythm, and force.

37. The nurse is preparing to measure the vital signs of a 6-month-old infant. Which action by the nurse is correct? a. Respirations are measured; then pulse and temperature. b. Vital signs should be measured more frequently than in an adult. c. Procedures are explained to the parent, and the infant is encouraged to handle the equipment. d. The nurse should first perform the physical examination to allow the infant to become more familiar with her and then measure the infants vital signs.

ANS: A With an infant, the order of vital sign measurements is reversed to respiration, pulse, and temperature. Taking the temperature first, especially if it is rectal, may cause the infant to cry, which will increase the respiratory and pulse rate, thus masking the normal resting values. The vital signs are measured with the same purpose and frequency as would be measured in an adult.

A patient tells the nurse that he has noticed that one of his moles has started to burn and bleed. When assessing his skin, the nurse pays special attention to the danger signs for pigmented lesions and is concerned with which additional finding? a. Color variation b. Border regularity c. Symmetry of lesions d. Diameter of less than 6 mm

ANS: A Abnormal characteristics of pigmented lesions are summarized in the mnemonic ABCD: asymmetry of pigmented lesion, border irregularity, color variation, and diameter greater than 6 mm.

During a health history of a patient who complains of chronic constipation, the patient asks the nurse about high-fiber foods. The nurse relates that an example of a high-fiber food would be: A) broccoli. B) hamburger. C) iceberg lettuce. D) yogurt.

ANS: A High-fiber foods are either soluble type (i.e., beans, prunes, barley, broccoli) and insoluble type (i.e., cereals, wheat germ). The other examples are not considered high-fiber foods.

The mother of a 5-year-old girl tells the nurse that she has noticed her daughter "scratching at her bottom a lot the last few days." During the assessment, the nurse finds redness and raised skin in the anal area. This most likely indicates: A) pinworms. B) chickenpox. C) constipation. D) bacterial infection.

ANS: A In children, pinworms are a common cause of intense itching and irritated anal skin. The other options are not correct.

An older man is concerned about his sexual performance. The nurse knows that in the absence of disease, a withdrawal from sexual activity later in life may be due to: A) side effects of medications. B) decreased libido with aging. C) decreased sperm production. D) decreased pleasure from sexual intercourse.

ANS: A In the absence of disease, a withdrawal from sexual activity may be due to side effects of medications such as antihypertensives, antidepressants, or sedatives. The other options are not correct.

A newborn baby boy is about to have a circumcision. The nurse knows that indications for circumcision include: A) cultural and religious beliefs. B) prevention of testicular cancer. C) improving the sperm count later in life. D) preventing dysuria

ANS: A Indications for circumcision include cultural and religious beliefs, prevention of phimosis and inflammation of the glans penis and foreskin, decreasing the incidence of cancer of the penis, and decreasing the incidence of urinary tract infections in infancy.

A 70-year-old man is visiting the clinic for difficulty in passing urine. In the history he indicates he has to urinate frequently, especially at night. He has burning when he urinates and has noticed pain in his back. Given this history, what might the nurse expect to find during the physical assessment? A) Asymmetric, hard, fixed prostate gland B) Occult blood and perianal pain to palpation C) Symmetrically enlarged, soft prostate gland D) A soft nodule protruding from rectal mucosa

ANS: A Subjective symptoms of carcinoma of the prostate include frequency, nocturia, hematuria, weak stream, hesitancy, pain or burning on urination, and continuous pain in lower back, pelvis, and thighs. Objective symptoms of carcinoma of the prostate include a malignant neoplasm often starts as a single hard nodule on the posterior surface, producing asymmetry and a change in consistency. As it invades normal tissue, multiple hard nodules appear, or the entire gland feels stone hard and fixed.

The nurse is examining a patient who tells the nurse, I sure sweat a lot, especially on my face and feet but it doesnt have an odor. The nurse knows that this condition could be related to: a. Eccrine glands. b. Apocrine glands. c. Disorder of the stratum corneum. d. Disorder of the stratum germinativum.

ANS: A The eccrine glands are coiled tubules that directly open onto the skin surface and produce a dilute saline solution called sweat. Apocrine glands are primarily located in the axillae, anogenital area, nipples, and naval area and mix with bacterial flora to produce the characteristic musky body odor. The patients statement is not related to disorders of the stratum corneum or the stratum germinativum. DIF: Cognitive Level: Applying (Application) MSC: Client Needs: Physiologic Integrity: Physiologic Adaptation

The nurse is caring for a newborn infant. Thirty hours after birth, the infant passes a dark green meconium stool. The nurse recognizes that this is important because: A) this stool would indicate anal patency. B) the dark green color could indicate occult blood in the stool. C) meconium stool can be reflective of distress in the newborn. D) the newborn should have passed the first stool within 12 hours after birth.

ANS: A The first stool passed by the newborn is dark green meconium and occurs within 24 to 48 hours of birth, indicating anal patency. The other responses are not correct

The nurse is reviewing the changes that occur with menopause. Which of these are changes associated with menopause? A) Uterine and ovarian atrophy along with thinning vaginal epithelium B) Ovarian atrophy, increased vaginal secretions, and increasing clitoral size C) Cervical hypertrophy, ovarian atrophy, and increased acidity of vaginal secretions D) Vaginal mucosa fragility, increased acidity of vaginal secretions, and uterine hypertrophy

ANS: A The uterus shrinks because of its decreased myometrium. The ovaries atrophy to 1 to 2 cm and are not palpable after menopause. The sacral ligaments relax, and the pelvic musculature weakens, so the uterus droops. The cervix shrinks and looks paler with a thick glistening epithelium. The vaginal epithelium atrophies, becoming thinner, drier, and itchy. The vaginal pH becomes more alkaline, and secretions are decreased. This results in a fragile mucosal surface that is at risk for bleeding and vaginitis.

While performing an assessment of a 65-year-old man with a history of hypertension and coronary artery disease, the nurse notices the presence of bilateral pitting edema in the lower legs. The skin is puffy and tight but normal in color. No increased redness or tenderness is observed over his lower legs, and the peripheral pulses are equal and strong. In this situation, the nurse suspects that the likely cause of the edema is which condition? a. Heart failure b. Venous thrombosis c. Local inflammation d. Blockage of lymphatic drainage

ANS: A Bilateral edema or edema that is generalized over the entire body is caused by a central problem such as heart failure or kidney failure. Unilateral edema usually has a local or peripheral cause. DIF: Cognitive Level: Analyzing (Analysis) MSC: Client Needs: Physiologic Integrity: Physiologic Adaptation

. During an examination, the nurse finds that a patient has excessive dryness of the skin. The best term to describe this condition is: a. Xerosis. b. Pruritus. c. Alopecia. d. Seborrhea.

ANS: A Xerosis is the term used to describe skin that is excessively dry. Pruritus refers to itching, alopecia refers to hair loss, and seborrhea refers to oily skin. DIF: Cognitive Level: Remembering (Knowledge) MSC: Client Needs: Health Promotion and Maintenance

A patient has been diagnosed with venous stasis. Which of these findings would the nurse most likely observe? a. Unilateral cool foot b. Thin, shiny, atrophic skin c. Pallor of the toes and cyanosis of the nail beds d. Brownish discoloration to the skin of the lower leg

ANS: A brownish discoloration to the skin of the lower leg A brown discoloration occurs with chronic venous stasis as a result of hemosiderin deposits (a by-product of red blood cell degradation). Pallor, cyanosis, atrophic skin, and unilateral coolness are all signs associated with arterial problems.

The nurse knows that which statement is true regarding the pain experienced by infants?

ANS: A procedure that induces pain in adults will also induce pain in the infant. If a procedure or disease process causes pain in an adult, then it will also cause pain in an infant. Physiologic changes cannot be used exclusively to confirm or deny pain because other factors, such as medications, fluid status, or stress may cause physiologic changes. The Faces Pain Scale—Revised can be used starting at around age 4 years.

A semiconscious woman is brought to the emergency department after she was found on the floor in her kitchen. Her face, nail beds, lips, and oral mucosa are a bright cherry-red color. The nurse suspects that this coloring is due to: a. Polycythemia. b. Carbon monoxide poisoning. c. Carotenemia. d. Uremia.

ANS: B A bright cherry-red coloring in the face, upper torso, nail beds, lips, and oral mucosa appears in cases of carbon monoxide poisoning.

The nurse is preparing for a certification course in skin care and needs to be familiar with the various lesions that may be identified on assessment of the skin. Which of the following definitions are correct? Select all that apply. a. Petechiae: Tiny punctate hemorrhages, 1 to 3 mm, round and discrete, dark red, purple, or brown in color b. Bulla: Elevated, circumscribed lesion filled with turbid fluid (pus) c. Papule: Hypertrophic scar d. Vesicle: Known as a friction blister e. Nodule: Solid, elevated, and hard or soft growth that is larger than 1 cm

ANS: A, D, E A pustule is an elevated, circumscribed lesion filled with turbid fluid (pus). A hypertrophic scar is a keloid. A bulla is larger than 1 cm and contains clear fluid. A papule is solid and elevated but measures less than 1 cm.

The nurse is preparing for a certification course in skin care and needs to be familiar with the various lesions that may be identified on assessment of the skin. Which of the following definitions are correct? Select all that apply. a. Petechiae: Tiny punctate hemorrhages, 1 to 3 mm, round and discrete, dark red, purple, or brown in color b. Bulla: Elevated, circumscribed lesion filled with turbid fluid (pus) c. Papule: Hypertrophic scar d. Vesicle: Known as a friction blister e. Nodule: Solid, elevated, and hard or soft growth that is larger than 1 cm

ANS: A, D, E A pustule is an elevated, circumscribed lesion filled with turbid fluid (pus). A hypertrophic scar is a keloid. A bulla is larger than 1 cm and contains clear fluid. A papule is solid and elevated but measures less than 1 cm. DIF: Cognitive Level: Understanding (Comprehension) MSC: Client Needs: Physiologic Integrity: Physiologic Adaptation

30. The nurse will perform a palpated pressure before auscultating blood pressure. The reason for this is to: a. More clearly hear the Korotkoff sounds. b. Detect the presence of an auscultatory gap. c. Avoid missing a falsely elevated blood pressure. d. More readily identify phase IV of the Korotkoff sounds.

ANS: B Inflation of the cuff 20 to 30 mm Hg beyond the point at which a palpated pulse disappears will avoid missing an auscultatory gap, which is a period when the Korotkoff sounds disappear during auscultation.

During a digital examination of the rectum, the nurse notices that the patient has hard feces in the rectum. The patient complains of feeling "full," has a distended abdomen, and states that she has not had a bowel movement "for several days." The nurse suspects which condition? A) Rectal polyp B) Fecal impaction C) Rectal abscess D) Rectal prolapse

ANS: B A fecal impaction is a collection of hard, desiccated feces in the rectum. The obstruction often results from decreased bowel motility, in which more water is reabsorbed from the stool. See Table 25-2 for descriptions of rectal polyp and abscess; See Table 25-1 for description of rectal prolapse

A 40-year-old black man is in the office for his annual physical. Which statement regarding the prostate-specific antigen (PSA) blood test is true, according to the American Cancer Society? The PSA: A) should be done with this visit. B) should be done at age 45 years. C) should be done at age 50 years. D) is only necessary if there is a family history of prostate cancer.

ANS: B According to the American Cancer Society (2006) the PSA blood test should be done annually for black men beginning at age 45 years, and annually for all other men over age 50 years.

A woman has just been diagnosed with HPV, or genital warts. The nurse should counsel her to receive regular examinations because this virus makes her at a higher risk for _____ cancer. A) uterine B) cervical C) ovarian D) endometrial

ANS: B HPV is the virus responsible for most cases of cervical cancer, not the other options.

A married couple has come to the clinic seeking advice on pregnancy. They have been trying to conceive for 4 months and have not been successful. What should the nurse do first? A) Ascertain whether either of them has been using broad-spectrum antibiotics. B) Explain that couples are considered infertile after 1 year of unprotected intercourse. C) Immediately refer the woman to an expert in pelvic inflammatory disease—the most common cause of infertility. D) Explain that couples are considered infertile after 3 months of engaging in unprotected intercourse and that they will need a referral to a fertility expert.

ANS: B Infertility is considered after 1 year of engaging in unprotected sexual intercourse without conceiving. The other actions are not appropriate

During a skin assessment, the nurse notices that a Mexican-American patient has skin that is yellowish-brown; however, the skin on the hard and soft palate is pink and the patient's scleras are not yellow. From this finding, the nurse could probably rule out: a. Pallor b. Jaundice c. Cyanosis d. Iron deficiency

ANS: B Jaundice is exhibited by a yellow color, which indicates rising levels of bilirubin in the blood. Jaundice is first noticed in the junction of the hard and soft palate in the mouth and in the scleras.

The nurse is assessing the skin of a patient who has acquired immunodeficiency syndrome (AIDS) and notices multiple patchlike lesions on the temple and beard area that are faint pink in color. The nurse recognizes these lesions as: a. Measles (rubeola). b. Kaposi's sarcoma. c. Angiomas. d. Herpes zoster.

ANS: B Kaposi's sarcoma is a vascular tumor that, in the early stages, appears as multiple, patchlike, faint pink lesions over the patient's temple and beard areas. Measles is characterized by a red-purple maculopapular blotchy rash that appears on the third or fourth day of illness. The rash is first observed behind the ears, spreads over the face, and then spreads over the neck, trunk, arms, and legs. Cherry (senile) angiomas are small (1 to 5 mm), smooth, slightly raised bright red dots that commonly appear on the trunk in all adults over 30 years old. Herpes zoster causes vesicles up to 1 cm in size that are elevated with a cavity containing clear fluid.

A few days after a summer hiking trip, a 25-year-old man comes to the clinic with a rash. On examination, the nurse notes that the rash is red, macular, with a bull's eye pattern across his midriff and behind his knees. The nurse suspects: a. Rubeola. b. Lyme disease. c. Allergy to mosquito bites. d. Rocky Mountain spotted fever.

ANS: B Lyme disease occurs in people who spend time outdoors in May through September. The first disease state exhibits the distinctive bull's eye and a red macular or papular rash that radiates from the site of the tick bite with some central clearing. The rash spreads 5 cm or larger, and is usually in the axilla, midriff, inguinal, or behind the knee, with regional lymphadenopathy.

A 52-year-old patient states that when she sneezes or coughs she "wets herself a little." She is very concerned that something may be wrong with her. The nurse suspects that the problem is: A) dysuria. B) stress incontinence. C) hematuria. D) urge incontinence.

ANS: B Stress incontinence is involuntary urine loss with physical strain, sneezing, or coughing. Dysuria is pain or burning with urination. Hematuria is bleeding with urination. Urge incontinence is involuntary urine loss but it occurs due to an overactive detrusor muscle in the bladder that contracts and causes an urgent need to void

During an internal examination, the nurse notices that the cervix bulges outside the introitus when the patient is asked to strain. The nurse will document this as: A) uterine prolapse, graded first degree. B) uterine prolapse, graded second degree. C) uterine prolapse, graded third degree. D) a normal finding.

ANS: B The cervix should not be found to bulge into the vagina. Uterine prolapse is graded as follows: first degree—cervix appears at introitus with straining; second degree—cervix bulges outside introitus with straining; and third degree—whole uterus protrudes, even without straining (essentially, uterus is inside out).

The external male genital structures include the: A) testis. B) scrotum. C) epididymis. D) vas deferens.

ANS: B The external male genital structures include the penis and scrotum. The testis, epididymis, and vas deferens are internal structures.

During an examination of an aging male, the nurse recognizes that normal changes to expect would be: A) a change in scrotal color. B) a decrease in the size of the penis. C) enlargement of the testes and scrotum. D) an increase in the number of rugae over the scrotal sac.

ANS: B When assessing the genitals of an older man, the nurse may notice thinner, graying pubic hair and a decrease in the size of the penis. The size of the testes may be decreased, they may feel less firm, and the scrotal sac is pendulous with less rugae. There is no change in scrotal color.

A patient calls the clinic for instructions before having a Papanicolaou (Pap) smear. The most appropriate instructions from the nurse are: A) "If you are menstruating, please use pads to avoid placing anything into the vagina." B) "Avoid intercourse, inserting anything into the vagina, or douching within 24 hours of your appointment." C) "If you suspect that you have a vaginal infection, please gather a sample of the discharge to bring with you." D) "We would like you to use a mild saline douche before your examination. You may pick this up in our office."

ANS: B When instructing a patient before a Papanicolaou (Pap) smear is obtained, the nurse should follow these guidelines: Do not obtain during the woman's menses or if a heavy infectious discharge is present. Instruct the woman not to douche, have intercourse, or put anything into the vagina within 24 hours before collecting the specimens. Any specimens will be obtained during the visit, not beforehand.

A semiconscious woman is brought to the emergency department after she was found on the floor in her kitchen. Her face, nail beds, lips, and oral mucosa are a bright cherry-red color. The nurse suspects that this coloring is due to: a. Polycythemia. b. Carbon monoxide poisoning. c. Carotenemia. d. Uremia.

ANS: B A bright cherry-red coloring in the face, upper torso, nail beds, lips, and oral mucosa appears in cases of carbon monoxide poisoning. DIF: Cognitive Level: Analyzing (Analysis) MSC: Client Needs: Physiologic Integrity: Physiologic Adaptation

During a skin assessment, the nurse notices that a Mexican-American patient has skin that is yellowish- brown; however, the skin on the hard and soft palate is pink and the patients scleras are not yellow. From this finding, the nurse could probably rule out: a. Pallor b. Jaundice c. Cyanosis d. Iron deficiency

ANS: B Jaundice is exhibited by a yellow color, which indicates rising levels of bilirubin in the blood. Jaundice is first noticed in the junction of the hard and soft palate in the mouth and in the scleras. DIF: Cognitive Level: Analyzing (Analysis) MSC: Client Needs: Physiologic Integrity: Physiologic Adaptation

5. A patients blood pressure is 118/82 mm Hg. He asks the nurse, What do the numbers mean? The nurses best reply is: a. The numbers are within the normal range and are nothing to worry about. b. The bottom number is the diastolic pressure and reflects the stroke volume of the heart. c. The top number is the systolic blood pressure and reflects the pressure of the blood against the arteries when the heart contracts. d. The concept of blood pressure is difficult to understand. The primary thing to be concerned about is the top number, or the systolic blood pressure.

ANS: C The systolic pressure is the maximum pressure felt on the artery during left ventricular contraction, or systole. The diastolic pressure is the elastic recoil, or resting, pressure that the blood constantly exerts in between each contraction. The nurse should answer the patients question and use terms he can understand.

8. The nurse is preparing to measure the length, weight, chest, and head circumference of a 6-month-old infant. Which measurement technique is correct? a. Measuring the infants length by using a tape measure b. Weighing the infant by placing him or her on an electronic standing scale c. Measuring the chest circumference at the nipple line with a tape measure d. Measuring the head circumference by wrapping the tape measure over the nose and cheekbones

ANS: C To measure the chest circumference, the tape is encircled around the chest at the nipple line. The length should be measured on a horizontal measuring board. Weight should be measured on a platform-type balance scale. Head circumference is measured with the tape around the head, aligned at the eyebrows, and at the prominent frontal and occipital bonesthe widest span is correct.

36. The nurse is helping another nurse to take a blood pressure reading on a patients thigh. Which action is correct regarding thigh pressure? a. Either the popliteal or femoral vessels should be auscultated to obtain a thigh pressure. b. The best position to measure thigh pressure is the supine position with the knee slightly bent. c. If the blood pressure in the arm is high in an adolescent, then it should be compared with the thigh pressure. d. The thigh pressure is lower than the pressure in the arm, which is attributable to the distance away from the heart and the size of the popliteal vessels.

ANS: C When blood pressure measured at the arm is excessively high, particularly in adolescents and young adults, it is compared with thigh pressure to check for coarctation of the aorta. The popliteal artery is auscultated for the reading. Generally, thigh pressure is higher than that of the arm; however, if coarctation of the artery is present, then arm pressures are higher than thigh pressures.

A patient comes in for a physical examination and complains of "freezing to death" while waiting for her examination. The nurse notes that her skin is pale and cool and attributes this finding to: a. Venous pooling. b. Peripheral vasodilation. c. Peripheral vasoconstriction. d. Decreased arterial perfusion.

ANS: C A chilly or air-conditioned environment causes vasoconstriction, which results in false pallor and coolness (see Table 12-1).

A 13-year-old girl is interested in obtaining information about the cause of her acne. The nurse should share with her that acne: a. Is contagious. b. Has no known cause. c. Is caused by increased sebum production. d. Has been found to be related to poor hygiene.

ANS: C Approximately 90% of males and 80% of females will develop acne; causes are increased sebum production and epithelial cells that do not desquamate normally.

During the interview a patient reveals that she has some vaginal discharge. She is worried that it may be a sexually transmitted infection. The nurse's most appropriate response to this would be: A) "Oh, don't worry. Some cyclic vaginal discharge is normal." B) "Have you been engaging in unprotected sexual intercourse?" C) "I'd like some information about the discharge. What color is it?" D) "Have you had any urinary incontinence associated with the discharge?"

ANS: C Ask questions that help the patient reveal more information about her symptoms in a nonthreatening manner. Assess vaginal discharge further by asking about the amount, color, and odor. Normal vaginal discharge is small, clear or cloudy, and always nonirritating.

The changes normally associated with menopause occur generally because the cells in the reproductive tract are: A) aging. B) becoming fibrous. C) estrogen dependent. D) able to respond to estrogen.

ANS: C Because cells in the reproductive tract are estrogen dependent, decreased estrogen levels during menopause bring dramatic physical changes. The other options are not correct.

The nurse has discovered decreased skin turgor in a patient and knows that this finding is expected in which condition? a. Severe obesity b. Childhood growth spurts c. Severe dehydration d. Connective tissue disorders such as scleroderma

ANS: C Decreased skin turgor is associated with severe dehydration or extreme weight loss. DIF: Cognitive Level: Understanding (Comprehension) MSC: Client Needs: Physiologic Integrity: Physiologic Adaptation

During a genital examination, the nurse notices that a male patient has clusters of small vesicles on the glans, surrounded by erythema. The nurse recognizes that these lesions are: A) Peyronie disease. B) genital warts. C) genital herpes. D) syphilitic cancer.

ANS: C Genital herpes, or HSV-2, infections are indicated with clusters of small vesicles with surrounding erythema, which are often painful and erupt on the glans or foreskin. See Table 24-4 for descriptions of the other options.

An accessory glandular structure for the male genital organs is the: A) testis. B) penis. C) prostate. D) vas deferens.

ANS: C Glandular structures accessory to the male genital organs are the prostate, seminal vesicles, and bulbourethral glands.

During a bimanual examination, the nurse detects a solid tumor on the ovary that is heavy and fixed, with a poorly defined mass. This finding is suggestive of: A) an ovarian cyst. B) endometriosis. C) ovarian cancer. D) an ectopic pregnancy.

ANS: C Ovarian tumors that are solid, heavy, and fixed, with poorly defined mass are suggestive of malignancy. Benign masses may feel mobile and solid. An ovarian cyst may feel smooth, round, fluctuant, mobile, and nontender. With an ectopic pregnancy, the examiner may feel a palpable, tender pelvic mass that is solid, mobile, and unilateral. Endometriosis may have masses (in various locations in the pelvic area) that are small, firm, nodular and tender to palpation, with enlarged ovaries.

A physician has diagnosed a patient with purpura. After leaving the room, a nursing student asks the nurse what the physician saw that led to that diagnosis. The nurse should say, "The physician is referring to the: a. "Blue dilation of blood vessels in a star-shaped linear pattern on the legs." b. "Fiery red, star-shaped marking on the cheek that has a solid circular center." c. "Confluent and extensive patch of petechiae and ecchymoses on the feet." d. "Tiny areas of hemorrhage that are less than 2 mm, round, discrete, and dark red in color."

ANS: C Purpura is a confluent and extensive patch of petechiae and ecchymoses and a flat macular hemorrhage observed in generalized disorders such as thrombocytopenia and scurvy. The blue dilation of blood vessels in a star-shaped linear pattern on the legs describes a venous lake. The fiery red, star-shaped marking on the cheek that has a solid circular center describes a spider or star angioma. The tiny areas of hemorrhage that are less than 2 mm, round, discrete, and dark red in color describes petechiae.

A patient has been admitted for severe psoriasis. The nurse expects to see what finding in the patient's fingernails? a. Splinter hemorrhages b. Paronychia c. Pitting d. Beau lines

ANS: C Sharply defined pitting and crumbling of the nails, each with distal detachment characterize pitting nails and are associated with psoriasis. (See Table 12-13 for descriptions of the other terms.)

The nurse notices that a patient has had a pale, yellow, greasy stool, or steatorrhea, and recalls that this is caused by: A) occult bleeding. B) absent bile pigment. C) increased fat content. D) ingestion of bismuth preparations.

ANS: C Steatorrhea (pale, yellow, greasy stool) is caused by increased fat content in the stools, as in malabsorption syndrome. Occult bleeding and ingestion of bismuth products cause black stool, and absent bile pigment causes gray, tan stool.

The nurse is preparing for an internal genitalia examination of a woman. Which order of the examination is correct? A) Bimanual, speculum, rectovaginal B) Speculum, rectovaginal, bimanual C) Speculum, bimanual, rectovaginal D) Rectovaginal, bimanual, speculum

ANS: C The correct sequence is speculum examination, then bimanual examination after removing the speculum, and then rectovaginal examination. The examiner should change gloves before performing the rectovaginal examination to avoid spreading any possible infection

A newborn infant is in the clinic for a well-baby checkup. The nurse observes the infant for the possibility of fluid loss because of which of these factors? a. Subcutaneous fat deposits are high in the newborn. b. Sebaceous glands are overproductive in the newborn. c. The newborn's skin is more permeable than that of the adult. d. The amount of vernix caseosa dramatically rises in the newborn.

ANS: C The newborn's skin is thin, smooth, and elastic and is relatively more permeable than that of the adult; consequently, the infant is at greater risk for fluid loss. The subcutaneous layer in the infant is inefficient, not thick, and the sebaceous glands are present but decrease in size and production. Vernix caseosa is not produced after birth.

Which statement would be most appropriate when the nurse is introducing the topic of sexual relationships during an interview? A) "Now it is time to talk about your sexual history. When did you first have intercourse?" B) "Women often feel dissatisfied with their sexual relationships. Would it be okay to discuss this now?" C) "Often women have questions about their sexual relationship and how it affects their health. Do you have any questions?" D) "Most women your age have had more than one sexual partner. How many would you say you have had?"

ANS: C The nurse should begin with an open-ended question to assess individual needs. The nurse should include appropriate questions as a routine part of the history, because doing so communicates that the nurse accepts the individual's sexual activity and believes it is important. The nurse's comfort with discussion prompts the patient's interest and possibly relief that the topic has been introduced. This establishes a database for comparison with any future sexual activities and provides an opportunity to screen sexual problems.

The nurse is examining only the rectal area of a woman and should place the woman in what position? A) Lithotomy position B) Prone position C) Left lateral decubitus position D) Bending over the table while standing

ANS: C The nurse should place the female patient in lithotomy position if examining genitalia as well; use the left lateral decubitus position for the rectal area alone.

A 46-year-old man requires assessment of his sigmoid colon. The nurse is aware that which of these is most appropriate for this examination? A) Proctoscope B) Ultrasound C) Colonoscope D) Rectal exam with an examining finger

ANS: C The sigmoid colon is 40 cm long and is accessible to examination only with the colonoscope. The other responses are not appropriate for examination of the entire sigmoid colon.

A 46-year-old woman is in the clinic for her annual gynecologic examination. She voices a concern about ovarian cancer because her mother and sister died of it. The nurse knows that which of these statements is correct regarding ovarian cancer? A) Ovarian cancer rarely has any symptoms. B) The Pap smear detects the presence of ovarian cancer. C) Women at high risk for ovarian cancer should have annual transvaginal ultrasonography for screening. D) Women over age 40 years should have a thorough pelvic examination every 3 years.

ANS: C With ovarian cancer, the patient may have abdominal pain, pelvic pain, increased abdominal size, bloating, and nonspecific gastrointestinal symptoms, or she may be asymptomatic. The Pap smear does not detect the presence of ovarian cancer. Annual transvaginal ultrasonography may detect ovarian cancer at an earlier stage in women who are at high risk for it.

A patient comes in for a physical examination and complains of freezing to death while waiting for her examination. The nurse notes that her skin is pale and cool and attributes this finding to: a. Venous pooling. b. Peripheral vasodilation. c. Peripheral vasoconstriction. d. Decreased arterial perfusion.

ANS: C A chilly or air-conditioned environment causes vasoconstriction, which results in false pallor and coolness. DIF: Cognitive Level: Applying (Application) MSC: Client Needs: Physiologic Integrity: Basic Care and Comfort

A black patient is in the intensive care unit because of impending shock after an accident. The nurse expects to find what characteristics in this patients skin? a. Ruddy blue. b. Generalized pallor. c. Ashen, gray, or dull. d. Patchy areas of pallor.

ANS: C Pallor attributable to shock, with decreased perfusion and vasoconstriction, in black-skinned people will cause the skin to appear ashen, gray, or dull. DIF: Cognitive Level: Analyzing (Analysis) MSC: Client Needs: Physiologic Integrity: Physiologic Adaptation

15. When assessing a 75-year-old patient who has asthma, the nurse notes that he assumes a tripod position, leaning forward with arms braced on the chair. On the basis of this observation, the nurse should: a. Assume that the patient is eager and interested in participating in the interview. b. Evaluate the patient for abdominal pain, which may be exacerbated in the sitting position. c. Assume that the patient is having difficulty breathing and assist him to a supine position. d. Recognize that a tripod position is often used when a patient is having respiratory difficulties.

ANS: D Assuming a tripod positionleaning forward with arms braced on chair armsoccurs with chronic pulmonary disease. The other actions or assumptions are not correct.

During a health history, a patient tells the nurse that he has trouble in starting his urine stream. This problem is known as: A) urgency. B) dribbling. C) frequency. D) hesitancy.

ANS: D Hesitancy is trouble in starting the urine stream. Urgency is the feeling that one cannot wait to urinate. Dribbling is the lost of urine before or after the main act of urination. Frequency is urinating more often than usual.

The nurse is performing a genital examination on a male patient and notices urethral drainage. When collecting urethral discharge for microscopic examination and culture, the nurse should: A) ask the patient to urinate into a sterile cup. B) ask the patient to obtain a specimen of semen. C) insert a cotton-tipped applicator into the urethra. D) compress the glans between the examiner's thumb and forefinger and collect any discharge.

ANS: D If urethral discharge is noticed, then the examiner should collect a smear for microscopic examination and culture by compressing the glans anteroposteriorly between the thumb and forefinger. The other options are not correct actions

A man has come in to the clinic for a skin assessment because he is worried he might have skin cancer. During the skin assessment the nurse notices several areas of pigmentation that look greasy, dark, and "stuck on" his skin. Which is the best prediction? a. Senile lentigines, which do not become cancerous b. Actinic keratoses, which are precursors to basal cell carcinoma c. Acrochordons, which are precursors to squamous cell carcinoma d. Seborrheic keratoses, which do not become cancerous

ANS: D Seborrheic keratoses appear like dark, greasy, "stuck-on" lesions that primarily develop on the trunk. These lesions do not become cancerous. Senile lentigines are commonly called liver spots and are not precancerous. Actinic (senile or solar) keratoses are lesions that are red-tan scaly plaques that increase over the years to become raised and roughened. They may have a silvery-white scale adherent to the plaque. They occur on sun-exposed surfaces and are directly related to sun exposure. They are premalignant and may develop into squamous cell carcinoma. Acrochordons are skin tags and are not precancerous.

A 25-year-old woman comes to the emergency department with a sudden fever of 101° F and abdominal pain. Upon examination, the nurse notices that she has rigid, boardlike lower abdominal musculature. When the nurse tries to perform a vaginal examination, the patient has severe pain when the uterus and cervix are moved. The nurse knows that these signs and symptoms are suggestive of: A) endometriosis. B) uterine fibroids. C) ectopic pregnancy. D) pelvic inflammatory disease.

ANS: D These signs and symptoms are suggestive of acute pelvic inflammatory disease, also known as acute salpingitis. See Table 26-7. For description of endometriosis and uterine fibroids, see Table 26-6; for description of ectopic pregnancy, see Table 26-7.

During a history, the patient states, "It really hurts back there, and sometimes it itches, too. I have even seen blood on the tissue when I have a bowel movement. Is there something there?" The nurse should expect to see which of these upon examination of the anus? A) Rectal prolapse B) Internal hemorrhoid C) External hemorrhoid that has resolved D) External hemorrhoid that is thrombosed

ANS: D These symptoms are consistent with an external hemorrhoid. An external hemorrhoid, when thrombosed, contains clotted blood and becomes a painful, swollen, shiny blue mass that itches and bleeds with defecation. When the external hemorrhoid resolves, it leaves a flabby, painless skin sac around the anal orifice. An internal hemorrhoid is not palpable, but may appear as a red mucosal mass when the person performs a Valsalva maneuver. A rectal prolapse appears as a moist, red doughnut with radiating lines

When the nurse is performing a genital examination on a male patient, which of these actions is correct? A) Auscultate for the presence of a bruit over the scrotum. B) Palpate for the vertical chain of lymph nodes along the groin inferior to the inguinal ligament. C) Palpate the inguinal canal only if there is a bulge present in the inguinal region during inspection. D) Have the patient shift his weight onto the left (unexamined) leg when palpating for a hernia on the right side.

ANS: D When palpating for the presence of a hernia on the right side, ask the male patient to shift his weight onto the left (unexamined) leg. It is not appropriate to auscultate for a bruit over the scrotum. When palpating for lymph notes, palpate the horizontal chain. The inguinal canal should be palpated whether or not a bulge is present.

When testing stool for occult blood, the nurse is aware that a false-positive result may occur with: A) absent bile pigment. B) increased fat content. C) increased ingestion of iron medication. D) a large amount of red meat within the last 3 days.

ANS: D When testing for occult blood, a false-positive finding may occur if the person has ingested significant amounts of red meat within 3 days of the test. Absent bile pigment causes the stools to be gray or tan in color. Increased fat content causes the stool to be pale, yellow, and greasy. Increased ingestion of iron medication causes the stool to be black in color.

When auscultating the lungs of an adult patient, the nurse notes that over the posterior lower lobes low-pitched, soft breath sounds are heard, with inspiration being longer than expiration. The nurse interprets that these are: a. Normally auscultated over the trachea. b. Bronchial breath sounds and normal in that location. c. Vesicular breath sounds and normal in that location. d. Bronchovesicular breath sounds and normal in that location.

ANS: vesicular breath sounds and are normal in that location. Vesicular breath sounds are low-pitched, soft sounds with inspiration being longer than expiration. These breath sounds are expected over peripheral lung fields where air flows through smaller bronchioles and alveoli.

A 19-year-old college student is brought to the emergency department with a severe headache he describes as "Like nothing I've ever had before." His temperature is 104° F, and he has a stiff neck. What do these signs and symptoms suggest? 1. Head injury 2. Cluster headache 3. Migraine headache 4. Meningeal inflammation

ANS:4 Acute onset of neck stiffness and pain along with headache and fever occurs with meningeal inflammation. A severe headache in an adult or child who has never had it before is a red flag.

The nurse is assessing a new patient who has recently immigrated to the United States. Which question is appropriate to add to the health history? a."Why did you come to the United States?" b."When did you come to the United States and from what country?" c."What made you leave your native country?" d."Are you planning to return to your home?"

B Biographic data, such as when the person entered the United States and from what country, are appropriate additions to the health history. The other answers do not reflect appropriate questions.

While performing the otoscopic examination of a 3-year-old boy who has been pulling on his left ear, the nurse finds that his left tympanic membrane is bright red and that the light reflex is not visible. The nurse interprets these findings to indicate a(n): a. Fungal infection. b. Acute otitis media. c. Perforation of the eardrum. d. Cholesteatoma.

B Absent or distorted light reflex and a bright red color of the eardrum are indicative of acute otitis media. (See Table 15-5 for descriptions of the other conditions.)

13. A patient tells the nurse that his food just doesn't have any taste anymore. The nurse's best response would be:

B) "When did you first notice this change?"

The nurse recognizes that the concept of prevention in describing health is essential because: a. Disease can be prevented by treating the external environment. b. The majority of deaths among Americans under age 65 years are not preventable. c. Prevention places the emphasis on the link between health and personal behavior. d. The means to prevention is through treatment provided by primary health care practitioners.

C A natural progression to prevention rounds out the present concept of health. Guidelines to prevention place the emphasis on the link between health and personal behavior.

The nurse is performing an assessment on a 65-year-old man. He reports a crusty nodule behind the pinna. It intermittently bleeds and has not healed over the past 6 months. On physical assessment, the nurse finds an ulcerated crusted nodule with an indurated base. The preliminary analysis in this situation is that this: a. Is most likely a benign sebaceous cyst. b. Is most likely a keloid. c. Could be a potential carcinoma, and the patient should be referred for a biopsy. d. Is a tophus, which is common in the older adult and is a sign of gout.

C An ulcerated crusted nodule with an indurated base that fails to heal is characteristic of a carcinoma. These lesions fail to heal and intermittently bleed. Individuals with such symptoms should be referred for a biopsy (see Table 15-2). The other responses are not correct.

During an assessment of a 20-year-old Asian patient, the nurse notices that he has dry, flaky cerumen in his canal. What is the significance of this finding? This finding: a. Is probably the result of lesions from eczema in his ear. b. Represents poor hygiene. c. Is a normal finding, and no further follow-up is necessary. d. Could be indicative of change in cilia; the nurse should assess for hearing loss

C Asians and Native Americans are more likely to have dry cerumen, whereas Blacks and Whites usually have wet cerumen.

The nurse is preparing to percuss the abdomen of a patient. The purpose of the percussion is to assess the __________ of the underlying tissue. a.Turgor b.Texture c.Density d.Consistency

C Percussion yields a sound that depicts the location, size, and density of the underlying organ. Turgor and texture are assessed with palpation.

The nurse is preparing to use an otoscope for an examination. Which statement is true regarding the otoscope? The otoscope: a.Is often used to direct light onto the sinuses. b.Uses a short, broad speculum to help visualize the ear. c.Is used to examine the structures of the internal ear. d.Directs light into the ear canal and onto the tympanic membrane.

D The otoscope directs light into the ear canal and onto the tympanic membrane that divides the external and middle ear. A short, broad speculum is used to visualize the nares.

9. The nurse is seeing for the first time a patient who has no history of nutrition-related problems. The initial nutritional screening should include which activity?

D) Measurement of weight and weight history

A 5-year-old boy is being admitted to the hospital to have his tonsils removed. Which information should the nurse collect before this procedure? a.Child's birth weight b.Age at which he crawled c.Whether the child has had the measles d.Child's reactions to previous hospitalizations

D How the child reacted to previous hospitalizations and any complications should be assessed. If the child reacted poorly, then he or she may be afraid now and will need special preparation for the examination that is to follow. The other items are not significant for the procedure.

During an interview, the patient states he has the sensation that "everything around him is spinning." The nurse recognizes that the portion of the ear responsible for this sensation is the: a. Cochlea. b. CN VIII. c. Organ of Corti. d. Labyrinth.

D If the labyrinth ever becomes inflamed, then it feeds the wrong information to the brain, creating a staggering gait and a strong, spinning, whirling sensation called vertigo.

When reviewing the concepts of health, the nurse recalls that the components of holistic health include which of these? a. Disease originates from the external environment. b. The individual human is a closed system. c. Nurses are responsible for a patients health state. d. Holistic health views the mind, body, and spirit as interdependent.

D Consideration of the whole person is the essence of holistic health, which views the mind, body, and spirit as interdependent. The basis of disease originates from both the external environment and from within the person. Both the individual human and the external environment are open systems, continually changing and adapting, and each person is responsible for his or her own personal health state.

The nurse is conducting a child safety class for new mothers. Which factor places young children at risk for ear infections? a. Family history b. Air conditioning c. Excessive cerumen d. Passive cigarette smoke

D Exposure to passive and gestational smoke is a risk factor for ear infections in infants and children.

When examining an infant, the nurse should examine which area first? a.Ear b.Nose c.Throat d.Abdomen

D The least-distressing steps are performed first, saving the invasive steps of the examination of the eye, ear, nose, and throat until last.

20. To assess the muscle mass and fat stores on a 40-year-old woman, the nurse would use:

D) mid-upper arm circumference.

The nurse is asking a patient for his reason for seeking care and asks about the signs and symptoms he is experiencing. Which of these is an example of a symptom? a.Chest pain b.Clammy skin c.Serum potassium level at 4.2 mEq/L d.Body temperature of 100° F

A A symptom is a subjective sensation (e.g., chest pain) that a person feels from a disorder. A sign is an objective abnormality that the examiner can detect on physical examination or in laboratory reports, as illustrated by the other responses.

The nurse knows that developing appropriate nursing interventions for a patient relies on the appropriateness of the __________ diagnosis. a. Nursing b. Medical c. Admission d. Collaborative

A An accurate nursing diagnosis provides the basis for the selection of nursing interventions to achieve outcomes for which the nurse is accountable. The other items do not contribute to the development of appropriate nursing interventions.

3. The nurse is providing nutrition information to the mother of a 1-year-old child. Which of these statements represents accurate information for this age group?

A) It is important to maintain adequate fat and caloric intake.

33. When palpating the abdomen of a 20-year-old patient, the nurse notices the presence of tenderness in the left upper quadrant with deep palpation. Which of these structures is most likely to be involved? a. Spleen b. Sigmoid colon c. Appendix d. Gallbladder

A) Spleen The spleen is located in the left upper quadrant of the abdomen. The gallbladder is in the right upper quadrant, the sigmoid colon is in the left lower quadrant, and the appendix is in the right lower quadrant.

36. During report, the student nurse hears that a patient has "hepatomegaly" and recognizes that this term refers to: a. Enlarged liver. b. Enlarged spleen. c. Distended bowel. d. Excessive diarrhea.

A) an enlarged liver. The term hepatomegaly refers to an enlarged liver. The term splenomegaly refers to an enlarged spleen. The other responses are not correct.

19. A nurse notices that a patient has ascites, which indicates the presence of: a. Fluid. b. Feces. c. Flatus. d. Fibroid tumors.

A) fluid. Ascites is free fluid in the peritoneal cavity and occurs with heart failure, portal hypertension, cirrhosis, hepatitis, pancreatitis, and cancer.

17. When considering a nutritional assessment, the nurse is aware that the most common anthropometric measurements include:

A) height and weight.

24. In teaching a patient how to determine total body fat at home, the nurse includes instructions to obtain measurements of:

A) height and weight.

28. After completing a diet assessment on a 30-year-old woman, the nurse suspects that she may be deficient in iron. Laboratory studies to obtain to verify this condition would be:

A) hemoglobin and hematocrit.

24. During an assessment of a newborn infant, the nurse recalls that pyloric stenosis would be manifested by: a. Projectile vomiting. b. Hypoactive bowel activity. c. Palpable olive-sized mass in the right lower quadrant. d. Pronounced peristaltic waves crossing from right to left.

A) projectile vomiting. Significant peristalsis, together with projectile vomiting, in the newborn suggests pyloric stenosis. After feeding, pronounced peristaltic waves cross from left to right, leading to projectile vomiting. One can also palpate an olive-sized mass in the right upper quadrant.

How should the nurse document mild, slight pitting edema present at the ankles of a pregnant patient? a. 1+/0-4+ b. 3+/0-4+ c. 4+/0-4+ d. Brawny edema

ANS: 1+/0-4+ If pitting edema is present, then the nurse should grade it on a scale of 1+ (mild) to 4+ (severe). Brawny edema appears as nonpitting edema and feels hard to the touch.

The temporomandibular joint is just below the temporal artery and anterior to the:1. hyoid. 2. vagus. 3. tragus. 4. mandible.

ANS: 3 The temporomandibular joint is just below the temporal artery and anterior to the tragus.

A patient visits the clinic because he has recently noticed that the left side of his mouth is paralyzed. He states that he cannot raise his eyebrow or whistle. The nurse suspects that he has: 1. Cushing's syndrome. 2. Parkinson's syndrome. 3. Bell's palsy. 4. had a cerebrovascular accident (stroke).

ANS: 4 With an upper motor neuron lesion (as with CVA) the patient will have paralysis of lower facial muscles, but the upper half of the face is not affected owing to the intact nerve from the unaffected hemisphere. The person is still able to wrinkle the forehead and close the eyes.

29. A student is late for his appointment and has rushed across campus to the health clinic. The nurse should: a. Allow 5 minutes for him to relax and rest before checking his vital signs. b. Check the blood pressure in both arms, expecting a difference in the readings because of his recent exercise. c. Immediately monitor his vital signs on his arrival at the clinic and then 5 minutes later, recording any differences. d. Check his blood pressure in the supine position, which will provide a more accurate reading and will allow him to relax at the same time.

ANS: A A comfortable, relaxed person yields a valid blood pressure. Many people are anxious at the beginning of an examination; the nurse should allow at least a 5-minute rest period before measuring blood pressure.

19. Which technique is correct when the nurse is assessing the radial pulse of a patient? The pulse is counted for: a. 1 minute, if the rhythm is irregular. b. 15 seconds and then multiplied by 4, if the rhythm is regular. c. 2 full minutes to detect any variation in amplitude. d. 10 seconds and then multiplied by 6, if the patient has no history of cardiac abnormalities.

ANS: A Recent research suggests that the 30-second interval multiplied by 2 is the most accurate and efficient technique when heart rates are normal or rapid and when rhythms are regular. If the rhythm is irregular, then the pulse is counted for 1 full minute.

24. The nurse is assessing the vital signs of a 3-year-old patient who appears to have an irregular respiratory pattern. How should the nurse assess this childs respirations? a. Respirations should be counted for 1 full minute, noticing rate and rhythm. b. Childs pulse and respirations should be simultaneously checked for 30 seconds. c. Childs respirations should be checked for a minimum of 5 minutes to identify any variations in his or her respiratory pattern. d. Patients respirations should be counted for 15 seconds and then multiplied by 4 to obtain the number of respirations per minute.

ANS: A Respirations are counted for 1 full minute if an abnormality is suspected. The other responses are not correct actions.

30. The mother of a 10-month-old infant tells the nurse that she has noticed that her son becomes blue when he is crying and that the frequency of this is increasing. He is also not crawling yet. During the examination the nurse palpates a thrill at the left lower sternal border and auscultates a loud systolic murmur in the same area. What would be the most likely cause of these findings? A. Tetralogy of Fallot B. Atrial septal defect C. Patent ductus arteriosus D. Ventricular septal defect

ANS: A Tetralogy of Fallot subjective findings include (1) severe cyanosis, not in the first months of life but developing as the infant grows, and right ventricle outflow (i.e., pulmonic) stenosis gets worse; (2) cyanosis with crying and exertion at first, then at rest; and (3) slowed development. Objective findings include (1) thrill palpable at left lower sternal border; (2) S1 normal, S2 has A2 loud and P2 diminished or absent; and (3) murmur is systolic, loud, crescendo-decrescendo.

1. The nurse is performing a general survey. Which action is a component of the general survey? a. Observing the patients body stature and nutritional status b. Interpreting the subjective information the patient has reported c. Measuring the patients temperature, pulse, respirations, and blood pressure d. Observing specific body systems while performing the physical assessment

ANS: A The general survey is a study of the whole person that includes observing the patients physical appearance, body structure, mobility, and behavior.

During an examination, which tests will the nurse collect to screen for cervical cancer? A) Endocervical specimen, cervical scrape, and vaginal pool B) Endocervical specimen, vaginal pool, and acetic acid wash C) Endocervical specimen, KOH preparation, and acetic acid wash D) Cervical scrape, acetic acid wash, saline mount ("wet prep")

ANS: A Laboratories may vary in method, but usually the test consists of three specimens: endocervical specimen, cervical scrape, and vaginal pool. The other tests (acetic acid wash, KOH preparation, and saline mount) are used to test for sexually transmitted infections

A woman is in the clinic for an annual gynecologic examination. The nurse should plan to begin the interview with the: A) menstrual history because it is generally nonthreatening. B) obstetric history because it is the most important information. C) urinary system history because there may be problems in this area as well. D) sexual history because it will build rapport to discuss this first.

ANS: A Menstrual history is usually nonthreatening; thus it is a good place to start. Obstetric, urinary, and sexual histories are also part of the interview but not necessarily the best topics with which to start.

A 45-year-old farmer comes in for a skin evaluation and complains of hair loss on his head. His hair seems to be breaking off in patches, and he notices some scaling on his head. The nurse begins the examination suspecting: a. Tinea capitis. b. Folliculitis. c. Toxic alopecia. d. Seborrheic dermatitis.

ANS: A Tinea capitis is rounded patchy hair loss on the scalp, leaving broken-off hairs, pustules, and scales on the skin, and is caused by a fungal infection. Lesions are fluorescent under a Wood light and are usually observed in children and farmers; tinea capitis is highly contagious. (See Table 12-12, Abnormal Conditions of Hair, for descriptions of the other terms.)

A 75-year-old woman who has a history of diabetes and peripheral vascular disease has been trying to remove a corn on the bottom of her foot with a pair of scissors. The nurse will encourage her to stop trying to remove the corn with scissors because: a. The woman could be at increased risk for infection and lesions because of her chronic disease. b. With her diabetes, she has increased circulation to her foot, and it could cause severe bleeding. c. She is 75 years old and is unable to see; consequently, she places herself at greater risk for selfinjury with the scissors. d. With her peripheral vascular disease, her range of motion is limited and she may not be able to reach the corn safely.

ANS: A A personal history of diabetes and peripheral vascular disease increases a persons risk for skin lesions in the feet or ankles. The patient needs to seek a professional for assistance with corn removal. DIF: Cognitive Level: Applying (Application) MSC: Client Needs: Physiologic Integrity: Reduction of Risk Potential

48. The nurse is counting an infants respirations. Which technique is correct? a. Watching the chest rise and fall b. Watching the abdomen for movement c. Placing a hand across the infants chest d. Using a stethoscope to listen to the breath sounds

ANS: B Watching the abdomen for movement is the correct technique because the infants respirations are normally more diaphragmatic than thoracic. The other responses do not reflect correct techniques.

The nurse is attempting to assess the femoral pulse in an obese patient. Which of these actions would be most appropriate? a. The patient is asked to assume a prone position. b. The patient is asked to bend his or her knees to the side in a froglike position. c. The nurse firmly presses against the bone with the patient in a semi-Fowler position. d. The nurse listens with a stethoscope for pulsations; palpating the pulse in an obese person is extremely difficult.

ANS: Ask the patient to bend his or her knees to the side in a froglike position. To help expose the femoral area, particularly in obese people, the nurse should ask the person to bend his or her knees to the side in a froglike position.

15. In assessing the carotid arteries of an older patient with cardiovascular disease, the nurse would: A. Palpate the artery in the upper one third of the neck. B. Listen with the bell of the stethoscope to assess for bruits. C. Palpate both arteries simultaneously to compare amplitude. D. Instruct patient to take slow deep breaths during auscultation.

ANS: B If cardiovascular disease is suspected, then the nurse should auscultate each carotid artery for the presence of a bruit. The nurse should avoid compressing the artery because this could create an artificial bruit, and it could compromise circulation if the carotid artery is already narrowed by atherosclerosis. Avoid excessive pressure on the carotid sinus area higher in the neck; excessive vagal stimulation here could slow down the heart rate, especially in older adults. Palpate only one carotid artery at a time to avoid compromising arterial blood to the brain.

21. When assessing the pulse of a 6-year-old boy, the nurse notices that his heart rate varies with his respiratory cycle, speeding up at the peak of inspiration and slowing to normal with expiration. The nurses next action would be to: a. Immediately notify the physician. b. Consider this finding normal in children and young adults. c. Check the childs blood pressure, and note any variation with respiration. d. Document that this child has bradycardia, and continue with the assessment.

ANS: B Sinus arrhythmia is commonly found in children and young adults. During the respiratory cycle, the heart rate varies, speeding up at the peak of inspiration and slowing to normal with expiration.

6. The component of the conduction system referred to as the pacemaker of the heart is the: A. Atrioventricular (AV) node. B. Sinoatrial (SA) node. C. Bundle of His. D. Bundle branches

ANS: B Specialized cells in the SA node near the superior vena cava initiate an electrical impulse. Because the SA node has an intrinsic rhythm, it is the "pacemaker."

27. In assessing for an S4 heart sound with a stethoscope, the nurse would listen with the: A. Bell at the base with the patient leaning forward. B. Bell at the apex with the patient in the left lateral position. C. Diaphragm in the aortic area with the patient sitting. D. Diaphragm in the pulmonic area with the patient supine

ANS: B The S4 is a ventricular filling sound. It occurs when atria contract late in diastole. It is heard immediately before S1. This is a very soft sound with a very low pitch. The nurse needs a good bell and must listen for it. It is heard best at the apex, with the person in the left lateral position.

11. In assessing a 70-year-old man, the nurse finds the following: blood pressure 140/100 mm Hg; heart rate 104 and slightly irregular; split S2. Which of these findings can be explained by expected hemodynamic changes related to age? A. Increase in resting heart rate B. Increase in systolic blood pressure C. Decrease in diastolic blood pressure D. Increase in diastolic blood pressure

ANS: B With aging, there is an increase in systolic blood pressure. No significant change in diastolic pressure occurs with age. No change in resting heart rate occurs with aging. Cardiac output at rest is not changed with aging.

A 22-year-old woman comes to the clinic because of severe sunburn and states, I was out in the sun for just a couple of minutes. The nurse begins a medication review with her, paying special attention to which medication class? a. Nonsteroidal antiinflammatory drugs for pain b. Tetracyclines for acne c. Proton pump inhibitors for heartburn d. Thyroid replacement hormone for hypothyroidism

ANS: B Drugs that may increase sunlight sensitivity and give a burn response include sulfonamides, thiazide diuretics, oral hypoglycemic agents, and tetracycline. DIF: Cognitive Level: Applying (Application) MSC: Client Needs: Health Promotion and Maintenance

The nurse keeps in mind that a thorough skin assessment is extremely important because the skin holds information about a persons: a. Support systems. b. Circulatory status. c. Socioeconomic status. d. Psychological wellness.

ANS: B The skin holds information about the bodys circulation, nutritional status, and signs of systemic diseases, as well as topical data on the integumentary system itself. DIF: Cognitive Level: Understanding (Comprehension) MSC: Client Needs: Safe and Effective Care Environment: Management of Care

A patient has been admitted to a hospital after the staff in the nursing home noticed a pressure ulcer in his sacral area. The nurse examines the pressure ulcer and determines that it is a stage II ulcer. Which of these findings are characteristic of a stage II pressure ulcer? Select all that apply. a. Intact skin appears red but is not broken. b. Partial thickness skin erosion is observed with a loss of epidermis or dermis. c. Ulcer extends into the subcutaneous tissue. d. Localized redness in light skin will blanch with fingertip pressure. e. Open blister areas have a red-pink wound bed. f. Patches of eschar cover parts of the wound.

ANS: B, E Stage I pressure ulcers have intact skin that appears red but is not broken, and localized redness in intact skin will blanche with fingertip pressure. Stage II pressure ulcers have partial thickness skin erosion with a loss of epidermis or also the dermis; open blisters have a red-pink wound bed. Stage III pressure ulcers are full thickness, extending into the subcutaneous tissue; subcutaneous fat may be seen but not muscle, bone, or tendon. Stage IV pressure ulcers involve all skin layers and extend into supporting tissue, exposing muscle, bone, and tendon. Slough (stringy matter attached to the wound bed) or eschar (black or brown necrotic tissue) may be present.

14. A 60-year-old male patient has been treated for pneumonia for the past 6 weeks. He is seen today in the clinic for an unexplained weight loss of 10 pounds over the last 6 weeks. The nurse knows that: a. Weight loss is probably the result of unhealthy eating habits. b. Chronic diseases such as hypertension cause weight loss. c. Unexplained weight loss often accompanies short-term illnesses. d. Weight loss is probably the result of a mental health dysfunction.

ANS: C An unexplained weight loss may be a sign of a short-term illness or a chronic illness such as endocrine disease, malignancy, depression, anorexia nervosa, or bulimia.

Which of these veins are responsible for most of the venous return in the arm? a. Deep b. Ulnar c. Subclavian d. Superficial

ANS: Superficial The superficial veins of the arms are in the subcutaneous tissue and are responsible for most of the venous return.

44. When considering the concepts related to blood pressure, the nurse knows that the concept of mean arterial pressure (MAP) is best described by which statement? a. MAP is the pressure of the arterial pulse. b. MAP reflects the stroke volume of the heart. c. MAP is the pressure forcing blood into the tissues, averaged over the cardiac cycle. d. MAP is an average of the systolic and diastolic blood pressures and reflects tissue perfusion.

ANS: C MAP is the pressure that forces blood into the tissues, averaged over the cardiac cycle. Stroke volume is reflected by the blood pressure. MAP is not an arithmetic average of systolic and diastolic pressures because diastole lasts longer; rather, it is a value closer to diastolic pressure plus one third of the pulse pressure.

A 35-year-old pregnant woman comes to the clinic for a monthly appointment. During the assessment, the nurse notices that she has a brown patch of hyperpigmentation on her face. The nurse continues the skin assessment aware that another finding may be: a. Keratoses. b. Xerosis. c. Chloasma. d. Acrochordons.

ANS: C In pregnancy, skin changes can include striae, linea nigra (a brownish-black line down the midline), chloasma (brown patches of hyperpigmentation), and vascular spiders. Keratoses are raised, thickened areas of pigmentation that look crusted, scaly, and warty. Xerosis is dry skin. Acrochordons, or skin tags, occur more often in the aging adult. DIF: Cognitive Level: Analyzing (Analysis) MSC: Client Needs: Physiologic Integrity: Physiologic Adaptation

While assessing a hospitalized, bedridden patient, the nurse notices that the patient has been incontinent of stool. The stool is loose and gray-tan in color. The nurse recognizes that this finding indicates which of the following? A) Occult blood B) Inflammation C) Absent bile pigment D) Ingestion of iron preparations

ANS: C The presence of gray, tan stool indicates absent bile pigment, which can occur with obstructive jaundice. Ingestion of iron preparations and presence of occult blood would turn the stools to a black color. Jelly-like mucus shreds mixed in the stool would indicate inflammation.

2. When measuring a patients weight, the nurse is aware of which of these guidelines? a. The patient is always weighed wearing only his or her undergarments. b. The type of scale does not matter, as long as the weights are similar from day to day. c. The patient may leave on his or her jacket and shoes as long as these are documented next to the weight. d. Attempts should be made to weigh the patient at approximately the same time of day, if a sequence of weights is necessary.

ANS: D A standardized balance scale is used to measure weight. The patient should remove his or her shoes and heavy outer clothing. If a sequence of repeated weights is necessary, then the nurse should attempt to weigh the patient at approximately the same time of day and with the same types of clothing worn each time.

42. The nurse is assessing children in a pediatric clinic. Which statement is true regarding the measurement of blood pressure in children? a. Blood pressure guidelines for children are based on age. b. Phase II Korotkoff sounds are the best indicator of systolic blood pressure in children. c. Using a Doppler device is recommended for accurate blood pressure measurements until adolescence. d. The disappearance of phase V Korotkoff sounds can be used for the diastolic reading in children.

ANS: D The disappearance of phase V Korotkoff sounds can be used for the diastolic reading in children, as well as in adults.

26. While measuring a patients blood pressure, the nurse recalls that certain factors, such as __________, help determine blood pressure. a. Pulse rate b. Pulse pressure c. Vascular output d. Peripheral vascular resistance

ANS: D The level of blood pressure is determined by five factors: cardiac output, peripheral vascular resistance, volume of circulating blood, viscosity, and elasticity of the vessel walls.

10. A patient is asked to indicate on a form how many times he eats a specific food. This would describe which of these methods for obtaining dietary information?

D) Food frequency questionnaire

A 15-year-old boy is seen in the clinic for complaints of "dull pain and pulling" in the scrotal area. On examination the nurse palpates a soft, irregular mass posterior to and above the testis on the left. This mass collapses when the patient is supine and refills when he is upright. This description is consistent with: A) epididymitis. B) spermatocele. C) testicular torsion. D) varicocele.

ANS: D A varicocele consists of dilated, tortuous varicose veins in the spermatic cord caused by incompetent valves within the vein. Symptoms include dull pain or constant pulling or dragging feeling, or the individual may be asymptomatic. When palpating the mass, the examiner will feel a soft, irregular mass posterior to and above the testis that collapses when the individual is supine and refills when the individual is upright. See Table 24-6 for more information and for descriptions of the other options.

The nurse is bathing an 80-year-old man and notices that his skin is wrinkled, thin, lax, and dry. This finding would be related to which factor in the older adult? a. Increased vascularity of the skin b. Increased numbers of sweat and sebaceous glands c. An increase in elastin and a decrease in subcutaneous fat d. An increased loss of elastin and a decrease in subcutaneous fat

ANS: D An accumulation of factors place the aging person at risk for skin disease and breakdown: the thinning of the skin, a decrease in vascularity and nutrients, the loss of protective cushioning of the subcutaneous layer, a lifetime of environmental trauma to skin, the social changes of aging, a increasingly sedentary lifestyle, and the chance of immobility.

A 65-year-old man with emphysema and bronchitis has come to the clinic for a follow-up appointment. On assessment, the nurse might expect to see which finding? a. Anasarca b. Scleroderma c. Pedal erythema d. Clubbing of the nails

ANS: D Clubbing of the nails occurs with congenital cyanotic heart disease and neoplastic and pulmonary diseases. The other responses are assessment findings not associated with pulmonary diseases.

A 22-year-old woman has been considering using oral contraceptives. As a part of her history, the nurse should ask: A) "Do you have a history of heart murmurs?" B) "Will you be in a monogamous relationship?" C) "Have you thought this choice through carefully?" D) "If you smoke, how many cigarettes do you smoke per day?"

ANS: D Oral contraceptives, together with cigarette smoking, increase the risk for cardiovascular side effects. If cigarettes are used, then the nurse should assess smoking history. The other questions are not appropriate.

A newborn infant has Down syndrome. During the skin assessment, the nurse notices a transient mottling in the trunk and extremities in response to the cool temperature in the examination room. The infant's mother also notices the mottling and asks what it is. The nurse knows that this mottling is called: a. Café au lait. b. Carotenemia. c. Acrocyanosis. d. Cutis marmorata.

ANS: D Persistent or pronounced cutis marmorata occurs with infants born with Down syndrome or those born prematurely and is a transient mottling in the trunk and extremities in response to cool room temperatures. A café au lait spot is a large round or oval patch of light-brown pigmentation. Carotenemia produces a yellow-orange color in light-skinned persons. Acrocyanosis is a bluish color around the lips, hands and fingernails, and feet and toenails.

The nurse is providing patient teaching about an erectile dysfunction drug. One of the drug's potential side effects is prolonged, painful erection of the penis without sexual stimulation, which is known as: A) orchitis. B) stricture. C) phimosis. D) priapism.

ANS: D Priapism is prolonged, painful erection of the penis without sexual desire. Orchitis is inflammation of the testes. Stricture is a narrowing of the opening of the urethral meatus. Phimosis is the inability to retract the foreskin

Which of these statements about the anal canal is true? The anal canal: A) is about 2 cm long in the adult. B) slants backward toward the sacrum. C) contains hair and sebaceous glands. D) is the outlet for the gastrointestinal tract.

ANS: D The anal canal is the outlet for the gastrointestinal tract, and it is about 3.8 cm long in the adult. It is lined with a modified skin that does not contain hair or sebaceous glands, and it slants forward toward the umbilicus.

During the assessment of an 18-month-old child, the mother expresses concern to the nurse about the infant's inability to toilet train. What would be the nurse's best response? A) "Some children are just more difficult to train, so I wouldn't worry about it yet." B) "Have you considered reading any of the books on toilet training? They can be very helpful." C) "This could mean there is a problem in your baby's development. We'll watch her closely for the next few months." D) "The nerves that will allow your baby to have control over the passing of stools are not developed until at least 18 to 24 months of age."

ANS: D The infant passes stools by reflex. Voluntary control of the external anal sphincter cannot occur until the nerves supplying the area have become fully myelinated, usually around 1 1/2 to 2 years of age. Toilet training usually starts after the age of 2.

When performing a genital assessment on a middle-aged man, the nurse notices multiple soft, moist, painless papules in the shape of cauliflower-like patches scattered across the shaft of the penis. These lesions are characteristic of: A) carcinoma. B) syphilitic chancres. C) genital herpes. D) genital warts.

ANS: D The lesions of genital warts are soft, pointed, moist, fleshy, painless papules that may be single or multiple in a cauliflower-like patch. They occur on the shaft of the penis, behind the corona, or around the anus, where they may grow into large grape-like clusters. See Table 24-4 for more information and for descriptions of the other options.

The nurse is assessing for clubbing of the fingernails and expects to find: a. Nail bases that are firm and slightly tender. b. Curved nails with a convex profile and ridges across the nails. c. Nail bases that feel spongy with an angle of the nail base of 150 degrees. d. Nail bases with an angle of 180 degrees or greater and nail bases that feel spongy.

ANS: D The normal nail is firm at its base and has an angle of 160 degrees. In clubbing, the angle straightens to 180 degrees or greater and the nail base feels spongy.

The nurse is discussing breast self-examination with a postmenopausal woman. The best time for postmenopausal women to perform breast self-examination is: a. On the same day every month. b. Daily, during the shower or bath. c. One week after her menstrual period. d. Every year with her annual gynecologic examination.

ANS: the same day every month. Postmenopausal women are no longer experiencing regular menstrual cycles but need to continue to perform breast self-examination on a monthly basis. Choosing the same day of the month is a helpful reminder to perform breast self-examination.

The nurse is performing a middle ear assessment on a 15-year-old patient who has had a history of chronic ear infections. When examining the right tympanic membrane, the nurse sees the presence of dense white patches. The tympanic membrane is otherwise unremarkable. It is pearly, with the light reflex at 5 o'clock and landmarks visible. The nurse should: a. Refer the patient for the possibility of a fungal infection. b. Know that these are scars caused from frequent ear infections. c. Consider that these findings may represent the presence of blood in the middle ear. d. Be concerned about the ability to hear because of this abnormality on the tympanic membrane.

B Dense white patches on the tympanic membrane are sequelae of repeated ear infections. They do not necessarily affect hearing.

During an oral examination of a 4-year-old Native-American child, the nurse notices that her uvula is partially split. Which of these statements is accurate? a.This condition is a cleft palate and is common in Native Americans. b.A bifid uvula may occur in some Native-American groups. c.This condition is due to an injury and should be reported to the authorities. d.A bifid uvula is palatinus, which frequently occurs in Native Americans.

B Bifid uvula, a condition in which the uvula is split either completely or partially, occurs in some Native-American groups.

The nurse is palpating the sinus areas. If the findings are normal, then the patient should report which sensation? a.No sensation b.Firm pressure c.Pain during palpation d.Pain sensation behind eyes

B The person should feel firm pressure but no pain. Sinus areas are tender to palpation in persons with chronic allergies or an acute infection (sinusitis).

1. The nurse is assessing an obese patient for signs of metabolic syndrome. This condition is diagnosed when three or more certain risk factors are present. Which of these assessment findings are risk factors for metabolic syndrome? Select all that apply.

B) Fasting plasma glucose level greater than or equal to 110 mg/dL C) Blood pressure reading of 140/90 mm Hg

30. In performing an assessment on a 49-year-old woman who has imbalanced nutrition as a result of dysphagia, which data would the nurse expect to find?

B) Inadequate nutrient food intake

8. The nurse is reviewing a patient's nutritional assessment. Which statement is true concerning the nutritional assessment?

B) It identifies patients who are at risk of malnutrition.

16. The nurse is reviewing the nutritional assessment of an 82-year-old patient. Which of these factors ismost likely to affect the nutritional status of an elderly person?

B) Living alone on a fixed income

1. The nurse suspects that a patient has appendicitis. Which of these procedures are appropriate for use when assessing for appendicitis or a perforated appendix? Select all that apply. a. Test for the Murphy sign b. Test for the Blumberg sign c. Test for shifting dullness d. Perform the iliopsoas muscle test e. Test for fluid wave

B) Test for Blumberg's sign. D) Perform iliopsoas muscle test. Testing for the Blumberg sign (rebound tenderness) and performing the iliopsoas muscle test should be used when assessing for appendicitis. The Murphy sign is used when assessing for an inflamed gallbladder or cholecystitis. Testing for a fluid wave and shifting dullness is performed when assessing for ascites.

28. A patient is suspected of having inflammation of the gallbladder, or cholecystitis. The nurse should conduct which of these techniques to assess for this condition? a. Obturator test b. Test for Murphy sign c. Assess for rebound tenderness d. Iliopsoas muscle test

B) Test for Murphy's sign Normally, palpating the liver causes no pain. In a person with inflammation of the gallbladder, or cholecystitis, pain occurs as the descending liver pushes the inflamed gallbladder onto the examining hand during inspiration (Murphy test). The person feels sharp pain and abruptly stops midway during inspiration.

23. The nurse is assessing the abdomen of an aging adult. Which of these statements regarding the aging adult and abdominal assessment is true? a. Abdominal tone is increased. b. Abdominal musculature is thinner. c. Abdominal rigidity with an acute abdominal condition is more common. d. The older adult with an acute abdominal condition complains more about pain than the younger person.

B) The abdominal musculature is thinner. In the older adult, the abdominal musculature is thinner and has less tone than that of the younger adult, and abdominal rigidity with an acute abdominal condition is less common in the aging person. The older adult with an acute abdominal condition often complains less about pain than the younger person.

12. The nurse is listening to bowel sounds. Which of these statements is true of bowel sounds? a. Are usually loud, high-pitched, rushing, and tinkling sounds. b. Are usually high-pitched, gurgling, and irregular sounds. c. Sound like two pieces of leather being rubbed together. d. Originate from the movement of air and fluid through the large intestine.

B) They are usually high-pitched, gurgling, irregular sounds. Bowel sounds are high-pitched, gurgling, and cascading sounds that irregularly occur from 5 to 30 times per minute. They originate from the movement of air and fluid through the small intestine.

39. The nurse is preparing to examine a patient who has been complaining of right lower quadrant pain. Which technique is correct during the assessment? The nurse should: a. Examine the tender area first. b. Examine the tender area last. c. Avoid palpating the tender area. d. Palpate the tender area first, and then auscultate for bowel sounds.

B) examine the tender area last. The nurse should save the examination of any identified tender areas until last. This method avoids pain and the resulting muscle rigidity that would obscure deep palpation later in the examination. Auscultation is performed before percussion and palpation because percussion and palpation can increase peristalsis, which would give a false interpretation of bowel sounds.

While gathering equipment after an injection, a nurse accidentally received a prick from an improperly capped needle. To interpret this sensation, which of these areas must be intact? a.Corticospinal tract, medulla, and basal ganglia b.Pyramidal tract, hypothalamus, and sensory cortex c.Lateral spinothalamic tract, thalamus, and sensory cortex d.Anterior spinothalamic tract, basal ganglia, and sensory cortex

C The spinothalamic tract contains sensory fibers that transmit the sensations of pain, temperature, and crude or light touch. Fibers carrying pain and temperature sensations ascend the lateral spinothalamic tract, whereas the sensations of crude touch form the anterior spinothalamic tract. At the thalamus, the fibers synapse with another sensory neuron, which carries the message to the sensory cortex for full interpretation. The other options are not correct.

A patient has been diagnosed with strep throat. The nurse is aware that without treatment, which complication may occur? a.Rubella b.Leukoplakia c.Rheumatic fever d.Scarlet fever

C Untreated strep throat may lead to rheumatic fever. When performing a health history, the patient should be asked whether his or her sore throat has been documented as streptococcal.

40. During a health history, the patient tells the nurse, "I have pain all the time in my stomach. It's worse two hours after I eat, but it gets better if I eat again!" The nurse suspects that the patient has which condition, based on these symptoms? a. Appendicitis b. Gastric ulcer c. Duodenal ulcer d. Cholecystitis

C) Duodenal ulcer Pain associated with duodenal ulcers occurs 2 to 3 hours after a meal; it may relieved by more food. Chronic pain associated with gastric ulcers usually occurs on an empty stomach. Severe, acute pain would occur with appendicitis and cholecystitis.

35. A 16-year-old girl is being seen at the clinic for gastrointestinal complaints and weight loss. The nurse determines that many of her complaints may be related to erratic eating patterns, eating predominantly fast foods, and high caffeine intake. In this situation, which is most appropriate when collecting current dietary intake information?

C) Have the patient complete a food diary for 3 days, including 2 weekdays and 1 weekend day.

29. A 50-year-old woman with elevated total cholesterol and triglyceride levels is visiting the clinic today to find out about her laboratory results. What would be important for the nurse to include in patient teaching in relation to these tests?

C) Information regarding a diet low in saturated fat

37. During an assessment the nurse notices that a patient's umbilicus is enlarged and everted. It is midline, and there is no change in skin color. The nurse recognizes that the patient may have which condition? a. Intra-abdominal bleeding b. Constipation c. Umbilical hernia d. Abdominal tumor

C) Umbilical hernia The umbilicus is normally midline and inverted with no signs of discoloration. With an umbilical hernia, the mass is enlarged and everted. The other responses are incorrect.

3. A patient is having difficulty in swallowing medications and food. The nurse would document that this patient has: a. Aphasia. b. Dysphasia. c. Dysphagia. d. Anorexia.

C) dysphagia. Dysphagia is a condition that occurs with disorders of the throat or esophagus and results in difficulty swallowing. Aphasia and dysphasia are speech disorders. Anorexia is a loss of appetite.

38. A 65-year-old man is brought to the emergency department after he was found dazed and incoherent, alone in his apartment. He has an enlarged liver and is moderately dehydrated. When evaluating his serum albumin level, the nurse must keep in mind that:

C) low serum albumin levels may be caused by reasons other than protein-calorie malnutrition.

9. While examining a patient, the nurse observes abdominal pulsations between the xiphoid and umbilicus. The nurse would suspect that these are: a. Pulsations of the renal arteries. b. Pulsations of the inferior vena cava. c. Normal abdominal aortic pulsations. d. Increased peristalsis from a bowel obstruction.

C) normal abdominal aortic pulsations. Normally, the pulsations from the aorta are observed beneath the skin in the epigastric area, particularly in thin persons who have good muscle wall relaxation.

26. The mother of an 8-year-old boy is concerned about the amount of weight her son has gained. To determine whether this is a problem, the nurse will measure:

C) skinfold thickness.

16. The nurse is performing percussion during an abdominal assessment. Percussion notes heard during the abdominal assessment may include: a. Flatness, resonance, and dullness. b. Resonance, dullness, and tympany. c. Tympany, hyperresonance, and dullness. d. Resonance, hyperresonance, and flatness.

C) tympany, hyperresonance, and dullness. Percussion notes normally heard during the abdominal assessment may include tympany, which should predominate because air in the intestines rises to the surface when the person is supine; hyperresonance, which may be present with gaseous distention; and dullness, which may be found over a distended bladder, adipose tissue, fluid, or a mass.

42. A 50-year-old patient has been brought to the emergency department after a housemate found that he could not get out of bed alone. He has lived in a group home for years, but for several months he has not participated in the activities and has stayed in his room. The nurse assesses for signs of under nutrition, and x-rays reveal that he has osteomalacia, which is a deficiency of:

C) vitamin D and calcium.

35. The nurse is assessing a patient for possible peptic ulcer disease and knows that which condition often causes this problem? a. Hypertension b. Streptococcal infections c. Recurrent constipation with frequent laxative use d. Frequent use of nonsteroidal antiinflammatory drugs

D) Frequent use of nonsteroidal anti-inflammatory drugs Peptic ulcer disease occurs with the frequent use of nonsteroidal antiinflammatory drugs, alcohol use, smoking, and Helicobacter pylori infection.

The nurse is conducting a class on priority setting for a group of new graduate nurses. Which is an example of a first-level priority problem? a. Patient with postoperative pain b. Newly diagnosed patient with diabetes who needs diabetic teaching c. Individual with a small laceration on the sole of the foot d. Individual with shortness of breath and respiratory distress

D First-level priority problems are those that are emergent, life-threatening, and immediate (e.g., establishing an airway, supporting breathing, maintaining circulation, monitoring abnormal vital signs) (see Table 1-1).

An assessment of a 23-year-old patient reveals the following: an auricle that is tender and reddish-blue in color with small vesicles. The nurse would need to know additional information that includes which of these? a. Any change in the ability to hear b. Any recent drainage from the ear c. Recent history of trauma to the ear d. Any prolonged exposure to extreme cold

D Frostbite causes reddish-blue discoloration and swelling of the auricle after exposure to extreme cold. Vesicles or bullae may develop, and the person feels pain and tenderness.

In performing an examination of a 3-year-old child with a suspected ear infection, the nurse would: a. Omit the otoscopic examination if the child has a fever. b. Pull the ear up and back before inserting the speculum. c. Ask the mother to leave the room while examining the child. d. Perform the otoscopic examination at the end of the assessment

D In addition to its place in the complete examination, eardrum assessment is mandatory for any infant or child requiring care for an illness or fever. For the infant or young child, the timing of the otoscopic examination is best toward the end of the complete examination.

A 32-year-old woman tells the nurse that she has noticed "very sudden, jerky movements" mainly in her hands and arms. She says, "They seem to come and go, primarily when I am trying to do something. I haven't noticed them when I'm sleeping." This description suggests: a.Tics. b.Athetosis. c.Myoclonus. d.Chorea.

D Chorea is characterized by sudden, rapid, jerky, purposeless movements that involve the limbs, trunk, or face. Chorea occurs at irregular intervals, and the movements are all accentuated by voluntary actions.

When examining an older adult, the nurse should use which technique? a.Avoid touching the patient too much. b.Attempt to perform the entire physical examination during one visit. c.Speak loudly and slowly because most aging adults have hearing deficits. d.Arrange the sequence of the examination to allow as few position changes as possible.

D When examining the older adult, arranging the sequence of the examination to allow as few position changes as possible is best. Physical touch is especially important with the older person because other senses may be diminished.

The mother of a 16-month-old toddler tells the nurse that her daughter has an earache. What would be an appropriate response? a."Maybe she is just teething." b."I will check her ear for an ear infection." c."Are you sure she is really having pain?" *d."Describe what she is doing to indicate she is having pain."*

D With a very young child, the parent is asked, "How do you know the child is in pain?" A young child pulling at his or her ears should alert parents to the child's ear pain. Statements about teething and questioning whether the child is really having pain do not explore the symptoms, which should be done before a physical examination.

34. The nurse in a family practice clinic is reviewing the patients scheduled for appointments. Which of these statements is true regarding routine laboratory testing in the following individuals?

D) Laboratory tests for iron and lead levels should be assessed at 9 to 12 months.

4. The nurse suspects that a patient has a distended bladder. How should the nurse assess for this condition? a. Percuss and palpate in the lumbar region. b. Inspect and palpate in the epigastric region. c. Auscultate and percuss in the inguinal region. d. Percuss and palpate the midline area above the suprapubic bone.

D) Percuss and palpate the midline area above the suprapubic bone. Dull percussion sounds would be elicited over a distended bladder, and the hypogastric area would seem firm to palpation.

7. A patient's abdomen is bulging and stretched in appearance. The nurse should describe this finding as: a. Obese. b. Herniated. c. Scaphoid. d. Protuberant.

D) protuberant. A protuberant abdomen is rounded, bulging, and stretched (see Figure 21-7). A scaphoid abdomen caves inward.

2. When assessing a patient's nutritional status, the nurse recalls that the best definition of optimal nutritional status is:

D) sufficient nutrients to provide for daily body requirements and for increased metabolic demands.

When the nurse is evaluating the reliability of a patient's responses, which of these statements would be correct? The patient: a. Has a history of drug abuse and therefore is not reliable. b. Provided consistent information and therefore is reliable. c. Smiled throughout interview and therefore is assumed reliable. d. Would not answer questions concerning stress and therefore is not reliable.

*b. Provided consistent information and therefore is reliable.* A reliable person always gives the same answers, even when questions are rephrased or are repeated later in the interview. The other statements are not correct.

When performing an otoscopic examination of a 5-year-old child with a history of chronic ear infections, the nurse sees that his right tympanic membrane is amber-yellow in color and that air bubbles are visible behind the tympanic membrane. The child reports occasional hearing loss and a popping sound with swallowing. The preliminary analysis based on this information is that the child: a. Most likely has serous otitis media. b. Has an acute purulent otitis media. c. Has evidence of a resolving cholesteatoma. d. Is experiencing the early stages of perforation.

A An amber-yellow color to the tympanic membrane suggests serum or pus in the middle ear. Air or fluid or bubbles behind the tympanic membrane are often visible. The patient may have feelings of fullness, transient hearing loss, and a popping sound with swallowing. These findings most likely suggest that the child has serous otitis media. The other responses are not correct.

Barriers to incorporating EBP include: a. Nurses lack of research skills in evaluating the quality of research studies. b. Lack of significant research studies. c. Insufficient clinical skills of nurses. d. Inadequate physical assessment skills.

A As individuals, nurses lack research skills in evaluating the quality of research studies, are isolated from other colleagues who are knowledgeable in research, and often lack the time to visit the library to read research. The other responses are not considered barriers.

While discussing the history of a 6-month-old infant, the mother tells the nurse that she took a significant amount of aspirin while she was pregnant. What question would the nurse want to include in the history? a. "Does your baby seem to startle with loud noises?" b. "Has your baby had any surgeries on her ears?" c. "Have you noticed any drainage from her ears?" d. "How many ear infections has your baby had since birth?

A Children at risk for a hearing deficit include those exposed in utero to a variety of conditions, such as maternal rubella or to maternal ototoxic drugs.

A newly admitted patient is in acute pain, has not been sleeping well lately, and is having difficulty breathing. How should the nurse prioritize these problems? a. Breathing, pain, and sleep b. Breathing, sleep, and pain c. Sleep, breathing, and pain d. Sleep, pain, and breathing

A First-level priority problems are immediate priorities, remembering the ABCs (airway, breathing, and circulation), followed by second-level problems, and then third-level problems.

The nurse is preparing to do an otoscopic examination on a 2-year-old child. Which one of these reflects the correct procedure? a. Pulling the pinna down b. Pulling the pinna up and back c. Slightly tilting the child's head toward the examiner d. Instructing the child to touch his chin to his chest

A For an otoscopic examination on an infant or on a child under 3 years of age, the pinna is pulled down. The other responses are not part of the correct procedure.

A 31-year-old patient tells the nurse that he has noticed a progressive loss in his hearing. He says that it does seem to help when people speak louder or if he turns up the volume of a television or radio. The most likely cause of his hearing loss is: a. Otosclerosis. b. Presbycusis. c. Trauma to the bones. d. Frequent ear infections.

A Otosclerosis is a common cause of conductive hearing loss in young adults between the ages of 20 and 40 years. Presbycusis is a type of hearing loss that occurs with aging. Trauma and frequent ear infections are not a likely cause of his hearing loss

The nurse needs to pull the portion of the ear that consists of movable cartilage and skin down and back when administering eardrops. This portion of the ear is called the: a. Auricle. b. Concha. c. Outer meatus. d. Mastoid process.

A The external ear is called the auricle or pinna and consists of movable cartilage and skin.

The nurse is assessing a patient who may have hearing loss. Which of these statements is true concerning air conduction? a. Air conduction is the normal pathway for hearing. b. Vibrations of the bones in the skull cause air conduction. c. Amplitude of sound determines the pitch that is heard. d. Loss of air conduction is called a conductive hearing loss.

A The normal pathway of hearing is air conduction, which starts when sound waves produce vibrations on the tympanic membrane. Conductive hearing loss results from a mechanical dysfunction of the external or middle ear. The other statements are not true concerning air conduction.

The patients record, laboratory studies, objective data, and subjective data combine to form the: a. Data base. b. Admitting data. c. Financial statement. d. Discharge summary.

A Together with the patients record and laboratory studies, the objective and subjective data form the data base. The other items are not part of the patients record, laboratory studies, or data.

The nurse assesses the hearing of a 7-month-old by clapping hands. What is the expected response? The infant: a. Turns his or her head to localize the sound. b. Shows no obvious response to the noise. c. Shows a startle and acoustic blink reflex. d. Stops any movement, and appears to listen for the sound.

A With a loud sudden noise, the nurse should notice the infant turning his or her head to localize the sound and to respond to his or her own name. A startle reflex and acoustic blink reflex is expected in newborns; at age 3 to 4 months, the infant stops any movement and appears to listen.

During an examination, the nurse notices that the patient stumbles a little while walking, and, when she sits down, she holds on to the sides of the chair. The patient states, "It feels like the room is spinning!" The nurse notices that the patient is experiencing: a. Objective vertigo. b. Subjective vertigo. c. Tinnitus. d. Dizziness.

A With objective vertigo, the patient feels like the room spins; with subjective vertigo, the person feels like he or she is spinning. Tinnitus is a sound that comes from within a person; it can be a ringing, crackling, or buzzing sound. It accompanies some hearing or ear disorders. Dizziness is not the same as true vertigo; the person who is dizzy may feel unsteady and lightheaded.

A 21-year-old patient has a head injury resulting from trauma and is unconscious. There are no other injuries. During the assessment what would the nurse expect to find when testing the patient's deep tendon reflexes? a.Reflexes will be normal. b.Reflexes cannot be elicited. c.All reflexes will be diminished but present. d.Some reflexes will be present, depending on the area of injury.

A A reflex is a defense mechanism of the nervous system. It operates below the level of conscious control and permits a quick reaction to potentially painful or damaging situations

To assess the head control of a 4-month-old infant, the nurse lifts up the infant in a prone position while supporting his chest. The nurse looks for what normal response? The infant: a.Raises the head, and arches the back. b.Extends the arms, and drops down the head. c.Flexes the knees and elbows with the back straight. d.Holds the head at 45 degrees, and keeps the back straight.

A At 3 months of age, the infant raises the head and arches the back as if in a swan dive. This response is the Landau reflex, which persists until 1 years of age. The other responses are incorrec

The nurse is reviewing percussion techniques with a newly graduated nurse. Which technique, if used by the new nurse, indicates that more review is needed? a.Percussing once over each area b.Quickly lifting the striking finger after each stroke c.Striking with the fingertip, not the finger pad d.Using the wrist to make the strikes, not the arm

A For percussion, the nurse should percuss two times over each location. The striking finger should be quickly lifted because a resting finger damps off vibrations. The tip of the striking finger should make contact, not the pad of the finger. The wrist must be relaxed and is used to make the strikes, not the arm.

The nurse is obtaining a history from a 30-year-old male patient and is concerned about health promotion activities. Which of these questions would be appropriate to use to assess health promotion activities for this patient? *a."Do you perform testicular self-examinations?"* b."Have you ever noticed any pain in your testicles?" c."Have you had any problems with passing urine?" d."Do you have any history of sexually transmitted diseases?"

A Health promotion for a man would include the performance of testicular self-examinations. The other questions are asking about possible disease or illness issues.

In a person with an upper motor neuron lesion such as a cerebrovascular accident, which of these physical assessment findings should the nurse expect? a.Hyperreflexia b.Fasciculations c.Loss of muscle tone and flaccidity d.Atrophy and wasting of the muscles

A Hyperreflexia, diminished or absent superficial reflexes, and increased muscle tone or spasticity can be expected with upper motor neuron lesions. The other options reflect a lesion of lower motor neurons.

To test for gross motor skill and coordination of a 6-year-old child, which of these techniques would be appropriate? Ask the child to: a.Hop on one foot. b.Stand on his head. c.Touch his finger to his nose. d.Make "funny" faces at the nurse.

A Normally, a child can hop on one foot and can balance on one foot for approximately 5 seconds by 4 years of age and can balance on one foot for 8 to 10 seconds at 5 years of age. Children enjoy performing these tests. Failure to hop after 5 years of age indicates incoordination of gross motor skills. Asking the child to touch his or her finger to the nose checks fine motor coordination; and asking the child to make funny faces tests CN VII. Asking a child to stand on his or her head is not appropriate.

The nurse is assessing a patient with a history of intravenous drug abuse. In assessing his mouth, the nurse notices a dark red confluent macule on the hard palate. This could be an early sign of: a.Acquired immunodeficiency syndrome (AIDS). b.Measles. c.Leukemia. d.Carcinoma.

A Oral Kaposi's sarcoma is a bruiselike, dark red or violet, confluent macule that usually occurs on the hard palate. It may appear on the soft palate or gingival margin. Oral lesions may be among the earliest lesions to develop with AIDS.

Which of these techniques uses the sense of touch to assess texture, temperature, moisture, and swelling when the nurse is assessing a patient? a.Palpation b.Inspection c.Percussion d.Auscultation

A Palpation uses the sense of touch to assess the patient for these factors. Inspection involves vision; percussion assesses through the use of palpable vibrations and audible sounds; and auscultation uses the sense of hearing.

When percussing over the liver of a patient, the nurse notices a dull sound. The nurse should: a.Consider this a normal finding. b.Palpate this area for an underlying mass. c.Reposition the hands, and attempt to percuss in this area again. d.Consider this finding as abnormal, and refer the patient for additional treatment.

A Percussion over relatively dense organs, such as the liver or spleen, will produce a dull sound. The other responses are not correct.

While obtaining a health history of a 3-month-old infant from the mother, the nurse asks about the infant's ability to suck and grasp the mother's finger. What is the nurse assessing? a.Reflexes b.Intelligence c.CNs d.Cerebral cortex function

A Questions regarding reflexes include such questions as, What have you noticed about the infants behavior, Are the infants sucking and swallowing seem coordinated, and Does the infant grasp your finger? The other responses are incorrect.

During the assessment of an 80-year-old patient, the nurse notices that his hands show tremors when he reaches for something and his head is always nodding. No associated rigidity is observed with movement. Which of these statements is most accurate? a.These findings are normal, resulting from aging. b.These findings could be related to hyperthyroidism. c.These findings are the result of Parkinson disease. d.This patient should be evaluated for a cerebellar lesion.

A Senile tremors occasionally occur. These benign tremors include an intention tremor of the hands, head nodding (as if saying yes or no), and tongue protrusion. Tremors associated with Parkinson disease include rigidity, slowness, and a weakness of voluntary movement. The other responses are incorrect

The nurse is testing the deep tendon reflexes of a 30-year-old woman who is in the clinic for an annual physical examination. When striking the Achilles heel and quadriceps muscle, the nurse is unable to elicit a reflex. The nurse's next response should be to: a.Ask the patient to lock her fingers and pull. b.Complete the examination, and then test these reflexes again. c.Refer the patient to a specialist for further testing. d.Document these reflexes as 0 on a scale of 0 to 4+.

A Sometimes the reflex response fails to appear. Documenting the reflexes as absent is inappropriate this soon in the examination. The nurse should try to further encourage relaxation, varying the persons position or increasing the strength of the blow. Reinforcement is another technique to relax the muscles and enhance the response. The person should be asked to perform an isometric exercise in a muscle group somewhat away from the one being tested. For example, to enhance a patellar reflex, the person should be asked to lock the fingers together and pull.

The nurse knows that determining whether a person is oriented to his or her surroundings will test the functioning of which structure(s)? a.Cerebrum b.Cerebellum c.CNs d.Medulla oblongata

A The cerebral cortex is responsible for thought, memory, reasoning, sensation, and voluntary movement. The other structures are not responsible for a persons level of consciousness.

When preparing to perform a physical examination on an infant, the nurse should: a.Have the parent remove all clothing except the diaper on a boy. b.Instruct the parent to feed the infant immediately before the examination. c.Encourage the infant to suck on a pacifier during the abdominal examination. d.Ask the parent to leave the room briefly when assessing the infant's vital signs.

A The parent should always be present to increase the child's feeling of security and to understand normal growth and development. The timing of the examination should be 1 to 2 hours after feeding when the baby is neither too drowsy nor too hungry. Infants do not object to being nude; clothing should be removed, but a diaper should be left on a boy.

During an assessment of a 62-year-old man, the nurse notices the patient has a stooped posture, shuffling walk with short steps, flat facial expression, and pill-rolling finger movements. These findings would be consistent with: a.Parkinsonism. b.Cerebral palsy. c.Cerebellar ataxia. d.Muscular dystrophy.

A The stooped posture, shuffling walk, short steps, flat facial expression, and pill-rolling finger movements are all found in parkinsonism.

Which of these tests would the nurse use to check the motor coordination of an 11-month-old infant? a.Denver II b.Stereognosis c.Deep tendon reflexes d.Rapid alternating movements

A To screen gross and fine motor coordination, the nurse should use the DENVER 11 with its age specific developmental milestones. Stereognosis tests a persons ability to recognize objects by feeling them and is not appropriate for an 11-month-old infant. Testing the deep tendon reflexes is not appropriate for checking motor coordination. Testing rapid alternating movements is appropriate for testing coordination in adults.

During the taking of the health history, a patient tells the nurse that "it feels like the room is spinning around me." The nurse would document this finding as: a.Vertigo. b.Syncope. c.Dizziness. d.Seizure activity.

A True vertigo is rotational spinning caused by a neurologic dysfunction or a problem in the vestibular apparatus or the vestibular nuclei in the brainstem. Syncope is a sudden loss of strength or a temporary loss of consciousness. Dizziness is a lightheaded, swimming sensation. Seizure activity is characterized by altered or loss of consciousness, involuntary muscle movements, and sensory disturbances.

A 6-month-old infant has been brought to the well-child clinic for a check-up. She is currently sleeping. What should the nurse do first when beginning the examination? a.Auscultate the lungs and heart while the infant is still sleeping. b.Examine the infant's hips, because this procedure is uncomfortable. c.Begin with the assessment of the eye, and continue with the remainder of the examination in a head-to-toe approach. d.Wake the infant before beginning any portion of the examination to obtain the most accurate assessment of body systems.

A When the infant is quiet or sleeping is an ideal time to assess the cardiac, respiratory, and abdominal systems. Assessment of the eye, ear, nose, and throat are invasive procedures that should be performed at the end of the examination.

The nurse is performing an assessment on a 21-year-old patient and notices that his nasal mucosa appears pale, gray, and swollen. What would be the most appropriate question to ask the patient? a."Are you aware of having any allergies?" b."Do you have an elevated temperature?" c."Have you had any symptoms of a cold?" d."Have you been having frequent nosebleeds?"

A With chronic allergies, the mucosa looks swollen, boggy, pale, and gray. Elevated body temperature, colds, and nosebleeds do not cause these mucosal changes.

25. The nurse is evaluating patients for obesity-related diseases by calculating the waist-to-hip ratios. Which one of these patients would be at increased risk?

A) 29-year-old woman whose waist is 33 inches and whose hips are 36 inches

The nurse is preparing to palpate the thorax and abdomen of a patient. Which of these statements describes the correct technique for this procedure? Select all that apply. a.Warm the hands first before touching the patient. b.For deep palpation, use one long continuous palpation when assessing the liver. c.Start with light palpation to detect surface characteristics. d.Use the fingertips to examine skin texture, swelling, pulsation, and presence of lumps. e.Identify any tender areas, and palpate them last. f.Use the palms of the hands to assess temperature of the skin.

A, C, D, E The hands should always be warmed before beginning palpation. Intermittent pressure rather than one long continuous palpation is used; any tender areas are identified and palpated last. Fingertips are used to examine skin texture, swelling, pulsation, and the presence of lumps. The dorsa (backs) of the hands are used to assess skin temperature because the skin on the dorsa is thinner than on the palms.

The nurse is assessing a patient's headache pain. Which questions reflect one or more of the critical characteristics of symptoms that should be assessed? Select all that apply. a."Where is the headache pain?" b."Did you have these headaches as a child?" c."On a scale of 1 to 10, how bad is the pain?" d."How often do the headaches occur?" e."What makes the headaches feel better?" f."Do you have any family history of headaches?"

A, C, D, E The mnemonic PQRSTU may help the nurse remember to address the critical characteristics that need to be assessed: (1) P: provocative or palliative; (2) Q: quality or quantity; (3) R: region or radiation; (4) S: severity scale; (5) T: timing; and (6) U: understand the patient's perception. Asking, "Where is the pain?" reflects "region." Asking the patient to rate the pain on a 1 to 10 scale reflects "severity." Asking "How often..." reflects "timing." Asking what makes the pain better reflects "provocative." The other options reflect health history and family history.

During an examination, the nurse finds that a patient has excessive dryness of the skin. The best term to describe this condition is: a. Xerosis. b. Pruritus. c. Alopecia. d. Seborrhea.

ANS: A Xerosis is the term used to describe skin that is excessively dry. Pruritus refers to itching, alopecia refers to hair loss, and seborrhea refers to oily skin.

The nurse is preparing to teach a woman about breast self-examination (BSE). Which statement by the nurse is correct? a. "BSE is more important than ever for you because you have never had any children." b. "BSE is so important because one out of nine women will develop breast cancer in her lifetime." c. "BSE on a monthly basis will help you become familiar with your own breasts and feel their normal variations." d. "BSE will save your life because you are likely to find a cancerous lump between mammograms."

ANS: "BSE on a monthly basis will help you feel familiar with your own breasts and their normal variations." The nurse should stress that a regular monthly self-examination will familiarize her with her own breasts and their normal variations. This is a positive step that will reassure her of her healthy state. While teaching, the nurse should focus on the positive aspects of breast self-examination and should avoid citing frightening mortality statistics about breast cancer. This may generate excessive fear and denial that actually obstructs a woman's self-care action.

A 43-year-old woman is at the clinic for a routine examination. She reports that she has had a breast lump in her right breast for years. Recently, it has begun to change in consistency and is becoming harder. She reports that 5 years ago her physician evaluated the lump and determined that it "was nothing to worry about." The examination validates the presence of a mass in the right upper outer quadrant at 1 o'clock, approximately 5 cm from the nipple. It is firm, mobile, nontender, with borders that are not well defined. The nurse's recommendation to her is: a. "Because of the change in consistency of the lump, it should be further evaluated by a physician." b. "The changes could be related to your menstrual cycles. Keep track of the changes in the mass each month." c. "The lump is probably nothing to worry about because it has been present for years and was determined to be noncancerous 5 years ago." d. "Because you are experiencing no pain and the size has not changed, you should continue to monitor the lump and return to the clinic in 3 months."

ANS: "Because of the change in consistency of the lump, it should be further evaluated by a physician." A lump that has been present for years and is not exhibiting changes may not be serious but still should be explored. Any recent change or new lump should be evaluated. The other responses are not correct.

A 70-year-old patient is scheduled for open-heart surgery. The surgeon plans to use the great saphenous vein for the coronary bypass grafts. The patient asks, "What happens to my circulation when the veins are removed?" The nurse should reply: a. "Venous insufficiency is a common problem after this type of surgery." b. "Oh, you have lots of veins—you won't even notice that it has been removed." c. "You will probably experience decreased circulation after the vein is removed." d. "This vein can be removed without harming your circulation because the deeper veins in your leg are in good condition."

ANS: "Because the deeper veins in your leg are in good condition, this vein can be removed without harming your circulation." As long as the femoral and popliteal veins remain intact, the superficial veins can be excised without harming the circulation. The other responses are not correct.

A 55-year-old postmenopausal woman is being seen in the clinic for a yearly examination. She is concerned about changes in her breasts that she has noticed over the past 5 years. She states that her breasts have decreased in size and that the elasticity has changed so that her breasts seem "flat and flabby." The nurse's best reply would be: a. "This change occurs most often because of long-term use of bras that do not provide enough support to the breast tissues." b. "This is a normal change that occurs as women get older and is due to the increased levels of progesterone during the aging process." c. "Decreases in hormones after menopause causes atrophy of the glandular tissue in the breast and is a normal process of aging." d. "Postural changes in the spine make it appear that your breasts have changed in shape. Exercises to strengthen the muscles of the upper back and chest wall will help prevent the changes in elasticity and size."

ANS: "Decreases in hormones after menopause causes atrophy of the glandular tissue in the breast. This is a normal process of aging." The hormonal changes of menopause cause the breast glandular tissue to atrophy, making the breasts more pendulous, flattened, and sagging.

The nurse is describing a weak, thready pulse on the documentation flow sheet. Which statement is correct? a. "Is easily palpable; pounds under the fingertips." b. "Has greater than normal force, then suddenly collapses." c. "Is hard to palpate, may fade in and out, and is easily obliterated by pressure." d. "Rhythm is regular, but force varies with alternating beats of large and small amplitude."

ANS: "Hard to palpate, may fade in and out, easily obliterated by pressure." A weak, thready pulse is hard to palpate, may fade in and out, and is easily obliterated by pressure. It is associated with decreased cardiac output and peripheral arterial disease.

The nurse is teaching a review class on the lymphatic system. A participant shows correct understanding of the material with which statement? a. "Lymph flow is propelled by the contraction of the heart." b. "The flow of lymph is slow, compared with that of the blood." c. "One of the functions of the lymph is to absorb lipids from the biliary tract." d. "Lymph vessels have no valves; therefore, lymph fluid flows freely from the tissue spaces into the bloodstream."

ANS: "The flow of lymph is slow compared with that of the blood." The flow of lymph is slow compared with that of the blood. Lymph flow is not propelled by the heart, but rather by contracting skeletal muscles, pressure changes secondary to breathing, and by contraction of the vessel walls. Lymph does not absorb lipids from the biliary tract. The vessels do have valves, so flow is one way from the tissue spaces to the bloodstream.

A woman in her 26th week of pregnancy states that she is "not really short of breath" but feels that she is aware of her breathing and the need to breathe. What is the nurse's best reply? a. "The diaphragm becomes fixed during pregnancy, making it difficult to take in a deep breath." b. "The increase in estrogen levels during pregnancy often causes a decrease in the diameter of the rib cage and makes it difficult to breathe." c. "What you are experiencing is normal. Some women may interpret this as shortness of breath, but it is a normal finding and nothing is wrong." d. "This increased awareness of the need to breathe is normal as the fetus grows because of the increased oxygen demand on the mother's body, which results in an increased respiratory rate."

ANS: "What you are experiencing is normal. Some women may interpret this as shortness of breath, but it is a normal finding and nothing is wrong." During pregnancy, the woman may develop an increased awareness of the need to breathe. Some women may interpret this as dyspnea, even though structurally nothing is wrong. Estrogen increases relax the chest cage ligaments, causing an increase in transverse diameter. The growing fetus does increase the oxygen demand on the mother's body, but this is met easily by the increasing tidal volume (deeper breathing). Little change occurs in the respiratory rate.

A 14-year-old boy who has been diagnosed with Osgood-Schlatter disease reports painful swelling just below the knee for the past 5 months. Which response by the nurse is appropriate? A) "If these symptoms persist, you may need arthroscopic surgery." B) "You are experiencing degeneration of your knee, which may not resolve." C) "Your disease is due to repeated stress on the patellar tendon. It is usually self-limited, and your symptoms should resolve with rest." D) "Increasing your activity and performing knee-strengthening exercises will help to decrease the inflammation and maintain mobility in the knee."

ANS: "Your disease is due to repeated stress on the patellar tendon. It is usually self-limited, and your symptoms should resolve with rest." Osgood-Schlatter disease is painful swelling of the tibial tubercle just below the knee. It is most likely due to repeated stress on the patellar tendon. It is usually self-limited, occurring during rapid growth and most often in males. The symptoms resolve with rest. The other responses are not appropriate.

The nurse is interviewing a patient who has a hearing impairment. What techniques would be most beneficial in communicating with this patient? 1.Assess the communication method he prefers. 2.Avoid using facial and hand gestures because most hearing-impaired people find this degrading. 3.Request a sign language interpreter before meeting with him to help facilitate the communication. 4.Speak loudly and with exaggerated facial movement when talking with him because this helps with lip reading.

ANS: 1 Ask the deaf person the preferred way to communicate—by signing, lip reading, or writing. If the person prefers lip reading, be sure to face him or her squarely and have good lighting on your face. Do not exaggerate your lip movements because this distorts your words. Similarly, shouting distorts the reception of a hearing aid the person may wear. Speak slowly and supplement your voice with appropriate hand gestures or pantomime.

During an examination of a 3-year-old child, the nurse notes a bruit over the left temporal area. What should the nurse do? 1. Continue the examination because this is a normal finding for this age. 2. Check for the bruit again in 1 hour. 3. Notify the parents that a bruit has been detected in their child. 4. Stop the examination and notify the physician.

ANS: 1 Bruits are common in the skull in children under 4 or 5 years of age or in children with anemia. They are systolic or continuous and are heard over the temporal area.

The nurse is performing an assessment on a 7-year-old child who has symptoms of chronic watery eyes, sneezing, and clear nasal drainage. The nurse notes the presence of a transverse line across the bridge of the nose, dark blue shadows below the eyes, and a double crease on the lower eyelids. These findings are characteristic of: 1. allergies. 2. a sinus infection. 3. nasal congestion. 4. an upper respiratory infection.

ANS: 1 Chronic allergies often develop chronic facial characteristics. These include blue shadows below the eyes, a double or single crease on the lower eyelids, open-mouth breathing, and a transverse line on the nose.

During a well-baby checkup, a mother is concerned because her 2-month- old infant cannot hold her head up when she is pulled to a sitting position. Which response by the nurse is appropriate? 1. "Head control is usually achieved by 4 months of age." 2. "You shouldn't be trying to pull your baby up like that until she is older." 3. "This is a concern because head control should be achieved by this time." 4. "This is a concern because it indicates possible nerve damage to the neck muscles."

ANS: 1 Head control is achieved by 4 months, when the baby can hold the head erect and steady when pulled to a vertical position.

The nurse is performing a health interview on a patient, named Salil, who has a language barrier, and no interpreter is available. Which of the following is the best example of an appropriate question for the nurse to ask in this situation? 1. "Does Salil take medicine?" 2. "Do you sterilize the bottles?" 3. "Do you have nausea and vomiting?" 4. "He has been taking his medicine, hasn't he?"

ANS: 1 In a situation where there is a language barrier and no interpreter available, use simple words avoiding medical jargon. Avoid using contractions and pronouns. Use nouns repeatedly and discuss one topic at a time.

Because the physical environment in which an interview takes place is an important consideration for the success of an interview, the interviewer should: 1.reduce noise by turning off televisions and radios. 2.place the distance between the interviewer and the patient about 2 feet or closer. 3.provide a dim light that makes a room cozier and will help the patient relax. 4.arrange seating across a desk or table to allow the patient some personal space.

ANS: 1 Reduce noise. Multiple stimuli are confusing. Turn off the television, radio, and any unnecessary equipment.

As the nurse enters a patient's room, the nurse finds her crying. The patient states that she has just found out that the lump in her breast is cancer and says, "I'm so afraid of, um, you know." The nurse's most therapeutic response would be to say, in a gentle manner: 1."You're afraid you might lose your breast?" 2."No, I'm not sure what you are talking about." 3."I'll wait here until you get yourself under control and then we can talk." 4."I can see that you are very upset. Perhaps we should discuss this later."

ANS: 1 Reflection echoes the patient's words, repeating part of what the person just said. Reflection also can help express the feeling behind a person's words.

When taking a history from a newly admitted patient, the nurse notices that he is pausing often and looking at the nurse expectantly. What would be the nurse's best response to this behavior? 1.Be silent and allow him to continue when he is ready. 2.Smile at him and say, "Don't worry about all of this. I'm sure we can find out why you're having these pains." 3.Lean back in the chair and ask, "Mr. J., you are looking at me kind of funny; there isn't anything wrong, is there?" 4.Stand up and say, "I can see that this interview is uncomfortable for you. We can continue it another time."

ANS: 1 Silent attentiveness communicates that the person has time to think, to organize what he or she wishes to say without interruption from you. This "thinking silence" is the one health professionals interrupt most often.

During a follow-up visit, the nurse discovers that a patient has not been taking his insulin on a regular basis. The nurse asks, "Why haven't you taken your insulin?" Which of the following is an appropriate evaluation of this question? 1.It may put the patient on the defensive. 2.It is an innocent search for information. 3.It would have been better to discuss this with his wife. 4.It is the best way to discover the reasons for his behavior.

ANS: 1 The adult's use of "why" questions usually implies blame and condemnation and puts the person on the defensive.

The nurse suspects that a patient has hyperthyroidism and laboratory data indicate that the patient's thyroxine and tri-iodothyronine hormone levels are elevated. Which of the following findings would the nurse most likely find on examination? 1. Tachycardia 2. Constipation 3. Rapid dyspnea 4. Atrophied nodular thyroid

ANS: 1 Thyroxine and tri-iodothyronine are thyroid hormones that stimulate the rate of cellular metabolism, thus resulting in tachycardia. With an enlarged thyroid as in hyperthyroidism, you might expect to find diffuse enlargement (goiter) or a nodular lump.

he nurse has just started an assessment of a newborn child of a Vietnamese mother. Considering the mother's cultural background, which statement is true regarding this examination? The mother: 1. will be offended if the infant's fontanels are examined. 2. will be offended if the infant's diaper area is touched during the examination. 3. would prefer to have the results of the examination communicated directly to her husband. 4. would prefer to receive written material about growth and development rather than a verbal explanation.

ANS: 1 Touching children may also have associated meaning cross-culturally. Many Asians believe that one's strength resides in the head and touching the head is considered disrespectful. Thus, palpating the fontanel of an infant from Southeast Asian descent should be approached with sensitivity.

During an examination, the nurse knows that the best way to palpate the lymph nodes in the neck is described by which statement? 1. Using gentle pressure, palpate with both hands to compare the two sides. 2. Using strong pressure, palpate with both hands to compare the two sides. 3. Gently pinch each node between one's thumb and forefinger and move down the neck muscle. 4. Using the index and middle fingers, gently palpate by applying pressure in a rotating pattern.

ANS: 1 Use gentle pressure because strong pressure could push the nodes into the neck muscles. It is usually most efficient to palpate with both hands, comparing the two sides symmetrically.

A female patient does not speak English well, and the nurse needs to choose an interpreter. Which of the following would be the most appropriate choice? 1. A trained interpreter 2. A male family member 3. A female family member 4. A volunteer college student from the foreign language studies department.

ANS: 1 Whenever possible, use a trained interpreter, preferably one who knows medical terminology. In general, an older, more mature interpreter is preferred to a younger, less experienced one, and the same sex is preferred when possible.

When examining children affected with Down syndrome (trisomy 21), the nurse looks for the possible presence of: 1. ear dysplasia. 2. a long, thin neck. 3. a protruding thin tongue. 4. a narrow and raised nasal bridge.

ANS: 1 With the chromosomal aberration trisomy 21, also known as Down syndrome, head and face characteristics may include upslanting eyes with inner epicanthal folds, a flat nasal bridge, a small broad flat nose, a protruding thick tongue, ear dysplasia, a short broad neck with webbing, and small hands with a single palmar crease.

A patient comes to the clinic complaining of neck and shoulder pain and is unable to turn her head. The nurse suspects damage to CN _____ and proceeds with the examination by: 1. XI; palpating the anterior and posterior triangles. 2. XI; asking the patient to shrug her shoulders against resistance. 3. XII; percussing the sternomastoid and submandibular neck muscles. 4. XII; assessing for a positive Romberg's sign.

ANS: 2 The major neck muscles are the sternomastoid and the trapezius. They are innervated by CN XI, the spinal accessory. The innervated muscles assist with head rotation and head flexion, movement of the shoulders, and extension and turning of the head.

A patient's thyroid is enlarged, and the nurse is preparing to auscultate the thyroid for the presence of a bruit. A bruit is a: 1. low gurgling sound best heard with the diaphragm of the stethoscope. 2. loud, whooshing, blowing sound best heard with the bell of the stethoscope. 3. soft, whooshing, pulsatile sound best heard with the bell of the stethoscope. 4. high-pitched tinkling sound best heard with the diaphragm of the stethoscope.

ANS: 3 If the thyroid gland is enlarged, auscultate it for the presence of a bruit, which is a soft, pulsatile, whooshing, blowing sound heard best with the bell of the stethoscope.

During an examination, the nurse knows that Paget's disease would be indicated by which of the following findings? 1. Positive Macewen's sign 2. Premature closure of the sagittal suture 3. Headache, vertigo, tinnitus, and deafness 4. Elongated head with heavy eyebrow ridge

ANS: 3 Paget's disease occurs more often in males and is characterized by bowed, long bones, sudden fractures, and enlarging skull bones that press on cranial nerves causing symptoms of headache, vertigo, tinnitus, and progressive deafness.

While performing a well-child assessment on a 5-year-old, the nurse notes the presence of palpable, bilateral, cervical, and inguinal lymph nodes. They are approxi- mately 0.5 cm in size, round, mobile, and nontender. The nurse suspects that this: 1. child has chronic allergies. 2. child may have an infection. 3. is a normal finding for a well child of this age. 4. child should be referred for additional evaluation.

ANS: 3 Palpable lymph nodes are normal in children until puberty when the lymphoid tissue begins to atrophy. Lymph nodes may be up to 1 cm in size in the cervical and inguinal areas, but are discrete, movable, and nontender.

During an interview, the nurse would expect that most of the interview will take place at which distance? 1.Intimate zone 2.Personal distance 3.Social distance 4.Public distance

ANS: 3 Social distance, 4 to 12 feet, is usually the distance category for most of the interview. Public distance, over 12 feet, is too much distance; the intimate zone is inappropriate, and the personal distance will be used for the physical assessment.

A physician tells the nurse that a patient's vertebra prominens is tender and asks the nurse to re-evaluate the area in 1 hour. The area of the body the nurse will assess is the area: 1.just above the diaphragm. 2.just lateral to the knee cap. 3.at the level of the C7 vertebra. 4.at the level of the T11 vertebra.

ANS: 3 The C7 vertebra has a long spinous process, called the vertebra prominens, that is palpable when the head is flexed.

During an examination of a patient in her third trimester of pregnancy, the nurse notices that the patient's thyroid gland is slightly enlarged. No enlargement had been noted previously. The nurse suspects that: 1. she has an iodine deficiency. 2. she is exhibiting early signs of goiter. 3. this is a normal finding during pregnancy. 4. further tests are needed for possible thyroid cancer.

ANS: 3 The thyroid gland enlarges slightly during pregnancy owing to hyperplasia of the tissue and increased vascularity.

During an interview, a woman says, "I have decided that I can no longer allow my children to live with their father's violence. I just can't seem to leave him though." Using interpretation, the nurse's best response would be: 1."You're going to leave him?" 2."If you're afraid for your children, why can't you leave?" 3."It sounds as if you might be afraid of how your husband will respond." 4."It sounds as though you have made your decision. I think it is a good one."

ANS: 3 This statement is not based on one's inference or conclusion. It links events, makes associations, or implies cause. Interpretation also ascribes feelings and helps the person understand his or her own feelings in relation to the verbal message.

When examining the face, the nurse is aware that the two pairs of salivary glands that are accessible to examination are the _____ glands. 1. occipital and submental 2. parotid and jugulodigastric 3. parotid and submandibular 4. submandibular and occipital

ANS: 3 Two pairs of salivary glands accessible to examination on the face are the parotid glands in the cheeks over the mandible, anterior to and below the ear, and the submandibular glands, beneath the mandible at the angle of the jaw. The parotid glands are not normally palpable.

Critique the following statement made by the nurse: "I know it may be hard, but you should do what the doctor ordered because he/she is the expert in this field." 1. This statement is inappropriate because it shows the nurse's bias. 2. This statement is appropriate because members of the health care team are experts in the area of patient care. 3. This type of statement promotes dependency and inferiority on the part of the patient and is best avoided in an interview situation. 4. At times, it is necessary to use authority statements when dealing with patients, especially when they are undecided about an issue.

ANS: 3 Using authority responses promotes dependency and inferiority. It is best to avoid using authority. Although the health care provider and patient do not have equal professional knowledge, both have equally worthy roles in the health process.

A nurse is taking complete health histories on all the patients attending a wellness workshop. On the history form, one of the written question asks, "You don't smoke, drink, or take drugs, do you?" This question is an example of: 1.talking too much. 2.using confrontation. 3.using biased or leading questions. 4.using blunt language to deal with distasteful topics.

ANS: 3 Using leading or biased questions. Asking, "You don't smoke, do you?" implies that one answer is "better" than another. If the person wants to please you, he or she is either forced to answer in a way corresponding to your values or is made to feel guilty when admitting the other answer.

A man has been admitted to the observation unit for observation after being treated for a large cut on his forehead. As the nurse works through the interview, one of the standard questions has to do with alcohol, tobacco, and drug use. When the nurse asks him about tobacco use, he states, "I quit smoking after my wife died 7 years ago." However, the nurse notices an open package of cigarettes in his shirt pocket. Using confrontation, the nurse could say: 1."Mr. K., I know that you are lying." 2."Mr. K., come on, tell me how much you smoke." 3."Mr. K., I didn't realize your wife had died. It must be difficult for you at this time. Please tell me more about that." 4."Mr. K., you have said that you don't smoke, but I see that you have an open package of cigarettes in your pocket."

ANS: 4 In the case of confrontation, a certain action, feeling, or statement has been observed, and you now focus the person's attention on it. You give your honest feedback about what you see or feel. This may focus on a discrepancy. Or you may confront the person when you notice parts of the story are inconsistent.

A patient complains that while studying for an examination he began to notice a severe headache in the frontotemporal area of his head that is throbbing and is somewhat relieved when he lies down. He tells the nurse that his mother also had these headaches. The nurse suspects that he may be suffering from: 1. hypertension. 2. cluster headaches. 3. tension headaches. 4. migraine headaches.

ANS: 4 Migraine headaches tend to be supraorbital, retro-orbital, or frontotemporal with a throbbing quality. They are of a severe quality and are relieved by lying down. Migraines are associated with family history of migraine.

During an interview, a parent of a hospitalized child is sitting in an open position. As the interviewer begins to discuss his son's treatment, however, he suddenly crosses his arms against his chest and crosses his legs. This would suggest that the parent is: 1.just changing positions. 2.more comfortable in this position. 3.tired and needs a break in the interview. 4.uncomfortable talking about his son's treatment.

ANS: 4 Note the patient's position. An open position with the extension of large muscle groups shows relaxation, physical comfort, and a willingness to share information. A closed position with the arms and legs crossed tends to look defensive and anxious. Note any change in posture. If a person in a relaxed position suddenly tenses, it suggests possible discomfort with the new topic.

A man arrives at the clinic for an annual wellness physical. He is experiencing no acute health problems. Which of the following statements by the nurse is most appropriate when beginning the interview? 1."How is your family?" 2."Why are you here today?" 3."Tell me about your hypertension?" 4."How has your health been since your last visit?"

ANS: 4 Open-ended questions are used for narrative information. Use this type of question- ing to begin the interview, to introduce a new section of questions, and whenever the person introduces a new topic.

The interview portion of data collection collects: 1.physical data. 2.historical data. 3.objective data. 4.subjective data.

ANS: 4 The interview is the first, and really the most important part of data collection, collecting subjective data—what the person says about himself or herself.

A female nurse is interviewing a male patient who is near the same age as the nurse. During the interview, the patient makes an overtly sexual comment. The nurse's best reaction would be: 1."Stop that immediately!" 2."Oh, you are too funny. Let's keep going with the interview." 3."Do you really think I'd be interested?" 4."It makes me uncomfortable when you talk that way. Please don't."

ANS: 4 The nurse's response must make it clear that she is a health professional who can best care for the person by maintaining a professional relationship. At the same time, the nurse should communicate that she accepts the person and understands the person's need to be self-assertive but that sexual advances cannot be tolerated.

During an interview, the nurse states: "You mentioned shortness of breath. Tell me more about that." Identify the verbal skill used with this statement. 1.Reflection 2.Facilitation 3.Direct question 4.Open-ended question

ANS: 4 The open-ended question asks for narrative information. It states the topic to be discussed but only in general terms. Use it to begin the interview, to introduce a new section of questions, and whenever the person introduces a new topic.

A patient has finished giving the nurse information about the reason he is seeking care. When reviewing the data, the nurse finds that some information about past hospitalizations is missing. Which statement by the nurse would be most appropriate to gather these data? 1."Mr. Y., at your age, surely you have been hospitalized before!" 2."Mr. Y., I just need permission to get your medical records from County Medical." 3."Mr. Y., you mentioned that you have been hospitalized on several occasions. Would you tell me more about that?" 4."Mr. Y., I just need to get some additional information about your past hospitaliza- tions. When was the last time you were admitted for chest pain?"

ANS: 4 Use direct questions after the person's opening narrative to fill in any details he or she left out. Also use direct questions when specific facts are needed, such as when asking about past health problems or during the review of system

The nurse notices that a patient's submental lymph nodes are enlarged. In an effort to identify the cause of the node enlargement, the nurse would assess the: 1. infraclavicular area. 2. supraclavicular area. 3. area distal to the enlarged node. 4. area proximal to the enlarged node.

ANS: 4 When nodes are abnormal, check the area they drain for the source of the problem. Explore the area proximal (upstream) to the location of the abnormal node.

When observing a patient's verbal and nonverbal communication, the nurse notices a discrepancy. Which statement is true regarding this situation? 1.The nurse should ask someone who knows the patient well to help interpret this discrepancy. 2.The nurse should focus on the patient's verbal message and try to ignore the nonverbal behaviors. 3.The nurse should try to integrate the verbal and nonverbal messages and then interpret them as an "average." 4.The nurse should focus on the patient's nonverbal behaviors because these are often more reflective of a patient's true feelings.

ANS: 4 When nonverbal and verbal messages are congruent, the verbal is reinforced. When they are incongruent, the nonverbal message tends to be the true one because it is under less conscious control. Thus it is important to study the nonverbal messages of patients and examiners and to understand their meanings.

In using verbal responses to assist the patient's narrative, some responses focus on the patient's frame of reference and some focus on the health care provider's perspective. An example of a verbal response that focuses on the health care provider's perspective would be: 1.empathy. 2.reflection. 3.facilitation. 4.confrontation.

ANS: 4 When using the response of confrontation, the frame or reference shifts from the patient's perspective to yours, and you start to express your own thoughts and feelings.

18. To assess a rectal temperature accurately in an adult, the nurse would: a. Use a lubricated blunt tip thermometer. b. Insert the thermometer 2 to 3 inches into the rectum. c. Leave the thermometer in place up to 8 minutes if the patient is febrile. d. Wait 2 to 3 minutes if the patient has recently smoked a cigarette.

ANS: A A lubricated rectal thermometer (with a short, blunt tip) is inserted only 2 to 3 cm (1 inch) into the adult rectum and left in place for 2 minutes. Cigarette smoking does not alter rectal temperatures.

10. A 25-year-old woman in her fifth month of pregnancy has a blood pressure of 100/70 mm Hg. In reviewing her previous exam, the nurse notes that her blood pressure in her second month was 124/80 mm Hg. In evaluating this change, what does the nurse know to be true? A. This is the result of peripheral vasodilatation and is an expected change. B. Because of increased cardiac output, the blood pressure should be higher this time. C. This is not an expected finding because it would mean a decreased cardiac output. D. This would mean a decrease in circulating blood volume, which is dangerous for the fetus

ANS: A Despite the increased cardiac output, arterial blood pressure decreases in pregnancy because of peripheral vasodilatation. The blood pressure drops to its lowest point during the second trimester and then rises after that.

47. The nurse is assessing an 8-year-old child whose growth rate measures below the third percentile for a child his age. He appears significantly younger than his stated age and is chubby with infantile facial features. Which condition does this child have? a. Hypopituitary dwarfism b. Achondroplastic dwarfism c. Marfan syndrome d. Acromegaly

ANS: A Hypopituitary dwarfism is caused by a deficiency in growth hormone in childhood and results in a retardation of growth below the third percentile, delayed puberty, and other problems. The childs appearance fits this description. Achondroplastic dwarfism is a genetic disorder resulting in characteristic deformities; Marfan syndrome is an inherited connective tissue disorder characterized by a tall, thin stature and other features. Acromegaly is the result of excessive secretion of growth hormone in adulthood. (For more information, see Table 9-5, Abnormalities in Body Height and Proportion.)

33. When auscultating the blood pressure of a 25-year-old patient, the nurse notices the phase I Korotkoff sounds begin at 200 mm Hg. At 100 mm Hg, the Korotkoff sounds muffle. At 92 mm Hg, the Korotkoff sounds disappear. How should the nurse record this patients blood pressure? a. 200/92 b. 200/100 c. 100/200/92 d. 200/100/92

ANS: A In adults, the last audible sound best indicates the diastolic pressure. When a variance is greater than 10 to 12 mm Hg between phases IV and V, both phases should be recorded along with the systolic reading (e.g., 142/98/80).

45. A 75-year-old man with a history of hypertension was recently changed to a new antihypertensive drug. He reports feeling dizzy at times. How should the nurse evaluate his blood pressure? a. Blood pressure and pulse should be recorded in the supine, sitting, and standing positions. b. The patient should be directed to walk around the room and his blood pressure assessed after this activity. c. Blood pressure and pulse are assessed at the beginning and at the end of the examination. d. Blood pressure is taken on the right arm and then 5 minutes later on the left arm.

ANS: A Orthostatic vital signs should be taken when the person is hypertensive or is taking antihypertensive medications, when the person reports fainting or syncope, or when volume depletion is suspected. The blood pressure and pulse readings are recorded in the supine, sitting, and standing positions.

A 75-year-old woman who has a history of diabetes and peripheral vascular disease has been trying to remove a corn on the bottom of her foot with a pair of scissors. The nurse will encourage her to stop trying to remove the corn with scissors because: a. The woman could be at increased risk for infection and lesions because of her chronic disease. b. With her diabetes, she has increased circulation to her foot, and it could cause severe bleeding. c. She is 75 years old and is unable to see; consequently, she places herself at greater risk for self-injury with the scissors. d. With her peripheral vascular disease, her range of motion is limited and she may not be able to reach the corn safely.

ANS: A A personal history of diabetes and peripheral vascular disease increases a person's risk for skin lesions in the feet or ankles. The patient needs to seek a professional for assistance with corn removal.

When performing a genitourinary assessment, the nurse notices that the urethral meatus is positioned ventrally. This finding is: A) called hypospadias. B) the result of phimosis. C) probably due to a stricture. D) often associated with aging.

ANS: A Normally the urethral meatus is positioned just about centrally. Hypospadias is the ventral location of the urethral meatus. The position of the meatus does not change with aging. Phimosis is the inability to retract the foreskin. A stricture is a narrow opening of the meatus.

The nurse is preparing to interview a postmenopausal woman. Which of these statements is true with regard to the history of a postmenopausal woman? A) The nurse should ask a postmenopausal woman if she ever has vaginal bleeding. B) Once a woman reaches menopause, the nurse does not need to ask any further history questions. C) The nurse should screen for monthly breast tenderness. D) Postmenopausal women are not at risk for contracting sexually transmitted infections and thus these questions can be omitted.

ANS: A Postmenopausal bleeding warrants further workup and referral. The other statements are not true.

The nurse is inspecting the scrotum and testes of a 43-year-old man. Which finding would require additional follow-up and evaluation? A) The skin on the scrotum is taut. B) The left testicle hangs lower than the right testicle. C) The scrotal skin has yellowish 1-cm nodules that are firm and nontender. D) The testes move closer to the body in response to cold temperatures.

ANS: A Scrotal swelling may cause the skin to be taut and to display pitting edema. Normal scrotal skin is rugae, and asymmetry is normal with the left scrotal half usually lower than the right. The testes may move closer to the body in response to cold temperatures.

During an examination the nurse observes a female patient's vestibule and expects to see the: A) urethral meatus and vaginal orifice. B) vaginal orifice and vestibular (Bartholin) glands. C) urethral meatus and paraurethral (Skene) glands. D) paraurethral (Skene) and vestibular (Bartholin) glands.

ANS: A The labial structures encircle a boat-shaped space, or cleft, termed the vestibule. Within it are numerous openings. The urethral meatus and vaginal orifice are visible. The ducts of the paraurethral (Skene) glands and the vestibular (Bartholin) glands are present but not visible.

The nurse is preparing to palpate the rectum and should use which of these techniques? A) Flex the finger and insert slowly toward the umbilicus. B) Instruct the patient first that this will be a painful procedure. C) Insert an extended index finger at a right angle to the anus. D) Place the finger directly into the anus to overcome the tight sphincter.

ANS: A The nurse should place the pad of the index finger gently against the anal verge. The nurse will feel the sphincter tighten and then relax. As it relaxes, the nurse should flex the tip of the finger and slowly insert it into the anal canal in a direction toward the umbilicus. The nurse should never approach the anus at right angles with the index finger extended—this would cause pain. The nurse should instruct the patient that palpation is not painful but may feel like needing to move the bowels.

During a vaginal examination of a 38-year-old woman, the nurse notices that the vulva and vagina are erythematous and edematous with thick, white, curdlike discharge adhering to the vaginal walls. The woman reports intense pruritus and thick white discharge from her vagina. The nurse knows that these history and physical examination findings are most consistent with which of these conditions? A) Candidiasis B) Trichomoniasis C) Atrophic vaginitis D) Bacterial vaginosis

ANS: A The woman with candidiasis often reports intense pruritus and thick white discharge. The vulva and vagina are erythematous and edematous. The discharge is usually thick, white, and curdlike. Infection with trichomoniasis causes a profuse, watery, gray-green, and frothy discharge. Bacterial vaginosis causes a profuse discharge that has a "foul, fishy, rotten" odor. Atrophic vaginitis may have a mucoid discharge. See Table 26-5 for complete descriptions of each option.

During a physical examination, the nurse finds that a male patient's foreskin is fixed and tight and will not retract over the glans. The nurse recognizes that this condition is: A) phimosis. B) epispadias. C) urethral stricture. D) Peyronie's disease.

ANS: A With phimosis, the foreskin is nonretractable, forming a pointy tip of the penis with a tiny orifice at the end of the glans. The foreskin is advanced and so tight that it is impossible to retract over the glans. This may be congenital or acquired from adhesions related to infection. See Table 24-3 for information on urethral stricture. See Table 24-4 for information on epispadias and Peyronie's disease.

41. The nurse is assessing the apical pulse of a 3-month-old infant and finds that the heart rate is 135 beats per minute. The nurse interprets this result as: A. Normal for this age. B. Lower than expected. C. Higher than expected, probably as a result of crying. D. Higher than expected, reflecting persistent tachycardia.

ANS: A The heart rate may range from 100 to 180 beats per minute immediately after birth and then stabilize to an average of 120 to 140 beats per minute. Infants normally have wide fluctuations with activity, from 170 beats per minute or more with crying or being active to 70 to 90 beats per minute with sleeping. Persistent tachycardia is greater than 200 beats per minute in newborns or greater than 150 beats per minute in infants.

A patient tells the nurse that he has noticed that one of his moles has started to burn and bleed. When assessing his skin, the nurse pays special attention to the danger signs for pigmented lesions and is concerned with which additional finding? a. Color variation b. Border regularity c. Symmetry of lesions d. Diameter of less than 6 mm

ANS: A Abnormal characteristics of pigmented lesions are summarized in the mnemonic ABCD: asymmetry of pigmented lesion, border irregularity, color variation, and diameter greater than 6 mm. DIF: Cognitive Level: Understanding (Comprehension) MSC: Client Needs: Health Promotion and Maintenance

A patient is recovering from several hours of orthopedic surgery. During an assessment of the patient's lower legs, the nurse will monitor for signs of acute venous symptoms. Signs of acute venous symptoms include which of the following? Select all that apply. a. Intense, sharp pain, with the deep muscle tender to the touch b. Aching, tired pain, with a feeling of fullness c. Pain that is worse at the end of the day d. Sudden onset e. Warm, red, and swollen calf f. Pain that is relieved with elevation of the leg

ANS: A. Intense, sharp pain, with the deep muscle tender to touch D. Sudden onset E. Warm, red, and swollen calf Signs and symptoms of acute venous problems include pain in the calf that has a sudden onset and that is intense and sharp with tenderness in the deep muscle when touched. The calf is warm, red, and swollen. The other options are symptoms of chronic venous problems.

A patient has been admitted with chronic arterial symptoms. During the assessment, the nurse should expect which findings? Select all that apply. a. Patient has a history of diabetes and cigarette smoking. b. Skin of the patient is pale and cool. c. His ankles have two small, weeping ulcers. d. Patient works long hours sitting at a computer desk. e. He states that the pain gets worse when walking. f. Patient states that the pain is worse at the end of the day.

ANS: A. The patient has a history of diabetes and cigarette smoking. B. The patient's skin is pale and cool. E. The patient states that the pain gets worse when walking. See Table 20-3. Patients with chronic arterial symptoms often have a history of smoking and diabetes (among other risk factors). The pain has a gradual onset, with exertion, and is relieved with rest or dangling. The skin appears cool and pale. The other responses reflect chronic venous problems.

The nurse is assessing voice sounds during a respiratory assessment. Which of these findings indicates a normal assessment? Select all that apply. a. Voice sounds are faint, muffled, and almost inaudible when the patient whispers "one, two, three" in a very soft voice. b. As the patient repeatedly says "ninety-nine," the examiner clearly hears the words "ninety-nine." c. When the patient speaks in a normal voice, the examiner can hear a sound but cannot exactly distinguish what is being said. d. As the patient says a long "ee-ee-ee" sound, the examiner also hears a long "ee-ee-ee" sound. e. As the patient says a long "ee-ee-ee" sound, the examiner hears a long "aaaaaa" sound.

ANS: A. Voice sounds are faint, muffled, and almost inaudible when the patient whispers "one, two, three" in a very soft voice. C. When the patient speaks in a normal voice, the examiner can hear a sound but cannot distinguish exactly what is being said. D. As the patient says a long "ee-ee-ee" sound, the examiner also hears a long "ee-ee-ee" sound. As a patient says "ninety-nine" repeatedly, normally, the examiner hears sound but cannot distinguish what is being said. If a clear "ninety-nine" is auscultated, then it could indicate increased lung density, which enhances transmission of voice sounds. This is a measure of bronchophony. When a patient says a long "ee-ee-ee" sound, normally the examiner also hears a long "ee-ee-ee" sound through auscultation. This is a measure of egophony. If the examiner hears a long "aaaaaa" sound instead, this could indicate areas of consolidation or compression. With whispered pectoriloquy, as when a patient whispers a phrase such as "one-two-three," the normal response when auscultating voice sounds is to hear sounds that are faint, muffled, and almost inaudible. If the examiners hears the whispered voice clearly, as if the patient is speaking through the stethoscope, then consolidation of the lung fields may exist.

The nurse is reviewing statistics regarding breast cancer. Which woman, aged 40 years in the United States, has the highest risk for development of breast cancer? a. Black b. White c. Asian d. American Indian

ANS: African-American The incidence of breast cancer varies with different cultural groups. White women have a higher incidence of breast cancer than African-American women starting at age 45 years; but African-American women have a higher incidence before age 45 years. Asian, Hispanic, and American Indian women have a lower risk for development of breast cancer (American Cancer Society, 2009-2010).

The nurse is preparing to assess the ankle-brachial index (ABI) of a patient. Which statement about the ABI is true? a. Normal ABI indices are from 0.5 to 1.0. b. Normal ankle pressure is slightly lower than the brachial pressure. c. The ABI is a reliable measurement of peripheral vascular disease in individuals with diabetes. d. An ABI of 0.9 to 0.7 indicates the presence of peripheral vascular disease and mild claudication.

ANS: An ABI of 0.90 to 0.70 indicates the presence of peripheral vascular disease and mild claudication. Use of the Doppler stethoscope is a noninvasive way to determine the extent of peripheral vascular disease. The normal ankle pressure is slightly greater than or equal to the brachial pressure. An ABI of 0.90 to 0.70 indicates the presence of peripheral vascular disease and mild claudication. The ABI is less reliable in patients with diabetes mellitus because of claudication, which makes the arteries noncompressible and may give a falsely high ankle pressure.

A patient has a long history of chronic obstructive pulmonary disease. During the assessment, the nurse is most likely to observe which of these? a. Unequal chest expansion b. Increased tactile fremitus c. Atrophied neck and trapezius muscles d. Anteroposterior-to-transverse diameter ratio of 1:1

ANS: An anteroposterior-to-transverse diameter ratio of 1:1 An anteroposterior-to-transverse diameter of 1:1 or "barrel chest" is seen in individuals with chronic obstructive pulmonary disease because of hyperinflation of the lungs. The ribs are more horizontal, and the chest appears as if held in continuous inspiration. Neck muscles are hypertrophied from aiding in forced respiration. Chest expansion may be decreased but symmetric. Decreased tactile fremitus occurs from decreased transmission of vibrations.

A 65-year-old patient with a history of heart failure comes to the clinic with complaints of "being awakened from sleep with shortness of breath." Which action by the nurse is most appropriate? a. Obtaining a detailed health history of the patient's allergies and a history of asthma b. Telling the patient to sleep on his or her right side to facilitate ease of respirations c. Assessing for other signs and symptoms of paroxysmal nocturnal dyspnea d. Assuring the patient that paroxysmal nocturnal dyspnea is normal and will probably resolve within the next week

ANS: Assess for other signs and symptoms of paroxysmal nocturnal dyspnea. The patient is experiencing paroxysmal nocturnal dyspnea: being awakened from sleep with shortness of breath and the need to be upright to achieve comfort.

During an admission assessment of a patient with dementia, the nurse assesses for pain because the patient has recently had several falls. Which of these are appropriate for the nurse to assess in a patient with dementia? Select all that apply.

ANS: Assess the patient's breathing independent of vocalization., Note whether the patient is calling out, groaning, or crying., Observe the patient's body language for pacing and agitation. Patients with dementia may say "no" when, in reality, they are very uncomfortable because words have lost their meaning. Patients with dementia become less able to identify and describe pain over time, even though pain is still present. People with dementia communicate pain through their behaviors. Agitation, pacing, and repetitive yelling may indicate pain and not a worsening of the dementia. See Figure 10-10 for the Pain Assessment in Advanced Dementia (PAINAD) Scale, which may also be used to assess pain in persons with dementia.

When assessing a patient the nurse notes that the left femoral pulse as diminished, 1+/4+. What should the nurse do next? a. Document the finding. b. Auscultate the site for a bruit. c. Check for calf pain. d. Check capillary refill in the toes.

ANS: Auscultate the site for a bruit. If a pulse is weak or diminished at the femoral site, the nurse should auscultate for a bruit. Presence of a bruit, or turbulent blood flow, indicates partial occlusion. The other responses are not correct.

28. A 70-year-old patient with a history of hypertension has a blood pressure of 180/100 mm Hg and a heart rate of 90 beats per minute. The nurse hears an extra heart sound at the apex immediately before S1. The sound is heard only with the bell while the patient is in the left lateral position. With these findings and the patient's history, the nurse knows that this extra heart sound is most likely a(n): A. Split S1. B. Atrial gallop. C. Diastolic murmur. D. Summation sound

ANS: B A pathologic S4 is termed an atrial gallop or an S4 gallop. It occurs with decreased compliance of the ventricle and with systolic overload (afterload), including outflow obstruction to the ventricle (aortic stenosis) and systemic hypertension. A left-sided S4 occurs with these conditions. It is heard best at the apex with the patient in the left lateral position.

3. A patients weekly blood pressure readings for 2 months have ranged between 124/84 mm Hg and 136/88 mm Hg, with an average reading of 126/86 mm Hg. The nurse knows that this blood pressure falls within which blood pressure category? a. Normal blood pressure b. Prehypertension c. Stage 1 hypertension d. Stage 2 hypertension

ANS: B According to the Seventh Report of the Joint National Committee (JNC 7) guidelines, prehypertension blood pressure readings are systolic readings of 120 to 139 mm Hg or diastolic readings of 50 to 89 mm Hg.

39. The nurse is conducting a health fair for older adults. Which statement is true regarding vital sign measurements in aging adults? a. The pulse is more difficult to palpate because of the stiffness of the blood vessels. b. An increased respiratory rate and a shallower inspiratory phase are expected findings. c. A decreased pulse pressure occurs from changes in the systolic and diastolic blood pressures. d. Changes in the bodys temperature regulatory mechanism leave the older person more likely to develop a fever.

ANS: B Aging causes a decrease in vital capacity and decreased inspiratory reserve volume. The examiner may notice a shallower inspiratory phase and an increased respiratory rate. An increase in the rigidity of the arterial walls makes the pulse actually easier to palpate. Pulse pressure is widened in older adults, and changes in the body temperature regulatory mechanism leave the older person less likely to have fever but at a greater risk for hypothermia.

20. The nurse is preparing to auscultate for heart sounds. Which technique is correct? A. Listen to the sounds at the aortic, tricuspid, pulmonic, and mitral areas. B. Listen by inching the stethoscope in a rough Z pattern, from the base of the heart across and down, then over to the apex. C. Listen to the sounds only at the site where the apical pulse is felt to be the strongest. D. Listen for all possible sounds at a time at each specified area

ANS: B Do not limit auscultation of breath sounds to only four locations. Sounds produced by the valves may be heard all over the precordium. Inch the stethoscope in a rough Z pattern from the base of the heart across and down, then over to the apex. Or, start at the apex and work your way up. See Figure 19-22. Listen selectively to one sound at a time.

27. A nurse is helping at a health fair at a local mall. When taking blood pressures on a variety of people, the nurse keeps in mind that: a. After menopause, blood pressure readings in women are usually lower than those taken in men. b. The blood pressure of a Black adult is usually higher than that of a White adult of the same age. c. Blood pressure measurements in people who are overweight should be the same as those of people who are at a normal weight. d. A teenagers blood pressure reading will be lower than that of an adult.

ANS: B In the United States, a Black adults blood pressure is usually higher than that of a White adult of the same age. The incidence of hypertension is twice as high in Blacks as it is in Whites. After menopause, blood pressure in women is higher than in men; blood pressure measurements in people who are obese are usually higher than in those who are not overweight. Normally, a gradual rise occurs through childhood and into the adult years.

3. The nurse is assessing the vital signs of a 20-year-old male marathon runner and documents the following vital signs: temperature36 C; pulse48 beats per minute; respirations14 breaths per minute; blood pressure104/68 mm Hg. Which statement is true concerning these results? a. The patient is experiencing tachycardia. b. These are normal vital signs for a healthy, athletic adult. c. The patients pulse rate is not normalhis physician should be notified. d. On the basis of these readings, the patient should return to the clinic in 1 week.

ANS: B In the adult, a heart rate less than 50 beats per minute is called bradycardia, which normally occurs in the well-trained athlete whose heart muscle develops along with the skeletal muscles.

33. The nurse knows that normal splitting of the second heart sound is associated with: A. Expiration. B. Inspiration. C. Exercise state. D. Low resting heart rate.

ANS: B Normal or physiologic splitting of the second heart sound is associated with inspiration because of the increased blood return to the right side of the heart, delaying closure of the pulmonic valve.

35. A 70-year-old man has a blood pressure of 150/90 mm Hg in a lying position, 130/80 mm Hg in a sitting position, and 100/60 mm Hg in a standing position. How should the nurse evaluate these findings? a. These readings are a normal response and attributable to changes in the patients position. b. The change in blood pressure readings is called orthostatic hypotension. c. The blood pressure reading in the lying position is within normal limits. d. The change in blood pressure readings is considered within normal limits for the patients age.

ANS: B Orthostatic hypotension is a drop in systolic pressure of more than 20 mm Hg, which occurs with a quick change to a standing position. Aging people have the greatest risk of this problem.

46. Which of these specific measurements is the best index of a childs general health? a. Vital signs b. Height and weight c. Head circumference d. Chest circumference

ANS: B Physical growth, measured by height and weight, is the best index of a childs general health.

25. While auscultating heart sounds on a 7-year-old child for a routine physical, the nurse hears an S3, a soft murmur at left midsternal border, and a venous hum when the child is standing. Which of these would be a correct interpretation of these findings? A. S3 is indicative of heart disease in children. B. These can all be normal findings in a child. C. These are indicative of congenital problems. D. The venous hum most likely indicates an aneurysm

ANS: B Physiologic S3 is common in children. A venous hum, caused by turbulence of blood flow in the jugular venous system, is common in healthy children and has no pathologic significance. Heart murmurs that are innocent (or functional) in origin are very common through childhood.

32. The nurse has collected the following information on a patient: palpated blood pressure180 mm Hg; auscultated blood pressure170/100 mm Hg; apical pulse60 beats per minute; radial pulse70 beats per minute. What is the patients pulse pressure? a. 10 b. 70 c. 80 d. 100

ANS: B Pulse pressure is the difference between systolic and diastolic blood pressure (170 100 = 70) and reflects the stroke volume.

2. The direction of blood flow through the heart is best described by which of these? A. Vena cava right atrium right ventricle lungs pulmonary artery left atrium left ventricle B. Right atrium right ventricle pulmonary artery lungs pulmonary vein left atrium left ventricle C. Aorta right atrium right ventricle lungs pulmonary vein left atrium left ventricle vena cava D. Right atrium right ventricle pulmonary vein lungs pulmonary artery left atrium left ventricle

ANS: B Returning blood from the body empties into the right atrium and flows into the right ventricle and then goes to the lungs through the pulmonary artery. The lungs oxygenate the blood and it is then returned to the left atrium by the pulmonary vein. It goes from there to the left ventricle and then out to the body through the aorta.

11. The nurse is examining a patient who is complaining of feeling cold. Which is a mechanism of heat loss in the body? a. Exercise b. Radiation c. Metabolism d. Food digestion

ANS: B The body maintains a steady temperature through a thermostat or feedback mechanism, which is regulated in the hypothalamus of the brain. The hypothalamus regulates heat production from metabolism, exercise, food digestion, and external factors with heat loss through radiation, evaporation of sweat, convection, and conduction.

5. A 1-month-old infant has a head measurement of 34 cm and has a chest circumference of 32 cm. Based on the interpretation of these findings, the nurse would: a. Refer the infant to a physician for further evaluation. b. Consider these findings normal for a 1-month-old infant. c. Expect the chest circumference to be greater than the head circumference. d. Ask the parent to return in 2 weeks to re-evaluate the head and chest circumferences.

ANS: B The newborns head measures approximately 32 to 38 cm and is approximately 2 cm larger than the chest circumference. Between 6 months and 2 years, both measurements are approximately the same, and after age 2 years, the chest circumference is greater than the head circumference.

38. A 4-month-old child is at the clinic for a well-baby check-up and immunizations. Which of these actions is most appropriate when the nurse is assessing an infants vital signs? a. The infants radial pulse should be palpated, and the nurse should notice any fluctuations resulting from activity or exercise. b. The nurse should auscultate an apical rate for 1 minute and then assess for any normal irregularities, such as sinus arrhythmia. c. The infants blood pressure should be assessed by using a stethoscope with a large diaphragm piece to hear the soft muffled Korotkoff sounds. d. The infants chest should be observed and the respiratory rate counted for 1 minute; the respiratory pattern may vary significantly.

ANS: B The nurse palpates or auscultates an apical rate with infants and toddlers. The pulse should be counted for 1 full minute to account for normal irregularities, such as sinus arrhythmia. Children younger than 3 years of age have such small arm vessels; consequently, hearing Korotkoff sounds with a stethoscope is difficult. The nurse should use either an electronic blood pressure device that uses oscillometry or a Doppler ultrasound device to amplify the sounds.

16. Which of these actions illustrates the correct technique the nurse should use when assessing oral temperature with a mercury thermometer? a. Wait 30 minutes if the patient has ingested hot or iced liquids. b. Leave the thermometer in place 3 to 4 minutes if the patient is afebrile. c. Place the thermometer in front of the tongue, and ask the patient to close his or her lips. d. Shake the mercury-in-glass thermometer down to below 36.6 C before taking the temperature.

ANS: B The thermometer should be left in place 3 to 4 minutes if the person is afebrile and up to 8 minutes if the person is febrile. The nurse should wait 15 minutes if the person has just ingested hot or iced liquids and 2 minutes if he or she has just smoked.

28. The nurse notices a colleague is preparing to check the blood pressure of a patient who is obese by using a standard-sized blood pressure cuff. The nurse should expect the reading to: a. Yield a falsely low blood pressure. b. Yield a falsely high blood pressure. c. Be the same, regardless of cuff size. d. Vary as a result of the technique of the person performing the assessment.

ANS: B Using a cuff that is too narrow yields a falsely high blood pressure because it takes extra pressure to compress the artery.

10. When assessing an older adult, which vital sign changes occur with aging? a. Increase in pulse rate b. Widened pulse pressure c. Increase in body temperature d. Decrease in diastolic blood pressure

ANS: B With aging, the nurse keeps in mind that the systolic blood pressure increases, leading to widened pulse pressure. With many older people, both the systolic and diastolic pressures increase. The pulse rate and temperature do not increase.

A 54-year-old woman who has just completed menopause is in the clinic today for a yearly physical examination. Which of these statements should the nurse include in patient education? "A postmenopausal woman: A) is not at any greater risk for heart disease than a younger woman is." B) should be aware that she is at increased risk for dyspareunia because of decreased vaginal secretions." C) has only stopped menstruating; there really are no other significant changes with which she should be concerned." D) is likely to have difficulty with sexual pleasure as a result of drastic changes in the female sexual response cycle."

ANS: B Decreased vaginal secretions leave the vagina dry and at risk for irritation and pain with intercourse (dyspareunia). The other statements are incorrect.

A 22-year-old woman comes to the clinic because of severe sunburn and states, "I was out in the sun for just a couple of minutes." The nurse begins a medication review with her, paying special attention to which medication class? a. Nonsteroidal antiinflammatory drugs for pain b. Tetracyclines for acne c. Proton pump inhibitors for heartburn d. Thyroid replacement hormone for hypothyroidism

ANS: B Drugs that may increase sunlight sensitivity and give a burn response include sulfonamides, thiazide diuretics, oral hypoglycemic agents, and tetracycline.

A 55-year-old man is experiencing severe pain of sudden onset in the scrotal area. It is somewhat relieved by elevation. On examination the nurse notices an enlarged, red scrotum that is very tender to palpation. It is difficult to distinguish the epididymis from the testis, and the scrotal skin is thick and edematous. This description is consistent with which of these? A) Varicocele B) Epididymitis C) Spermatocele D) Testicular torsion

ANS: B Epididymitis presents as severe pain of sudden onset in the scrotum that is somewhat relieved by elevation. On examination, the scrotum is enlarged, reddened, and exquisitely tender. The epididymis is enlarged and indurated and may be hard to distinguish from the testis. The overlying scrotal skin may be thick and edematous. See Table 24-6 for more information and for descriptions of the other terms

A 30-year-old woman is visiting the clinic because of "pain in my bottom when I have a bowel movement." The nurse should assess for which problem? A) Pinworms B) Hemorrhoids C) Colon cancer D) Fecal incontinence

ANS: B Having painful bowel movements, known as dyschezia, may be due to a local condition (hemorrhoid or fissure) or constipation. The other responses are not correct.

The nurse is describing how to perform a testicular self-examination to a patient. Which of these statements is most appropriate? A) "A good time to examine your testicles is just before you take a shower." B) "If you notice an enlarged testicle or a painless lump, call your health care provider." C) "The testicle is egg shaped and movable. It feels firm and has a lumpy consistency." D) "Perform a testicular exam at least once a week to detect the early stages of testicular cancer."

ANS: B If the patient notices a firm painless lump, a hard area, or an overall enlarged testicle, he should call his health care provider for further evaluation. The testicle normally feels rubbery with a smooth surface. A good time to examine the testicles is during the shower or bath, when one's hands are warm and soapy, and the scrotum is warm. It should be performed once a month.

When performing a scrotal assessment, the nurse notices that the scrotal contents transilluminate and show a red glow. On the basis of this finding the nurse would: A) assess the patient for the presence of a hernia. B) suspect the presence of serous fluid in the scrotum. C) consider this normal and proceed with the examination. D) refer the patient for evaluation of a mass in the scrotum.

ANS: B Normal scrotal contents do not transilluminate. Serous fluid does transilluminate and s

The nurse is palpating a female patient's adnexa. The findings include a firm, smooth uterine wall; the ovaries are palpable and feel smooth and firm. The fallopian tube is firm and pulsating. The nurse's most appropriate course of action would be to: A) tell the patient that her examination was normal. B) give her an immediate referral to a gynecologist. C) suggest that she return in a month for a recheck to verify the findings. D) tell the patient that she may have an ovarian cyst that should be evaluated further.

ANS: B Normally the uterine wall feels firm and smooth, with the contour of the fundus rounded. Ovaries are not often palpable, but when they are, they normally feel smooth, firm, and almond shaped and are highly movable, sliding through the fingers. The fallopian tube is not palpable normally. No other mass or pulsation should be felt. Pulsation or palpable fallopian tube suggests ectopic pregnancy, which warrants immediate referral

A 50-year-old woman calls the clinic because she has noticed some changes in her body and breasts and wonders if they could be due to the hormone replacement therapy (HRT) she started 3 months ago. The nurse should tell her: A) "Hormone replacement therapy is at such a low dose that side effects are very unusual." B) "Hormone replacement therapy has several side effects, including fluid retention, breast tenderness, and vaginal bleeding." C) "It would be very unusual to have vaginal bleeding with hormone replacement therapy, and I suggest you come in to the clinic immediately to have this evaluated." D) "It sounds as if your dose of estrogen is too high; I think you may need to decrease the amount you are taking and then call back in a week."

ANS: B Side effects of hormone replacement therapy include fluid retention, breast pain, and vaginal bleeding. The other responses are not correct.

The nurse is performing an examination of the anus and rectum. Which of these statements is correct and important to remember during this examination? A) The rectum is about 8 cm long. B) The anorectal junction cannot be palpated. C) Above the anal canal, the rectum turns anteriorly. D) There are no sensory nerves in the anal canal or rectum.

ANS: B The anal columns are folds of mucosa that extend vertically down from the rectum and end in the anorectal junction. This junction is not palpable, but it is visible on proctoscopy. The rectum is 12 cm long, and just above the anal canal, the rectum dilates and turns posteriorly.

40. The nurse is assessing a patient with possible cardiomyopathy and assesses the hepatojugular reflux. If heart failure is present, then the nurse should see which finding while pushing on the right upper quadrant of the patient's abdomen, just below the rib cage? A. The jugular veins will rise for a few seconds and then recede back to the previous level if the heart is working properly. B. The jugular veins will remain elevated as long as pressure on the abdomen is maintained. C. An impulse will be visible at the fourth or fifth intercostal space, at or inside the midclavicular line. D. The jugular veins will not be detected during this maneuver

ANS: B When performing hepatojugular reflux, the jugular veins will rise for a few seconds and then recede back to the previous level if the heart is able to pump the additional volume created by the pushing; however, with heart failure, the jugular veins remain elevated as long as pressure on the abdomen is maintained.

A mother has noticed that her son, who has been to a new babysitter, has some blisters and scabs on his face and buttocks. On examination, the nurse notices moist, thin-roofed vesicles with a thin erythematous base and suspects: a. Eczema. b. Impetigo. c. Herpes zoster. d. Diaper dermatitis.

ANS: B Impetigo is moist, thin-roofed vesicles with a thin erythematous base and is a contagious bacterial infection of the skin and most common in infants and children. Eczema is characterized by erythematous papules and vesicles with weeping, oozing, and crusts. Herpes zoster (i.e., chickenpox or varicella) is characterized by small, tight vesicles that are shiny with an erythematous base. Diaper dermatitis is characterized by red, moist maculopapular patches with poorly defined borders. DIF: Cognitive Level: Applying (Application) MSC: Client Needs: Physiologic Integrity: Physiologic Adaptation

The nurse is assessing the skin of a patient who has acquired immunodeficiency syndrome (AIDS) and notices multiple patchlike lesions on the temple and beard area that are faint pink in color. The nurse recognizes these lesions as: a. Measles (rubeola). b. Kaposis sarcoma. c. Angiomas. d. Herpes zoster

ANS: B Kaposis sarcoma is a vascular tumor that, in the early stages, appears as multiple, patchlike, faint pink lesions over the patients temple and beard areas. Measles is characterized by a red-purple maculopapular blotchy rash that appears on the third or fourth day of illness. The rash is first observed behind the ears, spreads over the face, and then spreads over the neck, trunk, arms, and legs. Cherry (senile) angiomas are small (1 to 5 mm), smooth, slightly raised bright red dots that commonly appear on the trunk in all adults over 30 years old. Herpes zoster causes vesicles up to 1 cm in size that are elevated with a cavity containing clear fluid. DIF: Cognitive Level: Applying (Application) MSC: Client Needs: Physiologic Integrity: Physiologic Adaptation

A few days after a summer hiking trip, a 25-year-old man comes to the clinic with a rash. On examination, the nurse notes that the rash is red, macular, with a bulls eye pattern across his midriff and behind his knees. The nurse suspects: a. Rubeola. b. Lyme disease. c. Allergy to mosquito bites. d. Rocky Mountain spotted fever.

ANS: B Lyme disease occurs in people who spend time outdoors in May through September. The first disease state exhibits the distinctive bulls eye and a red macular or papular rash that radiates from the site of the tick bite with some central clearing. The rash spreads 5 cm or larger, and is usually in the axilla, midriff, inguinal, or behind the knee, with regional lymphadenopathy. DIF: Cognitive Level: Analyzing (Analysis)

A 40-year-old woman reports a change in mole size, accompanied by color changes, itching, burning, and bleeding over the past month. She has a dark complexion and has no family history of skin cancer, but she has had many blistering sunburns in the past. The nurse would: a. Tell the patient to watch the lesion and report back in 2 months. b. Refer the patient because of the suggestion of melanoma on the basis of her symptoms. c. Ask additional questions regarding environmental irritants that may have caused this condition. d. Tell the patient that these signs suggest a compound nevus, which is very common in young to middle-aged adults.

ANS: B The ABCD danger signs of melanoma are asymmetry, border irregularity, color variation, and diameter. In addition, individuals may report a change in size, the development of itching, burning, and bleeding, or a new- pigmented lesion. Any one of these signs raises the suggestion of melanoma and warrants immediate referral. DIF: Cognitive Level: Analyzing (Analysis) MSC: Client Needs: Physiologic Integrity: Physiologic Adaptation

The nurse is assessing a patient who has liver disease for jaundice. Which of these assessment findings is indicative of true jaundice? a. Yellow patches in the outer sclera b. Yellow color of the sclera that extends up to the iris c. Skin that appears yellow when examined under low light d. Yellow deposits on the palms and soles of the feet where jaundice first appears

ANS: B The yellow sclera of jaundice extends up to the edge of the iris. Calluses on the palms and soles of the feet often appear yellow but are not classified as jaundice. Scleral jaundice should not be confused with the normal yellow subconjunctival fatty deposits that are common in the outer sclera of dark-skinned persons. DIF: Cognitive Level: Understanding (Comprehension) MSC: Client Needs: Physiologic Integrity: Physiologic Adaptation

An older adult woman is brought to the emergency department after being found lying on the kitchen floor for 2 days; she is extremely dehydrated. What would the nurse expect to see during the examination? a. Smooth mucous membranes and lips b. Dry mucous membranes and cracked lips c. Pale mucous membranes d. White patches on the mucous membranes

ANS: B With dehydration, mucous membranes appear dry and the lips look parched and cracked. The other responses are not found in dehydration. DIF: Cognitive Level: Applying (Application) MSC: Client Needs: Physiologic Integrity: Physiologic Adaptation

42. The nurse is presenting a class on risk factors for cardiovascular disease. Which of these are considered modifiable risk factors for myocardial infarction (MI)? Select all that apply. A. Ethnicity B. Abnormal lipids C. Smoking D. Gender E. Hypertension F. Diabetes G. Family history

ANS: B, C, E, F. Nine modifiable risk factors for MI, as identified by a recent study, include abnormal lipids, smoking, hypertension, diabetes, abdominal obesity, psychosocial factors, consumption of fruits and vegetables, alcohol use, and regular physical activity.

The nurse is palpating an ovarian mass during an internal examination of a 63-year-old woman. Which findings of the mass's characteristics would suggest the presence of an ovarian cyst? Select all that apply. A) Heavy and solid B) Mobile and fluctuant C) Mobile and solid D) Fixed E) Smooth and round F) Poorly defined

ANS: B, E An ovarian cyst (fluctuant ovarian mass) is usually asymptomatic, and would feel like a smooth, round, fluctuant, mobile, nontender mass on the ovary. A mass that is heavy, solid, fixed, and poorly defined suggests malignancy. A benign mass may feel mobile and solid.

A patient has been admitted to a hospital after the staff in the nursing home noticed a pressure ulcer in his sacral area. The nurse examines the pressure ulcer and determines that it is a stage II ulcer. Which of these findings are characteristic of a stage II pressure ulcer? Select all that apply. a. Intact skin appears red but is not broken. b. Partial thickness skin erosion is observed with a loss of epidermis or dermis. c. Ulcer extends into the subcutaneous tissue. d. Localized redness in light skin will blanch with fingertip pressure. e. Open blister areas have a red-pink wound bed. f. Patches of eschar cover parts of the wound.

ANS: B, E Stage I pressure ulcers have intact skin that appears red but is not broken, and localized redness in intact skin will blanche with fingertip pressure. Stage II pressure ulcers have partial thickness skin erosion with a loss of epidermis or also the dermis; open blisters have a red-pink wound bed. Stage III pressure ulcers are full thickness, extending into the subcutaneous tissue; subcutaneous fat may be seen but not muscle, bone, or tendon. Stage IV pressure ulcers involve all skin layers and extend into supporting tissue, exposing muscle, bone, and tendon. Slough (stringy matter attached to the wound bed) or eschar (black or brown necrotic tissue) may be present. DIF: Cognitive Level: Applying (Application) MSC: Client Needs: Physiologic Integrity: Physiologic Adaptation

When assessing tactile fremitus, the nurse recalls that it is normal to feel tactile fremitus most intensely over which location? a. Between the scapulae b. Third intercostal space, MCL c. Fifth intercostal space, midaxillary line (MAL) d. Over the lower lobes, posterior side

ANS: Between the scapulae Normally, fremitus is most prominent between the scapulae and around the sternum. These are sites where the major bronchi are closest to the chest wall. Fremitus normally decreases as one progress down the chest because more tissue impedes sound transmission.

35. A cardiovascular assessment, the nurse knows that an S4 heart sound is: A. Heard at the onset of atrial diastole. B. Usually a normal finding in the elderly. C. Heard at the end of ventricular diastole. D. Heard best over the second left intercostal space with the individual sitting upright

ANS: C An S4 heart sound is heard at the end of diastole when the atria contract (atrial systole) and when the ventricles are resistant to filling. The S4 occurs just before the S1.

31. The nurse is taking an initial blood pressure reading on a 72-year-old patient with documented hypertension. How should the nurse proceed? a. Cuff should be placed on the patients arm and inflated 30 mm Hg above the patients pulse rate. b. Cuff should be inflated to 200 mm Hg in an attempt to obtain the most accurate systolic reading. c. Cuff should be inflated 30 mm Hg above the point at which the palpated pulse disappears. d. After confirming the patients previous blood pressure readings, the cuff should be inflated 30 mm Hg above the highest systolic reading recorded.

ANS: C An auscultatory gap occurs in approximately 5% of the people, most often in those with hypertension. To check for the presence of an auscultatory gap, the cuff should be inflated 20 to 30 mm Hg beyond the point at which the palpated pulse disappears.

6. The nurse is assessing an 80-year-old male patient. Which assessment findings would be considered normal? a. Increase in body weight from his younger years b. Additional deposits of fat on the thighs and lower legs c. Presence of kyphosis and flexion in the knees and hips d. Change in overall body proportion, including a longer trunk and shorter extremities

ANS: C Changes that occur in the aging person include more prominent bony landmarks, decreased body weight (especially in men), a decrease in subcutaneous fat from the face and periphery, and additional fat deposited on the abdomen and hips. Postural changes of kyphosis and slight flexion in the knees and hips also occur.

50. During an examination, the nurse notices that a female patient has a round moon face, central trunk obesity, and a cervical hump. Her skin is fragile with bruises. The nurse determines that the patient has which condition? a. Marfan syndrome b. Gigantism c. Cushing syndrome d. Acromegaly

ANS: C Cushing syndrome is characterized by weight gain and edema with central trunk and cervical obesity (buffalo hump) and round plethoric face (moon face). Excessive catabolism causes muscle wasting; weakness; thin arms and legs; reduced height; and thin, fragile skin with purple abdominal striae, bruising, and acne. (See Table 9-5, Abnormalities in Body Height and Proportion, for the definitions of the other conditions.)

5. Which of these statements describes the closure of the valves in a normal cardiac cycle? A. The aortic valve closes slightly before the tricuspid valve. B. The pulmonic valve closes slightly before the aortic valve. C. The tricuspid valve closes slightly later than the mitral valve. D. Both the tricuspid and pulmonic valves close at the same time.

ANS: C Events occur just slightly later in the right side of the heart because of the route of myocardial depolarization. As a result, two distinct components to each of the heart sounds exist, and sometimes they can be heard separately. In the first heart sound, the mitral component (M1) closes just before the tricuspid component (T1).

40. In a patient with acromegaly, the nurse will expect to discover which assessment findings? a. Heavy, flattened facial features b. Growth retardation and a delayed onset of puberty c. Overgrowth of bone in the face, head, hands, and feet d. Increased height and weight and delayed sexual development

ANS: C Excessive secretions of growth hormone in adulthood after normal completion of body growth causes an overgrowth of the bones in the face, head, hands, and feet but no change in height.

9. When assessing a newborn infant who is 5 minutes old, the nurse knows that which of these statements would be true? A. The left ventricle is larger and weighs more than the right ventricle. B. The circulation of a newborn is identical to that of an adult. C. There is an opening in the atrial septum where blood can flow into the left side of the heart. D. The foramen ovale closes just minutes before birth and the ductus arteriosus closes immediately after.

ANS: C First, about two thirds of the blood is shunted through an opening in the atrial septum, the foramen ovale into the left side of the heart, where it is pumped out through the aorta. The foramen ovale closes within the first hour because the pressure in the right side of the heart is now lower than in the left side.

13. In assessing a patient's major risk factors for heart disease, which would the nurse want to include when taking a history? A. Family history, hypertension, stress, age B. Personality type, high cholesterol, diabetes, smoking C. Smoking, hypertension, obesity, diabetes, high cholesterol D. Alcohol consumption, obesity, diabetes, stress, high cholesterol

ANS: C For major risk factors for coronary artery disease, collect data regarding elevated serum cholesterol, elevated blood pressure, blood glucose levels above 130 mg/dL or known diabetes mellitus, obesity, cigarette smoking, low activity level.

43. What type of blood pressure measurement error is most likely to occur if the nurse does not check for the presence of an auscultatory gap? a. Diastolic blood pressure may not be heard. b. Diastolic blood pressure may be falsely low. c. Systolic blood pressure may be falsely low. d. Systolic blood pressure may be falsely high.

ANS: C If an auscultatory gap is undetected, then a falsely low systolic or falsely high diastolic reading may result, which is common in patients with hypertension.

34. A patient is seen in the clinic for complaints of fainting episodes that started last week. How should the nurse proceed with the examination? a. Blood pressure readings are taken in both the arms and the thighs. b. The patient is assisted to a lying position, and his blood pressure is taken. c. His blood pressure is recorded in the lying, sitting, and standing positions. d. His blood pressure is recorded in the lying and sitting positions; these numbers are then averaged to obtain a mean blood pressure.

ANS: C If the person is known to have hypertension, is taking antihypertensive medications, or reports a history of fainting or syncope, then the blood pressure reading should be taken in three positions: lying, sitting, and standing.

29. The nurse is performing a cardiac assessment on a 65-year-old patient 3 days after her myocardial infarction. Heart sounds are normal when she is supine, but when she is sitting and leaning forward, the nurse hears a high-pitched, scratchy sound with the diaphragm of the stethoscope at the apex. It disappears on inspiration. The nurse suspects: A. Increased cardiac output. B. Another myocardial infarction. C. Inflammation of the precordium. D. Ventricular hypertrophy resulting from muscle damage.

ANS: C Inflammation of the precordium gives rise to a friction rub. The sound is high pitched and scratchy, like sandpaper being rubbed. It is best heard with the diaphragm of the stethoscope, with the person sitting up and leaning forward, and with the breath held in expiration. A friction rub can be heard any place on the precordium but usually is best heard at the apex and left lower sternal border, which are places where the pericardium comes in close contact with the chest wall.

19. The nurse is examining a patient who has possible cardiac enlargement. Which statement about percussion of the heart is true? A. Percussion is a useful tool for outlining the heart's borders. B. Percussion is easier in obese patients. C. Studies show that percussed cardiac borders do not correlate well with the true cardiac border D. Only expert health care providers should attempt percussion of the heart.

ANS: C Numerous comparison studies have shown that the percussed cardiac border correlates "only moderately" with the true cardiac border. Percussion is of limited usefulness with the female breast tissue or in an obese person, or a person with a muscular chest wall. Chest x-rays or echocardiographicD examinations are much more accurate in detecting heart enlargement.

26. During the precordial assessment on an patient who is 8 months pregnant, the nurse palpates the apical impulse at the fourth left intercostal space lateral to the midclavicular line. This finding would indicate: A. Right ventricular hypertrophy. B. Increased volume and size of the heart as a result of pregnancy. C. Displacement of the heart from elevation of the diaphragm. D. Increased blood flow through the internal mammary artery.

ANS: C Palpation of the apical impulse is higher and more lateral compared with the normal position because the enlarging uterus elevates the diaphragm and displaces the heart up and to the left and rotates it on its long axis.

12. A 45-year-old man is in the clinic for a routine physical. During the history the patient states he's been having difficulty sleeping. "I'll be sleeping great and then I wake up and feel like I can't get my breath." The nurse's best response to this would be: A. "When was your last electrocardiogram?" B. "It's probably because it's been so hot at night." C. "Do you have any history of problems with your heart?" D. "Have you had a recent sinus infection or upper respiratory infection?"

ANS: C Paroxysmal nocturnal dyspnea occurs with heart failure. Lying down increases volume of intrathoracic blood, and the weakened heart cannot accommodate the increased load. Classically, the person awakens after 2 hours of sleep, arises, and flings open a window with the perception of needing fresh air.

7. The nurse should measure rectal temperatures in which of these patients? a. School-age child b. Older adult c. Comatose adult d. Patient receiving oxygen by nasal cannula

ANS: C Rectal temperatures should be taken when the other routes are impractical, such as for comatose or confused persons, for those in shock, or for those who cannot close the mouth because of breathing or oxygen tubes, a wired mandible, or other facial dysfunctions.

22. When listening to heart sounds, the nurse knows that S1: A. Is louder than S2 at the base of the heart. B. Indicates the beginning of diastole. C. Coincides with the carotid artery pulse. D. Is caused by closure of the semilunar valves

ANS: C S1 coincides with the carotid artery pulse. S1 is the start of systole and is louder than S2 at the apex of the heart; S2 is louder than S1 at the base. The nurse should feel the carotid artery pulse gently while auscultating at the apex; the sound heard as each pulse is felt is S1.

24. Which of these findings would the nurse expect to notice during a cardiac assessment on a 4-year-old child? A. S3 when sitting up B. Persistent tachycardia above 150 C. Murmur at second left intercostal space when supine D. Palpable apical impulse in fifth left intercostal space lateral to midclavicular line

ANS: C Some murmurs are common in healthy children or adolescents and are termed innocent or functional. The innocent murmur is heard at the second or third left intercostal space and disappears with sitting, and the young person has no associated signs of cardiac dysfunction.

7. The electrical stimulus of the cardiac cycle follows which sequence? A. AV node SA node bundle of His B. Bundle of His AV node SA node C. SA node, AV node, bundle of His, bundle branches D. AV node, SA node, bundle of His, bundle branches

ANS: C Specialized cells in the SA node near the superior vena cava initiate an electrical impulse. The current flows in an orderly sequence, first across the atria to the AV node low in the atrial septum. There it is delayed slightly so that the atria have time to contract before the ventricles are stimulated. Then the impulse travels to the bundle of His, the right and left bundle branches, and then through the ventricles.

17. The nurse is taking temperatures in a clinic with a TMT. Which statement is true regarding use of the TMT? a. A tympanic temperature is more time consuming than a rectal temperature. b. The tympanic method is more invasive and uncomfortable than the oral method. c. The risk of cross-contamination is reduced, compared with the rectal route. d. The tympanic membrane most accurately reflects the temperature in the ophthalmic artery.

ANS: C The TMT is a noninvasive, nontraumatic device that is extremely quick and efficient. The chance of cross-contamination with the TMT is minimal because the ear canal is lined with skin, not mucous membranes.

36. The nurse is assessing a patient's apical impulse. Which of these statements is true regarding the apical impulse? A. It is palpable in all adults. B. It occurs with the onset of diastole. C. Its location may be indicative of heart size. D. It should normally be palpable in the anterior axillary line

ANS: C The apical impulse is palpable in about 50% of adults. It is located in the fifth left intercostal space in the midclavicular line. Horizontal or downward displacement of the apical impulse may indicate an enlargement of the left ventricle.

22. When assessing the force, or strength, of a pulse, the nurse recalls that the pulse: a. Is usually recorded on a 0- to 2-point scale. b. Demonstrates elasticity of the vessel wall. c. Is a reflection of the hearts stroke volume. d. Reflects the blood volume in the arteries during diastole.

ANS: C The heart pumps an amount of blood (the stroke volume) into the aorta. The force flares the arterial walls and generates a pressure wave, which is felt in the periphery as the pulse.

38. When the nurse is auscultating the carotid artery for bruits, which of these statements reflects correct technique? A. While listening with the bell of the stethoscope, have the patient take a deep breath and hold it. B. While auscultating one side with the bell of the stethoscope, palpate the carotid artery on the other side to check pulsations. C. Lightly apply the bell of the stethoscope over the carotid artery, and while listening, have the patient take a breath, exhale, and hold it briefly. D. Firmly place the bell of the stethoscope over the carotid artery, and while listening,have the patient take a breath, exhale, and hold it briefly

ANS: C The nurse should lightly apply the bell of the stethoscope over the carotid artery at three levels; while listening, the nurse should have the patient take a breath, exhale, and hold it briefly. Holding the breath on inhalation will also tense the levator scapulae muscles, which makes it hard to hear the carotids. Examine only one carotid artery at a time to avoid compromising arterial blood flow to the brain. Avoid pressure over the carotid sinus, which may lead to decreased heart rate, decreased blood pressure, and cerebral ischemia with syncope.

21. While counting the apical pulse on a 16-year-old patient, the nurse notices an irregular rhythm. His rate speeds up on inspiration and slows on expiration. What would be the nurse's response? A. Talk with the patient about his intake of caffeine. B. Perform an electrocardiogram after the examination. C. No further response is needed because this is normal. D. Refer the patient to a cardiologist for further testing

ANS: C The rhythm should be regular, although sinus arrhythmia occurs normally in young adults and children. With sinus arrhythmia, the rhythm varies with the person's breathing, increasing at the peak of inspiration, and slowing with expiration.

4. When listening to heart sounds, the nurse knows that the valve closures that can be heard best at the base of the heart are: A. Mitral and tricuspid. B. Tricuspid and aortic. C. Aortic and pulmonic. D. Mitral and pulmonic

ANS: C The second heart sound (S2) occurs with closure of the semilunar (aortic and pulmonic) valves and signals the end of systole. Although it is heard over all the precordium, S2 is loudest at the base of the heart.

The nurse is assessing for inflammation in a dark-skinned person. Which technique is the best? a. Assessing the skin for cyanosis and swelling b. Assessing the oral mucosa for generalized erythema c. Palpating the skin for edema and increased warmth d. Palpating for tenderness and local areas of ecchymosis

ANS: C Because inflammation cannot be seen in dark-skinned persons, palpating the skin for increased warmth, for taut or tightly pulled surfaces that may be indicative of edema, and for a hardening of deep tissues or blood vessels is often necessary.

A patient who is visiting the clinic complains of having "stomach pains for 2 weeks" and describes his stools as being "soft and black" for about the last 10 days. He denies taking any medications. The nurse is aware that these symptoms are most indicative of: A) excessive fat caused by malabsorption. B) increased iron intake resulting from a change in diet. C) occult blood resulting from gastrointestinal bleeding. D) absent bile pigment from liver problems.

ANS: C Black stools may be tarry due to occult blood (melena) from gastrointestinal bleeding or nontarry from ingestion of iron medications (not diet). Excessive fat causes the stool to become frothy; absence of bile pigment causes clay-colored stools.

The nurse is performing a genitourinary assessment on a 50-year-old obese male laborer. On examination the nurse notices a painless round swelling close to the pubis in the area of the internal inguinal ring that is easily reduced when the individual is supine. These findings are most consistent with a(n) _____ hernia. A) scrotal B) femoral C) direct inguinal D) indirect inguinal

ANS: C Direct inguinal hernias occur most often in men over the age of 40 years. It is an acquired weakness brought on by heavy lifting, obesity, chronic cough, or ascites. The direct inguinal hernia is usually a painless, round swelling close to the pubis in the area of the internal inguinal ring that is easily reduced when the individual is supine. See Table 24-6 for a description of scrotal hernia. See Table 24-7 for descriptions of femoral hernias and indirect inguinal hernias.

During an external genitalia examination of a woman, the nurse notices several lesions around the vulva. The lesions are pink, moist, soft, and pointed papules. The patient states that she is not aware of any problems in that area. The nurse recognizes that these lesions may be: A) syphilitic chancre. B) herpes simplex virus type 2 (herpes genitalis). C) human papillomavirus (HPV), or genital warts. D) pediculosis pubis (crab lice).

ANS: C HPV lesions are painless, warty growths that the woman may not notice. Lesions are pink or flesh colored, soft, pointed, moist, warty papules that occur in single or multiple cauliflower-like patches around the vulva, introitus, anus, vagina, or cervix. Herpetic lesions are painful clusters of small, shallow vesicles with surrounding erythema. Syphilitic chancres begin as a solitary silvery papule that erodes to a red, round or oval, superficial ulcer with a yellowish discharge. Pediculosis pubis causes severe perineal itching and excoriations and erythematous areas. See Table 26-2.

During the interview with a female patient, the nurse gathers data that indicate that the patient is perimenopausal. Which of these statements made by this patient leads to this conclusion? A) "I have noticed that my muscles ache at night when I go to bed." B) "I will be very happy when I can stop worrying about having a period." C) "I have been noticing that I sweat a lot more than I used to, especially at night." D) "I have only been pregnant twice, but both times I had breast tenderness as my first symptom."

ANS: C Hormone shifts occur during the perimenopausal period, and associated symptoms of menopause may occur, such as hot flashes, night sweats, numbness and tingling, headache, palpitations, drenching sweats, mood swings, vaginal dryness, and itching. The other responses are not correct.

A 2-month-old uncircumcised infant has been brought to the clinic for a well-baby checkup. How would the nurse proceed with the genital examination? A) Elicit the cremasteric reflex. B) Assess the glans for redness or lesions. C) Avoid retracting the foreskin until the infant is 3 months old. D) Note any dirt or smegma that has collected under the foreskin.

ANS: C If uncircumcised, then the foreskin is normally tight during the first 3 months and should not be retracted because of the risk of tearing the membrane attaching the foreskin to the shaft. The other options are not correct.

A 35-year-old pregnant woman comes to the clinic for a monthly appointment. During the assessment, the nurse notices that she has a brown patch of hyperpigmentation on her face. The nurse continues the skin assessment aware that another finding may be: a. Keratoses. b. Xerosis. c. Chloasma. d. Acrochordons.

ANS: C In pregnancy, skin changes can include striae, linea nigra (a brownish-black line down the midline), chloasma (brown patches of hyperpigmentation), and vascular spiders. Keratoses are raised, thickened areas of pigmentation that look crusted, scaly, and warty. Xerosis is dry skin. Acrochordons, or skin tags, occur more often in the aging adult.

During an examination of an aging male, the nurse recognizes that normal changes to expect would be: A) enlarged scrotal sac. B) increased pubic hair. C) decreased penis size. D) increased rugae over the scrotum.

ANS: C In the aging male the amount of pubic hair decreases, the penis size decreases, and there is a decrease in the rugae over the scrotal sac. The scrotal sac does not enlarge.

The nurse is aware that which of these statements is true regarding the incidence of testicular cancer? A) Testicular cancer is the most common cancer in men aged 30 to 50 years. B) The early symptoms of testicular cancer are pain and induration. C) Men with a history of cryptorchidism are at greatest risk for development of testicular cancer. D) The cure rate for testicular cancer is low.

ANS: C Men with undescended testicles (cryptorchidism) are at greatest risk for development of testicular cancer. The overall incidence of testicular cancer is rare. Testicular cancer has no early symptoms. When detected early and treated before metastasis, the cure rate is almost 100%.

A 2-year-old boy has been diagnosed with "physiologic cryptorchidism." Given this diagnosis, during assessment the nurse will most likely observe: A) testes that are hard and painful to palpation. B) an atrophic scrotum and absence of the testis bilaterally. C) an absence of the testis in the scrotum, but the testis can be milked down. D) testes that migrate into the abdomen when the child squats or sits cross-legged.

ANS: C Migratory testes (physiologic cryptorchidism) are common because of the strength of the cremasteric reflex and the small mass of the prepubertal testes. The affected side has a normally developed scrotum and the testis can be milked down. The other responses are not correct.

A 45-year-old mother of two children is seen at the clinic for complaints of "losing my urine when I sneeze." The nurse documents that she is experiencing: A) urinary frequency. B) enuresis. C) stress incontinence. D) urge incontinence.

ANS: C Stress incontinence is involuntary urine loss with physical strain, sneezing, or coughing that occurs due to weakness of the pelvic floor. Urinary frequency is urinating more times than usual (more than 5 to 6 times per day). Enuresis is involuntary passage of urine at night after age 5 to 6 years (bed wetting). Urge incontinence is involuntary urine loss from overactive detrusor muscle in the bladder. It contracts, causing an urgent need to void.

When the nurse is performing a testicular examination on a 25-year-old man, which of these findings is considered normal? A) Nontender subcutaneous plaques B) A scrotal area that is dry, scaly, and nodular C) Testes that feel oval and movable and are slightly sensitive to compression D) A single, hard, circumscribed, movable mass, less than 1 cm under the surface of the testes

ANS: C Testes normally feel oval, firm and rubbery, smooth, and equal bilaterally and are freely movable and slightly tender to moderate pressure. The scrotal skin should not be dry, scaly, or nodular or contain subcutaneous plaques. Any mass would be an abnormal finding.

A 13-year-old girl is interested in obtaining information about the cause of her acne. The nurse should share with her that acne: a. Is contagious. b. Has no known cause. c. Is caused by increased sebum production. d. Has been found to be related to poor hygiene

ANS: C Approximately 90% of males and 80% of females will develop acne; causes are increased sebum production and epithelial cells that do not desquamate normally. DIF: Cognitive Level: Understanding (Comprehension) MSC: Client Needs: Health Promotion and Maintenance

The nurse is assessing for inflammation in a dark-skinned person. Which technique is the best? a. Assessing the skin for cyanosis and swelling b. Assessing the oral mucosa for generalized erythema c. Palpating the skin for edema and increased warmth d. Palpating for tenderness and local areas of ecchymosis

ANS: C Because inflammation cannot be seen in dark-skinned persons, palpating the skin for increased warmth, for taut or tightly pulled surfaces that may be indicative of edema, and for a hardening of deep tissues or blood vessels is often necessary. DIF: Cognitive Level: Applying (Application) MSC: Client Needs: Physiologic Integrity: Physiologic Adaptation

A 42-year-old woman complains that she has noticed several small, slightly raised, bright red dots on her chest. On examination, the nurse expects that the spots are probably: a. Anasarca. b. Scleroderma. c. Senile angiomas. d. Latent myeloma.

ANS: C Cherry (senile) angiomas are small, smooth, slightly raised bright red dots that commonly appear on the trunk of adults over 30 years old. DIF: Cognitive Level: Applying (Application) MSC: Client Needs: Physiologic Integrity: Physiologic Adaptation

A patient comes to the clinic and tells the nurse that he has been confined to his recliner chair for approximately 3 days with his feet down and he asks the nurse to evaluate his feet. During the assessment, the nurse might expect to find: a. Pallor b. Coolness c. Distended veins d. Prolonged capillary filling time

ANS: C Keeping the feet in a dependent position causes venous pooling, resulting in redness, warmth, and distended veins. Prolonged elevation would cause pallor and coolness. Immobilization or prolonged inactivity would cause prolonged capillary filling time. DIF: Cognitive Level: Applying (Application) MSC: Client Needs: Physiologic Integrity: Physiologic Adaptation

A 70-year-old woman who loves to garden has small, flat, brown macules over her arms and hands. She asks, What causes these liver spots? The nurse tells her, They are: a. Signs of decreased hematocrit related to anemia. b. Due to the destruction of melanin in your skin from exposure to the sun. c. Clusters of melanocytes that appear after extensive sun exposure. d. Areas of hyperpigmentation related to decreased perfusion and vasoconstriction.

ANS: C Liver spots, or senile lentigines, are clusters of melanocytes that appear on the forearms and dorsa of the hands after extensive sun exposure. The other responses are not correct. DIF: Cognitive Level: Understanding (Comprehension) MSC: Client Needs: Physiologic Integrity: Physiologic Adaptation

A physician has diagnosed a patient with purpura. After leaving the room, a nursing student asks the nurse what the physician saw that led to that diagnosis. The nurse should say, The physician is referring to the: a. Blue dilation of blood vessels in a star-shaped linear pattern on the legs. b. Fiery red, star-shaped marking on the cheek that has a solid circular center. c. Confluent and extensive patch of petechiae and ecchymoses on the feet. d. Tiny areas of hemorrhage that are less than 2 mm, round, discrete, and dark red in color.

ANS: C Purpura is a confluent and extensive patch of petechiae and ecchymoses and a flat macular hemorrhage observed in generalized disorders such as thrombocytopenia and scurvy. The blue dilation of blood vessels in a star-shaped linear pattern on the legs describes a venous lake. The fiery red, star-shaped marking on the cheek that has a solid circular center describes a spider or star angioma. The tiny areas of hemorrhage that are less than 2 mm, round, discrete, and dark red in color describes petechiae. DIF: Cognitive Level: Understanding (Comprehension) MSC: Client Needs: Physiologic Integrity: Physiologic Adaptation

A newborn infant is in the clinic for a well-baby checkup. The nurse observes the infant for the possibility of fluid loss because of which of these factors? a. Subcutaneous fat deposits are high in the newborn. b. Sebaceous glands are overproductive in the newborn. c. The newborns skin is more permeable than that of the adult. d. The amount of vernix caseosa dramatically rises in the newborn.

ANS: C The newborns skin is thin, smooth, and elastic and is relatively more permeable than that of the adult; consequently, the infant is at greater risk for fluid loss. The subcutaneous layer in the infant is inefficient, not thick, and the sebaceous glands are present but decrease in size and production. Vernix caseosa is not produced after birth. DIF: Cognitive Level: Applying (Application) MSC: Client Needs: Health Promotion and Maintenance

A patient is especially worried about an area of skin on her feet that has turned white. The health care provider has told her that her condition is vitiligo. The nurse explains to her that vitiligo is: a. Caused by an excess of melanin pigment b. Caused by an excess of apocrine glands in her feet c. Caused by the complete absence of melanin pigment d. Related to impetigo and can be treated with an ointment

ANS: C Vitiligo is the complete absence of melanin pigment in patchy areas of white or light skin on the face, neck, hands, feet, body folds, and around orifices otherwise, the depigmented skin is normal. DIF: Cognitive Level: Applying (Application) MSC: Client Needs: Physiologic Integrity: Physiologic Adaptation

A patient states, "I can hear a crunching or grating sound when I kneel." She also states that "it is very difficult to get out of bed in the morning because of stiffness and pain in my joints." The nurse should assess for signs of what problem? A) Crepitation B) A bone spur C) A loose tendon D) Fluid in the knee joint

ANS: Crepitation Crepitation is an audible and palpable crunching or grating that accompanies movement and occurs when articular surfaces in the joints are roughened, as with rheumatoid arthritis. The other options are not correct.

23. During the cardiac auscultation the nurse hears a sound occurring immediately after S2 at the second left intercostal space. To further assess this sound, what should the nurse do? A. Have the patient turn to the left side while the nurse listens with the bell. B. Ask the patient to hold his breath while the nurse listens again. C. No further assessment is needed because the nurse knows it is an S3. D. Watch the patient's respirations while listening for effect on the sound

ANS: D A split S2 is a normal phenomenon that occurs toward the end of inspiration in some people. A split S2 is heard only in the pulmonic valve area, the second left interspace. When the split S2 is first heard, the nurse should not be tempted to ask the person to hold his or her breath so that the nurse can concentrate on the sounds. Breath holding will only equalize ejection times in the right and left sides of the heart and cause the split to go away. Instead, the nurse should concentrate on the split while watching the person's chest rise up and down with breathing.

8. The findings from an assessment of a 70-year-old patient with swelling in his ankles include jugular venous pulsations 5 cm above the sternal angle when the head of his bed is elevated 45 degrees. The nurse knows that this finding indicates: A. Decreased fluid volume. B. Increased cardiac output. C. Narrowing of jugular veins. D. Elevated pressure related to heart failure.

ANS: D Because no cardiac valve exists to separate the superior vena cava from the right atrium, the jugular veins give information about activity on the right side of the heart. They reflect filling pressures and volume changes. Normal jugular venous pulsation is 2 cm or less above the sternal angle. Elevated pressure is more than 3 cm above the sternal angle at 45 degrees and occurs with heart failure.

The nurse is reviewing an assessment of a patient's peripheral pulses and notices that the documentation states that the radial pulses are "2+." The nurse recognizes that this reading indicates what type of pulse? a. Bounding b. Normal c. Weak d. Absent

ANS: Normal When documenting the force, or amplitude, of pulses, 3+ indicates an increased, full, or bounding pulse, 2+ indicates a normal pulse, 1+ indicates a weak pulse, and 0 indicates an absent pulse.

37. During an assessment, the nurse notes that the patient's apical impulse is displaced laterally, and it is palpable over a wide area. This indicates: A. Systemic hypertension. B. Pulmonic hypertension. C. Pressure overload, as in aortic stenosis. D. Volume overload, as in mitral regurgitation

ANS: D Cardiac enlargement displaces the apical impulse laterally and over a wider area when left ventricular hypertrophy and dilatation are present. This is volume overload, as in mitral regurgitation, aortic regurgitation, or left-to-right shunts.

32. During a cardiac assessment on a 38 year-old patient in the hospital for "chest pain," the nurse finds the following: jugular vein pulsations 4 cm above sternal angle when he is elevated at 45 degrees, blood pressure 98/60 mm Hg, heart rate 130 beats per minute, ankle edema, difficulty in breathing when supine, and an S3 on auscultation. Which of these conditions best explains the cause of these findings? A. Fluid overload B. Atrial septal defect C. Myocardial infarction D. Heart failure

ANS: D Heart failure causes decreased cardiac output when the heart fails as a pump and the circulation becomes backed up and congested. Signs and symptoms include dyspnea, orthopnea, paroxysmal nocturnal dyspnea, decreased blood pressure, dependent and pitting edema; anxiety; confusion; jugular vein distention; and fatigue. The S3 is associated with heart failure and is always abnormal after age 35. The S3 may be the earliest sign of heart failure.

31. A 30-year-old woman with a history of mitral valve problems states that she has been "very tired." She has started waking up at night and feels like her "heart is pounding." During the assessment, the nurse palpates a thrill and lift at the fifth left intercostal space midclavicular line. In the same area the nurse also auscultates a blowing, swishing sound right after S1. These findings would be most consistent with: A. Heart failure. B. Aortic stenosis. C. Pulmonary edema. D. Mitral regurgitation

ANS: D Mitral regurgitation subjective findings include fatigue, palpitation, and orthopnea. Objective findings are (1) a thrill in systole at apex, (2) lift at apex, (3) apical impulse displaced down and to the left, (4) S1 diminished, S2 accentuated, S3 at apex often present, and (5) murmur: pansystolic, often loud, blowing, best heard at apex, radiating well to the left axilla.

17. During inspection of the precordium of an adult patient, the nurse notices the chest moving in a forceful manner along the sternal border. This finding most likely suggests: A. A normal heart. B. A systolic murmur. C. Enlargement of the left ventricle. D. Enlargement of the right ventricle

ANS: D Normally, the examiner may or may not see an apical impulse; when visible, it occupies the fourth or fifth intercostal space at or inside the midclavicular line. A heave or lift is a sustained forceful thrusting of the ventricle during systole. It occurs with ventricular hypertrophy as a result of increased workload. A right ventricular heave is seen at the sternal border; a left ventricular heave is seen at the apex.

When using a Doppler ultrasonic stethoscope, the nurse recognizes venous flow when which sound is heard? a. Low humming sound b. Regular "lub, dub" pattern c. Swishing, whooshing sound d. Steady, even, flowing sound

ANS: Swishing, whooshing sound When using the Doppler ultrasonic stethoscope, the pulse site is found when one hears a swishing, whooshing sound.

18. During an assessment of a healthy adult, where would the nurse expect to palpate the apical impulse? A. Third left intercostal space at the midclavicular line B. Fourth left intercostal space at the sternal border C. Fourth left intercostal space at the anterior axillary line D. Fifth left intercostal space at the midclavicular line

ANS: D The apical impulse should occupy only one intercostal space, the fourth or fifth, and it should be at or medial to the midclavicular line.

A 13-year-old girl is visiting the clinic for a sports physical. The nurse should remember to include which of these tests in the examination? A) Test for occult blood B) The Valsalva maneuver C) Internal palpation of the anus D) Inspection of the perianal area

ANS: D Inspect the perianal region of the school-aged child and adolescent during examination of the genitalia. Internal palpation is not performed routinely at this age. Testing for occult blood and doing the Valsalva maneuver are also not necessary.

14. The mother of a 3-month-old infant states that her baby has not been gaining weight. With further questioning, the nurse finds that the infant falls asleep after nursing and wakes up after a short amount of time, hungry again. What other information would the nurse want to have? A. The infant's sleeping position B. Sibling history of eating disorders C. Amount of background noise when eating D. Presence of dyspnea or diaphoresis when sucking

ANS: D To screen for heart disease in an infant, focus on feeding. Note fatigue during feeding. An infant with heart failure takes fewer ounces each feeding, becomes dyspneic with sucking, may be diaphoretic and then falls into exhausted sleep and awakens after a short time hungry again.

41. The nurse is performing a general survey of a patient. Which finding is considered normal? a. When standing, the patients base is narrow. b. The patient appears older than his stated age. c. Arm span (fingertip to fingertip) is greater than the height. d. Arm span (fingertip to fingertip) equals the patients height.

ANS: D When performing the general survey, the patients arm span (fingertip to fingertip) should equal the patients height. An arm span that is greater than the persons height may indicate Marfan syndrome. The base should be wide when the patient is standing, and an older appearance than the stated age may indicate a history of a chronic illness or chronic alcoholism.

The nurse knows that a common assessment finding in a boy younger than 2 years old is: A) an inflamed and tender spermatic cord. B) the presence of a hernia in the scrotum. C) a penis that looks large in relation to the scrotum. D) the presence of a hydrocele, or fluid in the scrotum.

ANS: D A common scrotal finding in boys younger than 2 years of age is a hydrocele, or fluid in the scrotum. The other options are not correct.

While performing a rectal examination, the nurse notices a firm, irregularly shaped mass. What should the nurse do next? A) Continue with the examination and document the finding in the chart. B) Instruct patient to return for repeat assessment in 1 month. C) Tell the patient that a mass was felt but it is nothing to worry about. D) Report the finding and refer the patient to a specialist for further examination.

ANS: D A firm or hard mass with irregular shape or rolled edges may signify carcinoma. Promptly report any mass that is discovered for further examination. The other responses are not correct

A 62-year-old man states that his doctor told him that he has an "inguinal hernia." He asks the nurse to explain what a hernia is. The nurse should: A) tell him not to worry and that most men his age develop hernias. B) explain that a hernia is often the result of prenatal growth abnormalities. C) refer him to his physician for additional consultation because the physician made the initial diagnosis. D) explain that a hernia is a loop of bowel protruding through a weak spot in the abdominal muscles.

ANS: D A hernia is a loop of bowel protruding through a weak spot in the musculature. The other options are not correct responses to the patient's question.

During an assessment of a 20-year-old man, the nurse finds a small palpable lesion with a tuft of hair located directly over the coccyx. The nurse knows that this lesion would most likely be a: A) polyp. B) pruritus ani. C) carcinoma. D) pilonidal cyst.

ANS: D A pilonidal cyst or sinus is a hair-containing cyst or sinus located in the midline over the coccyx or lower sacrum. It often opens as a dimple with a visible tuft of hair and, possibly, an erythematous halo. See Table 25-1 for more information, and also for description of pruritus ani. See Table 25-2 for descriptions of rectal polyps and carcinoma.

The nurse is bathing an 80-year-old man and notices that his skin is wrinkled, thin, lax, and dry. This finding would be related to which factor in the older adult? a. Increased vascularity of the skin b. Increased numbers of sweat and sebaceous glands c. An increase in elastin and a decrease in subcutaneous fat d. An increased loss of elastin and a decrease in subcutaneous fat

ANS: D An accumulation of factors place the aging person at risk for skin disease and breakdown: the thinning of the skin, a decrease in vascularity and nutrients, the loss of protective cushioning of the subcutaneous layer, a lifetime of environmental trauma to skin, the social changes of aging, a increasingly sedentary lifestyle, and the chance of immobility. DIF: Cognitive Level: Applying (Application) MSC: Client Needs: Health Promotion and Maintenance

A 65-year-old woman is in the office for routine gynecologic care. She had a complete hysterectomy 3 months ago after cervical cancer was detected. The nurse knows that which of these statements is true with regard to this visit? A) Her cervical mucosa will be red and dry looking. B) She will not need to have a Pap smear done. C) The nurse can expect to find that her uterus will be somewhat enlarged and her ovaries small and hard. D) The nurse should plan to lubricate the instruments and the examining hand well to avoid a painful examination.

ANS: D In the aging adult woman, natural lubrication is decreased; to avoid a painful examination, the nurse should take care to lubricate instruments and the examining hand adequately. Menopause, with the resulting decrease in estrogen production, shows numerous physical changes. The cervix shrinks and looks pale and glistening. With the bimanual examination, the uterus feels smaller and firmer and the ovaries are not palpable normally. Women should continue cervical cancer screening up to age 70 years if they have an intact cervix and are in good health. Women who have had a total hysterectomy for benign gynecologic disease do not need cervical cancer screening, but if the hysterectomy was done for cervical cancer, then Pap tests should continue until the patient has a 10-year history of no abnormal results.

While doing an assessment of the perianal area of a patient, the nurse notices that the pigmentation of anus is darker than surrounding skin, the anal opening is closed, and there is a skin sac that is shiny and blue. The patient mentioned that he has had pain with bowel movements and has noted some spots of blood occasionally. What would this assessment and history be most likely to indicate? A) Anal fistula B) Pilonidal cyst C) Rectal prolapse D) Thrombosed hemorrhoid

ANS: D The anus normally looks moist and hairless, with coarse folded skin that is more pigmented than the perianal skin. The anal opening is tightly closed. The shiny blue skin sac indicates a thrombosed hemorrhoid.

The nurse is examining the glans and knows that which of these is a normal finding for this area? A) The meatus may have a slight discharge when the glans is compressed. B) Hair is without pest inhabitants. C) The skin is wrinkled and without lesions. D) Smegma may be present under the foreskin of an uncircumcised male.

ANS: D The glans looks smooth and without lesions and does not have hair. The meatus should not have any discharge when the glans is compressed. Some cheesy smegma may have collected under the foreskin of an uncircumcised male.

A mother brings her child into the clinic for an examination of the scalp and hair. She states that the child has developed irregularly shaped patches with broken-off, stublike hair in some places; she is worried that this condition could be some form of premature baldness. The nurse tells her that it is: a. Folliculitis that can be treated with an antibiotic. b. Traumatic alopecia that can be treated with antifungal medications. c. Tinea capitis that is highly contagious and needs immediate attention. d. Trichotillomania; her child probably has a habit of absentmindedly twirling her hair.

ANS: D Trichotillomania, self-induced hair loss, is usually due to habit. It forms irregularly shaped patches with broken-off, stublike hairs of varying lengths. A person is never completely bald. It occurs as a child absentmindedly rubs or twirls the area while falling asleep, reading, or watching television. (See Table 12-12, Abnormal Conditions of Hair, for descriptions of the other terms.)

Which of these statements about the testes is true? A) The lymphatics of the testes drain into the abdominal lymph nodes. B) The vas deferens is located along the inferior portion of each testis. C) The right testis is lower than the left because the right spermatic cord is longer. D) The cremaster muscle contracts in response to cold and draws the testicles closer to the body.

ANS: D When it is cold, the cremaster muscle contracts, which raises the scrotal sac and brings the testes closer to the body to absorb heat necessary for sperm viability. The lymphatics of the testes drain into the inguinal lymph nodes. The vas deferens is located along the upper portion of each testis. The left testis is lower than the right because the left spermatic cord is longer.

When the nurse is performing a genital examination on a male patient, the patient has an erection. The nurse's most appropriate action or response is to: A) ask the patient if he would like someone else to examine him. B) continue with the examination as though nothing has happened. C) stop the examination, leave the room while stating that the examination will resume at a later time. D) reassure the patient that this is a normal response and continue with the examination.

ANS: D When the male patient has an erection, the nurse should reassure the patient that this is a normal physiologic response to touch and proceed with the rest of the examination. The other responses are not correct and may be perceived as judgmental.

The nurse notices that a school-aged child has bluish-white, red-based spots in her mouth that are elevated approximately 1 to 3 mm. What other signs would the nurse expect to find in this patient? a. Pink, papular rash on the face and neck b. Pruritic vesicles over her trunk and neck c. Hyperpigmentation on the chest, abdomen, and back of the arms d. Red-purple, maculopapular, blotchy rash behind the ears and on the face

ANS: D With measles (rubeola), the examiner assesses a red-purple, blotchy rash on the third or fourth day of illness that appears first behind the ears, spreads over the face, and then over the neck, trunk, arms, and legs. The rash appears coppery and does not blanch. The bluish-white, red-based spots in the mouth are known as Koplik spots.

The nurse notices that a patient has a solid, elevated, circumscribed lesion that is less than 1 cm in diameter. When documenting this finding, the nurse reports this as a: a. Bulla. b. Wheal. c. Nodule. d. Papule.

ANS: D A papule is something one can feel, is solid, elevated, circumscribed, less than 1 cm in diameter, and is due to superficial thickening in the epidermis. A bulla is larger than 1 cm, superficial, and thin walled. A wheal is superficial, raised, transient, erythematous, and irregular in shape attributable to edema. A nodule is solid, elevated, hard or soft, and larger than 1 cm. DIF: Cognitive Level: Understanding (Comprehension) MSC: Client Needs: Physiologic Integrity: Physiologic Adaptation

A 65-year-old man with emphysema and bronchitis has come to the clinic for a follow-up appointment. On assessment, the nurse might expect to see which finding? a. Anasarca b. Scleroderma c. Pedal erythema d. Clubbing of the nails

ANS: D Clubbing of the nails occurs with congenital cyanotic heart disease and neoplastic and pulmonary diseases. The other responses are assessment findings not associated with pulmonary diseases. DIF: Cognitive Level: Analyzing (Analysis) MSC: Client Needs: Physiologic Integrity: Physiologic Adaptation

A woman is leaving on a trip to Hawaii and has come in for a checkup. During the examination the nurse learns that she has diabetes and takes oral hypoglycemic agents. The patient needs to be concerned about which possible effect of her medications? a. Increased possibility of bruising b. Skin sensitivity as a result of exposure to salt water c. Lack of availability of glucose-monitoring supplies d. Importance of sunscreen and avoiding direct sunlight

ANS: D Drugs that may increase sunlight sensitivity and give a burn response include sulfonamides, thiazide diuretics, oral hypoglycemic agents, and tetracycline. DIF: Cognitive Level: Applying (Application) MSC: Client Needs: Physiologic Integrity: Reduction of Risk Potential

A patient comes to the clinic and states that he has noticed that his skin is redder than normal. The nurse understands that this condition is due to hyperemia and knows that it can be caused by: a. Decreased amounts of bilirubin in the blood b. Excess blood in the underlying blood vessels c. Decreased perfusion to the surrounding tissues d. Excess blood in the dilated superficial capillaries

ANS: D Erythema is an intense redness of the skin caused by excess blood (hyperemia) in the dilated superficial capillaries. DIF: Cognitive Level: Applying (Application) MSC: Client Needs: Physiologic Integrity: Physiologic Adaptation

A patient has had a terrible itch for several months that he has been continuously scratching. On examination, the nurse might expect to find: a. A keloid. b. A fissure. c. Keratosis. d. Lichenification.

ANS: D Lichenification results from prolonged, intense scratching that eventually thickens the skin and produces tightly packed sets of papules. A keloid is a hypertrophic scar. A fissure is a linear crack with abrupt edges, which extends into the dermis; it can be dry or moist. Keratoses are lesions that are raised, thickened areas of pigmentation that appear crusted, scaly, and warty. DIF: Cognitive Level: Understanding (Comprehension) MSC: Client Needs: Physiologic Integrity: Physiologic Adaptation

A father brings in his 2-month-old infant to the clinic because the infant has had diarrhea for the last 24 hours. He says his baby has not been able to keep any formula down and that the diarrhea has been at least every 2 hours. The nurse suspects dehydration. The nurse should test skin mobility and turgor over the infants: a. Sternum. b. Forehead. c. Forearms. d. Abdomen.

ANS: D Mobility and turgor are tested over the abdomen in an infant. Poor turgor, ortenting, indicates dehydration or malnutrition. The other sites are not appropriate for checking skin turgor in an infant. DIF: Cognitive Level: Analyzing (Analysis) MSC: Client Needs: Physiologic Integrity: Physiologic Adaptation

23. A newborn infant has Down syndrome. During the skin assessment, the nurse notices a transient mottling in the trunk and extremities in response to the cool temperature in the examination room. The infants mother also notices the mottling and asks what it is. The nurse knows that this mottling is called: a. Caf au lait. b. Carotenemia. c. Acrocyanosis. d. Cutis marmorata.

ANS: D Persistent or pronounced cutis marmorata occurs with infants born with Down syndrome or those born prematurely and is a transient mottling in the trunk and extremities in response to cool room temperatures. A caf au lait spot is a large round or oval patch of light-brown pigmentation. Carotenemia produces a yellow- orange color in light-skinned persons. Acrocyanosis is a bluish color around the lips, hands and fingernails, and feet and toenails. DIF: Cognitive Level: Understanding (Comprehension) MSC: Client Needs: Physiologic Integrity: Physiologic Adaptation

A man has come in to the clinic for a skin assessment because he is worried he might have skin cancer. During the skin assessment the nurse notices several areas of pigmentation that look greasy, dark, and stuck on his skin. Which is the best prediction? a. Senile lentigines, which do not become cancerous b. Actinic keratoses, which are precursors to basal cell carcinoma c. Acrochordons, which are precursors to squamous cell carcinoma d. Seborrheic keratoses, which do not become cancerous

ANS: D Seborrheic keratoses appear like dark, greasy, stuck-on lesions that primarily develop on the trunk. These lesions do not become cancerous. Senile lentigines are commonly called liver spots and are not precancerous. Actinic (senile or solar) keratoses are lesions that are red-tan scaly plaques that increase over the years to become raised and roughened. They may have a silvery-white scale adherent to the plaque. They occur on sun- exposed surfaces and are directly related to sun exposure. They are premalignant and may develop into squamous cell carcinoma. Acrochordons are skin tags and are not precancerous. DIF: Cognitive Level: Analyzing (Analysis) MSC: Client Needs: Physiologic Integrity: Physiologic Adaptation

The nurse educator is preparing an education module for the nursing staff on the dermis layer of skin. Which of these statements would be included in the module? The dermis: a. Contains mostly fat cells. b. Consists mostly of keratin. c. Is replaced every 4 weeks. d. Contains sensory receptors.

ANS: D The dermis consists mostly of collagen, has resilient elastic tissue that allows the skin to stretch, and contains nerves, sensory receptors, blood vessels, and lymphatic vessels. It is not replaced every 4 weeks. DIF: Cognitive Level: Understanding (Comprehension) MSC: Client Needs: General

The nurse is assessing for clubbing of the fingernails and expects to find: a. Nail bases that are firm and slightly tender. b. Curved nails with a convex profile and ridges across the nails. c. Nail bases that feel spongy with an angle of the nail base of 150 degrees. d. Nail bases with an angle of 180 degrees or greater and nail bases that feel spongy.

ANS: D The normal nail is firm at its base and has an angle of 160 degrees. In clubbing, the angle straightens to 180 degrees or greater and the nail base feels spongy. DIF: Cognitive Level: Understanding (Comprehension) MSC: Client Needs: Physiologic Integrity: Physiologic Adaptation

The nurse notices that a school-aged child has bluish-white, red-based spots in her mouth that are elevated approximately 1 to 3 mm. What other signs would the nurse expect to find in this patient? a. Pink, papular rash on the face and neck b. Pruritic vesicles over her trunk and neck c. Hyperpigmentation on the chest, abdomen, and back of the arms d. Red-purple, maculopapular, blotchy rash behind the ears and on the face

ANS: D With measles (rubeola), the examiner assesses a red-purple, blotchy rash on the third or fourth day of illness that appears first behind the ears, spreads over the face, and then over the neck, trunk, arms, and legs. The rash appears coppery and does not blanch. The bluish-white, red-based spots in the mouth are known as Koplik spots. DIF: Cognitive Level: Analyzing (Analysis) MSC: Client Needs: Physiologic Integrity: Physiologic Adaptation

A mother brings her child into the clinic for an examination of the scalp and hair. She states that the child has developed irregularly shaped patches with broken-off, stublike hair in some places; she is worried that this condition could be some form of premature baldness. The nurse tells her that it is: a. Folliculitis that can be treated with an antibiotic. b. Traumatic alopecia that can be treated with antifungal medications. c. Tinea capitis that is highly contagious and needs immediate attention. d. Trichotillomania; her child probably has a habit of absentmindedly twirling her hair.

ANS: D Trichotillomania, self-induced hair loss, is usually due to habit. It forms irregularly shaped patches with broken-off, stublike hairs of varying lengths. A person is never completely bald. It occurs as a child absentmindedly rubs or twirls the area while falling asleep, reading, or watching television. DIF: Cognitive Level: Applying (Application) MSC: Client Needs: Physiologic Integrity: Physiologic Adaptation

The nurse is checking the range of motion in a patient's knee and knows that the knee is capable of which movement(s)? A) Flexion and extension B) Supination and pronation C) Circumduction D) Inversion and eversion

ANS: Flexion and extension The knee is a hinge joint, permitting flexion and extension of the lower leg on a single plane. The knee is not capable of the other movements listed.

A patient with pleuritis has been admitted to the hospital and complains of pain with breathing. What other key assessment finding would the nurse expect to find upon auscultation? a. Stridor b. Friction rub c. Crackles d. Wheezing

ANS: Friction rub A patient with pleuritis will exhibit a pleural friction rub upon auscultation. This is the sound made when pleurae become inflamed and rub together during respiration. The sound is superficial, coarse, and low-pitched, as if two pieces of leather are being rubbed together. Stridor is associated with croup, acute epiglottitis in children, and foreign body inhalation. Crackles are associated with several diseases, such as pneumonia, heart failure, chronic bronchitis, and others (see Table 18-6). Wheezes are associated with diffuse airway obstruction caused by acute asthma or chronic emphysema.

A patient has been admitted to the emergency department for a suspected drug overdose. His respirations are shallow, with an irregular pattern, with a rate of 12 per minute. The nurse interprets this respiration pattern as which of the following? a. Bradypnea b. Cheyne-Stokes respirations c. Hypoventilation d. Chronic obstructive breathing

ANS: Hypoventilation Hypoventilation is characterized by an irregular, shallow pattern, and can be caused by an overdose of narcotics or anesthetics. Bradypnea is slow breathing, with a rate less than 10 respirations per minute. See Table 18-4 for descriptions of Cheyne-Stokes respirations and chronic obstructive breathing.

The nurse is reviewing venous blood flow patterns. Which of these statements best describes the mechanism(s) by which venous blood returns to the heart? a. Intraluminal valves ensure unidirectional flow toward the heart. b. Contracting skeletal muscles milk blood distally toward the veins. c. High-pressure system of the heart helps facilitate venous return. d. Increased thoracic pressure and decreased abdominal pressure facilitate venous return to the heart.

ANS: Intraluminal valves ensure unidirectional flow toward the heart. Blood moves through the veins by (1) contracting skeletal muscles that milk the blood proximally; (2) pressure gradients caused by breathing, in which inspiration makes the thoracic pressure decrease and the abdominal pressure increase; and (3) the intraluminal valves, which ensure unidirectional flow toward the heart.

During an examination, the nurse notes a supernumerary nipple just under the patient's left breast. The patient tells the nurse that she always thought it was a mole. Which statement about this finding is correct? a. This variation is normal and not a significant finding. b. This finding is significant and needs further investigation. c. A supernumerary nipple also contains glandular tissue and may leak milk during pregnancy and lactation. d. The patient is correct—a supernumerary nipple is actually a mole that happens to be located under the breast.

ANS: It is a normal variation and not a significant finding. A supernumerary nipple looks like a mole, but close examination reveals a tiny nipple and areola. It is not a significant finding.

The nurse is preparing to assess the dorsalis pedis artery. Where is the correct location for palpation? a. Behind the knee b. Over the lateral malleolus c. In the groove behind the medial malleolus d. Lateral to the extensor tendon of the great toe

ANS: Lateral to the extensor tendon of the great toe The dorsalis pedis artery is located on the dorsum of the foot. The nurse should palpate just lateral to and parallel with the extensor tendon of the big toe. The popliteal artery is palpated behind the knee. The posterior tibial pulse is palpated in the groove between the malleolus and the Achilles tendon. There is no pulse palpated at the lateral malleolus.

During an assessment, the nurse notices that a patient's left arm is swollen from the shoulder down to the fingers, with nonpitting brawny edema. The right arm is normal. The patient had a left-sided mastectomy 1 year ago. The nurse suspects which problem? a. Venous stasis b. Lymphedema c. Arteriosclerosis d. Deep-vein thrombosis

ANS: Lymphedema Lymphedema after breast cancer causes unilateral swelling and nonpitting brawny edema, with overlying skin indurated. It is caused by the removal of lymph nodes with breast surgery or damage to lymph nodes and channels with radiation therapy for breast cancer, and it can impede drainage of lymph. The other responses are not correct.

A new mother calls the clinic to report that part of her left breast is red, swollen, tender, very hot, and hard. She has a fever of 101 F. She has also had symptoms of the flu, such as chills, sweating, and feeling tired. The nurse notices that she has been breastfeeding for 1 month. From her description, what condition does the nurse suspect? a. Mastitis b. Paget disease c. Plugged milk duct d. Mammary duct ectasia

ANS: Mastitis The symptoms describe mastitis, which stems from infection or stasis caused by a plugged duct. A plugged duct does not have infection present. (See Table 17-7.) Refer to Table 17-6 for descriptions of Paget's disease and mammary duct ectasia.

During an assessment, the nurse knows that expected assessment findings in the normal adult lung include the presence of: a. Adventitious sounds and limited chest expansion. b. Increased tactile fremitus and dull percussion tones. c. Muffled voice sounds and symmetric tactile fremitus. d. Absent voice sounds and hyperresonant percussion tones.

ANS: Muffled voice sounds and symmetric tactile fremits. Normal lung findings include symmetric chest expansion, resonant percussion tones, vesicular breath sounds over the peripheral lung fields, muffled voice sounds, and no adventitious sounds.

The nurse is reviewing principles of pain. Which type of pain is due to an abnormal processing of the pain impulse through the peripheral or central nervous system?

ANS: Neuropathic Neuropathic pain implies an abnormal processing of the pain message. The other types of pain are named according to their sources.

The nurse is assessing the breasts of a 68-year-old woman and discovers a mass in the upper outer quadrant of the left breast. When assessing this mass, the nurse keeps in mind that characteristics of a cancerous mass include which of the following? Select all that apply. a. Nontender mass b. Dull, heavy pain on palpation c. Rubbery texture and mobile d. Hard, dense, and immobile e. Regular border f. Irregular, poorly delineated border

ANS: Nontender mass Hard, dense, and immobile Regular border Cancerous breast masses are solitary, unilateral, nontender, masses. They are solid, hard, dense, and fixed to underlying tissues or skin as cancer becomes invasive. Their borders are irregular and poorly delineated. They are often painless, although the person may have pain. They are most common in upper outer quadrant. A dull, heavy pain on palpation and a mass with a rubbery texture and a regular border are characteristics of benign breast disease.

The nurse is examining a 62-year-old man and notes that he has gynecomastia bilaterally. The nurse should explore his history for which related conditions? Select all that apply. a. Malnutrition b. Hyperthyroidism c. Type 2 diabetes mellitus d. Liver disease e. History of alcohol abuse

ANS: Obesity Hyperthyroidism Liver disease History of alcohol abuse Gynecomastia occurs with obesity, Cushing's syndrome, liver cirrhosis, adrenal disease, hyperthyroidism, and numerous drugs: alcohol and marijuana use, estrogen treatment for prostate cancer, antibiotics (metronidazole, isoniazid), digoxin, ACE inhibitors, diazepam, and tricyclic antidepressants.

The nurse is reviewing the principles of nociception. During which phase of nociception does the conscious awareness of a painful sensation occur?

ANS: Perception Perception is the third phase of nociception and indicates the conscious awareness of a painful sensation. During this phase, the sensation is recognized by higher cortical structures and identified as pain.

During an assessment of an older adult, the nurse should expect to notice which finding as a normal physiologic change associated with the aging process? a. Hormonal changes causing vasodilation and a resulting drop in blood pressure b. Progressive atrophy of the intramuscular calf veins, causing venous insufficiency c. Peripheral blood vessels growing more rigid with age, producing a rise in systolic blood pressure d. Narrowing of the inferior vena cava, causing low blood flow and increases in venous pressure resulting in varicosities

ANS: Peripheral blood vessels growing more rigid with age, producing a rise in systolic blood pressure Peripheral blood vessels grow more rigid with age, resulting in a rise in systolic blood pressure. Aging produces progressive enlargement of the intramuscular calf veins, not atrophy. The other options are not correct

The nurse is reviewing risk factors for venous disease. Which of these situations best describes a person at highest risk for development of venous disease? a. Woman in her second month of pregnancy b. Person who has been on bed rest for 4 days c. Person with a 30-year, 1 pack per day smoking habit d. Older adult taking anticoagulant medication

ANS: Person who has been on bed rest for 4 days At risk for venous disease are people who undergo prolonged standing, sitting, or bed rest. Hypercoagulable (not anticoagulated) states and vein wall trauma also place the person at risk for venous disease. Obesity and pregnancy are also risk factors, but not the early months of pregnancy.

During a morning assessment, the nurse notices that the patient's sputum is frothy and pink. Which condition could this finding indicate? a. Croup b. Tuberculosis c. Viral infection d. Pulmonary edema

ANS: Pulmonary edema Sputum alone is not diagnostic, but some conditions have characteristic sputum production. Pink, frothy sputum indicates pulmonary edema (or it may be a side effect of sympathomimetic medications). Croup is associated with a "barking" cough, not sputum production. Tuberculosis may produce rust-colored sputum. Viral infections may produce white or clear mucoid sputum.

During an assessment, a patient tells the nurse that her fingers often change color when she goes out in cold weather. She describes these episodes as her fingers first turning white, then blue, then red with a burning, throbbing pain. The nurse suspects that she is experiencing: a. Lymphedema. b. Raynaud disease. c. Deep-vein thrombosis. d. Chronic arterial insufficiency.

ANS: Raynaud's disease. The condition with episodes of abrupt, progressive tricolor change of the fingers in response to cold, vibration, or stress is known as Raynaud's disease. Lymphedema is described in Table 20-2; deep vein thrombosis is described in Table 20-5; chronic arterial insufficiency is described in Table 20-4.

A patient is complaining of pain in his joints that is worse in the morning, is better after he has moved around for awhile, and then gets worse again if he sits for long periods of time. The nurse should assess for other signs of what problem? A) Tendinitis B) Osteoarthritis C) Rheumatoid arthritis D) Intermittent claudication

ANS: Rheumatoid arthritis Rheumatoid arthritis is worse in the morning when arising. Movement increases most joint pain, except in rheumatoid arthritis, in which movement decreases pain. The other options are not correct.

A patient has had arthritic pain in her hips for several years since a hip fracture. She is able to move around in her room and has not offered any complaints so far this morning. However, when asked, she states that her pain is "bad this morning" and rates it at an 8 on a 1 to 10 scale. What does the nurse suspect?

ANS: She has experienced chronic pain for years and has adapted to it. Persons with chronic pain typically try to give little indication that they are in pain and, over time, adapt to the pain. As a result, they are at risk for underdetection.

A 70-year-old patient is being seen in the clinic for severe exacerbation of his heart failure. Which of these findings is the nurse most likely to observe in this situation? a. Shortness of breath, orthopnea, paroxysmal nocturnal dyspnea, and ankle edema b. Rasping cough, thick mucoid sputum, wheezing, and bronchitis c. Productive cough, dyspnea, weight loss, anorexia, and tuberculosis d. Fever, dry nonproductive cough, and diminished breath sounds

ANS: Shortness of breath, orthopnea, paroxysmal nocturnal dyspnea, ankle edema Heart failure often presents with increased respiratory rate, shortness of breath on exertion, orthopnea, paroxysmal nocturnal dyspnea, nocturia, ankle edema, and pallor in light-skinned individuals. A patient with rasping cough, thick mucoid sputum, and wheezing may have bronchitis. Productive cough, dyspnea, weight loss, and dyspnea are seen with tuberculosis; fever, dry nonproductive cough, and diminished breath sounds may indicate Pneumocystis jiroveci (P. carinii) pneumonia. See Table 18-8.

During assessment of a patient's pain, the nurse keeps in mind that certain nonverbal behaviors are associated with chronic pain. Which of these behaviors are associated with chronic pain? Select all that apply.

ANS: Sleeping, Bracing, Rubbing Behaviors that have been associated with chronic pain include bracing, rubbing, diminished activity, sighing, and change in appetite. In addition, those with chronic pain may sleep in an attempt at distraction. The other behaviors are associated with acute pain.

A patient who has had rheumatoid arthritis for years comes to the clinic to ask about changes in her fingers. The nurse will assess for signs of what problems? A) Heberden's nodes B) Bouchard's nodules C) Swan neck deformities D) Dupuytren's contractures

ANS: Swan neck deformities Changes in the fingers caused by chronic rheumatoid arthritis include swan neck and boutonniere deformities. Heberden's nodes and Bouchard's nodules are associated with osteoarthritis. Dupuytren's contractures occur because of chronic hyperplasia of the palmar fascia and causes contractures of the digits (see Table 22-4).

The nurse is teaching a class on osteoporosis prevention to a group of postmenopausal women. Which of these actions is the best way to prevent or delay bone loss in this group? A) Taking calcium and vitamin D supplements B) Taking medications to prevent osteoporosis C) Performing physical activity, such as fast walking D) Assessing bone density annually

ANS: Taking calcium and vitamin D supplements Physical activity, such as fast walking, delays or prevents bone loss in perimenopausal women. The faster the pace of walking, the higher the preventive effect on the risk of hip fracture. The other options are not correct.

A 4-year-old boy is brought to the emergency department by his mother. She says he points to his stomach and says, "It hurts so bad." Which pain assessment tool would be the best choice when assessing this child's pain?

ANS: The Faces Pain Scale—Revised (FPS-R) Rating scales can be introduced at the age of 4 or 5 years. The Faces Pain Scale—Revised (FPS-R) is designed for use by children and asks the child to choose a face that shows "how much hurt (or pain) you have now." Young children should not be asked to rate pain by using numbers.

Which of these statements is true regarding the arterial system? a. Arteries are large-diameter vessels. b. The arterial system is a high-pressure system. c. The walls of arteries are thinner than those of the veins. d. Arteries can greatly expand to accommodate a large blood volume increase.

ANS: The arterial system is a high-pressure system. The pumping heart makes the arterial system a high-pressure system.

The nurse is conducting a class about breast self-examination (BSE). Which of these statements indicates proper BSE technique? a. The best time to perform BSE is in the middle of the menstrual cycle. b. The woman needs to perform BSE only bimonthly unless she has fibrocystic breast tissue. c. The best time to perform a BSE is 4 to 7 days after the first day of the menstrual period. d. If she suspects that she is pregnant, then the woman should not perform a BSE until her baby is born.

ANS: The best time to perform BSE is 4 to 7 days after the first day of the menstrual period. The nurse should help each woman establish a regular schedule of self-care. The best time to conduct breast self-examination is right after the menstrual period, or the fourth through seventh day of the menstrual cycle, when the breasts are the smallest and least congested. Advise the pregnant or menopausal woman who is not having menstrual periods to select a familiar date to examine her breasts each month, for example, her birth date or the day the rent is due.

The nurse is assessing the lungs of an older adult. Which of these describes normal changes in the respiratory system of the older adult? a. Severe dyspnea is experienced on exertion, resulting from changes in the lungs. b. Respiratory muscle strength increases to compensate for a decreased vital capacity. c. Decrease in small airway closure occurs, leading to problems with atelectasis. d. Lungs are less elastic and distensible, which decreases their ability to collapse and recoil.

ANS: The lungs are less elastic and distensible, which decreases their ability to collapse and recoil. In the aging adult the lungs are less elastic and distensible, which decreases their ability to collapse and recoil. There is a decreased vital capacity and a loss of intraalveolar septa, causing less surface area for gas exchange. The lung bases become less ventilated, and the older person is at risk for dyspnea with exertion beyond his or her usual workload.

When assessing the respiratory system of a 4-year-old child, which of these findings would the nurse expect? a. Crepitus palpated at the costochondral junctions b. No diaphragmatic excursion as a result of a child's decreased inspiratory volume c. Presence of bronchovesicular breath sounds in the peripheral lung fields d. Irregular respiratory pattern and a respiratory rate of 40 breaths per minute at rest

ANS: The presence of bronchovesicular breath sounds in the peripheral lung fields Bronchovesicular breath sounds in the peripheral lung fields of the infant and young child up to age 5 or 6 years are a normal finding. Their thin chest walls with underdeveloped musculature do not dampen the sound, as do the thicker chest walls of adults, so breath sounds are louder and harsher.

The nurse is examining the lymphatic system of a healthy 3-year-old child. Which finding should the nurse expect? a. Excessive swelling of the lymph nodes b. Presence of palpable lymph nodes c. No palpable nodes because of the immature immune system of a child d. Fewer numbers and a smaller size of lymph nodes compared with those of an adult

ANS: The presence of palpable lymph nodes Lymph nodes are relatively large in children, and the superficial ones often are palpable even when the child is healthy.

When inspecting the anterior chest of an adult, the nurse should include which assessment? a. Diaphragmatic excursion b. Symmetric chest expansion c. Presence of breath sounds d. Shape and configuration of the chest wall

ANS: The shape and configuration of the chest wall Inspection of the anterior chest includes shape and configuration of the chest wall; assessment of the patient's level of consciousness, skin color and condition; quality of respirations; presence or absence of retraction and bulging of the intercostal spaces; and use of accessory muscles. Symmetric chest expansion is assessed by palpation. Diaphragmatic excursion is assessed by percussion of the posterior chest. Breath sounds are assessed by auscultation.

The nurse is examining a 6-month-old infant and places the infant's feet flat on the table and flexes his knees up. The nurse notes that the right knee is significantly lower than the left. Which of these statements is true of this finding? A) This is a positive Allis sign and suggests hip dislocation. B) The infant probably has a dislocated patella on the right. C) This is a normal finding for the Allis test for an infant of this age. D) The infant should return to the clinic in 2 weeks to see if this has changed.

ANS: This is a positive Allis sign and suggests hip dislocation. Finding one knee significantly lower than the other is a positive Allis sign and suggests hip dislocation. Normally the tops of the knees are at the same elevation. The other statements are not correct.

The nurse is preparing to perform a modified Allen test. Which is an appropriate reason for this test? a. To measure the rate of lymphatic drainage b. To evaluate the adequacy of capillary patency before venous blood draws c. To evaluate the adequacy of collateral circulation before cannulating the radial artery d. To evaluate the venous refill rate that occurs after the ulnar and radial arteries are temporarily occluded

ANS: To evaluate the adequacy of collateral circulation before cannulating the radial artery A modified Allen test is used to evaluate the adequacy of collateral circulation before the radial artery is cannulated. The other responses are not reasons for a modified Allen test.

The nurse is auscultating the chest in an adult. Which technique is correct? a. Instructing the patient to take deep, rapid breaths b. Instructing the patient to breathe in and out through his or her nose c. Firmly holding the diaphragm of the stethoscope against the chest d. Lightly holding the bell of the stethoscope against the chest to avoid friction

ANS: Use the diaphragm of the stethoscope held firmly against the chest. The diaphragm of the stethoscope held firmly on the chest is the correct way to auscultate breath sounds. The patient should be instructed to breathe through his or her mouth, a little deeper than usual, but not to hyperventilate.

During a clinic visit, a woman in her seventh month of pregnancy complains that her legs feel "heavy in the calf" and that she often has foot cramps at night. The nurse notices that the patient has dilated, tortuous veins in her lower legs. Which condition is reflected by these findings? a. Deep-vein thrombophlebitis b. Varicose veins c. Lymphedema d. Raynaud phenomenon

ANS: Varicose veins Superficial varicose veins are caused by incompetent distant valves on veins, which results in reflux of blood and producing dilated, tortuous veins. They are more common in women, and pregnancy can also be a cause. Symptoms include aching, heaviness in the calf, easy fatigability, and night leg or foot cramps. Dilated, tortuous veins are seen on assessment. See Table 20-5 for the description of deep vein thrombophlebitis. See Table 20-2 for descriptions of Raynaud's phenomenon and lymphedema.

The nurse is listening to the breath sounds of a patient with severe asthma. Air passing through narrowed bronchioles would produce which of these adventitious sounds? a. Wheezes b. Bronchial sounds c. Bronchophony d. Whispered pectoriloquy

ANS: Wheezes Wheezes are caused by air squeezed or compressed through passageways narrowed almost to closure by collapsing, swelling, secretions, or tumors, such as with acute asthma or chronic emphysema.

During an assessment of an adult, the nurse has noted unequal chest expansion and recognizes that this occurs in which situation? a. In an obese patient b. When part of the lung is obstructed or collapsed c. When bulging of the intercostal spaces is present d. When accessory muscles are used to augment respiratory effort

ANS: When part of the lung is obstructed or collapsed Unequal chest expansion occurs when part of the lung is obstructed or collapsed, as with pneumonia, or when guarding to avoid postoperative incisional pain or atelectasis.

During auscultation of the lungs, the nurse expects decreased breath sounds to be heard in which situation? a. When the bronchial tree is obstructed b. When adventitious sounds are present c. In conjunction with whispered pectoriloquy d. In conditions of consolidation, such as pneumonia

ANS: When the bronchial tree is obstructed Decreased or absent breath sounds occur when the bronchial tree is obstructed, as in emphysema, and when sound transmission is obstructed, as in pleurisy, pneumothorax, or pleural effusion.

A patient has been diagnosed with a ganglion cyst over the dorsum of his left wrist. He asks the nurse, "What is this thing?" The nurse's best answer would be, "It is: A) a common benign tumor." B) a tumor that will have to be watched because it may turn malignant." C) caused by chronic repetitive motion injury." D) a skin infection that will need to be drained."

ANS: a common benign tumor." A ganglionic cyst is a common benign tumor; it does not become malignant, and it does not need to be drained. It is not caused by chronic repetitive motion injury.

When performing a respiratory assessment on a patient, the nurse notices a costal angle of approximately 90 degrees. This characteristic is: a. Observed in patients with kyphosis. b. Indicative of pectus excavatum. c. A normal finding in a healthy adult. d. An expected finding in a patient with a barrel chest.

ANS: a normal finding in a healthy adult. The right and left costal margins form an angle where they meet at the xiphoid process. Usually, this angle is 90 degrees or less. The angle increases when the rib cage is chronically overinflated, as in emphysema.

A teenage patient comes to the emergency department with complaints of an inability to breathe and a sharp pain in the left side of his chest. The assessment findings include cyanosis, tachypnea, tracheal deviation to the right, decreased tactile fremitus on the left, hyperresonance on the left, and decreased breath sounds on the left. The nurse interprets that these assessment findings are consistent with: a. Bronchitis. b. Pneumothorax. c. Acute pneumonia. d. Asthmatic attack.

ANS: a pneumothorax. With a pneumothorax, free air in the pleural space causes partial or complete lung collapse. If the pneumothorax is large, then tachypnea and cyanosis are seen. Unequal chest expansion, decreased or absent tactile fremitus, tracheal deviation to the unaffected side, decreased chest expansion, hyperresonant percussion tones, and decreased or absent breath sounds are found with the presence of pneumothorax. See Table 18-8 for descriptions of the other conditions.

During a breast examination on a female patient, the nurse notices that the nipple is flat, broad, and fixed. The patient states it "started doing that a few months ago." This finding suggests: a. Dimpling. b. Retracted nipple. c. Nipple inversion. d. Deviation in nipple pointing.

ANS: a retracted nipple. The retracted nipple looks flatter and broader, like an underlying crater. A recent retraction suggests cancer, which causes fibrosis of the whole duct system and pulls in the nipple. It also may occur with benign lesions such as ectasia of the ducts. The nurse should not confuse retraction with the normal long-standing type of nipple inversion, which has no broadening and is not fixed.

During an annual physical exam, a 43-year-old patient states that she doesn't perform monthly breast self-examinations (BSE). She tells the nurse that she believes that mammograms "do a much better job than I ever could to find a lump." The nurse should explain to her that: a. BSEs may detect lumps that appear between mammograms. b. BSEs are unnecessary until the age of 50 years. c. She is correct—mammography is a good replacement for BSE. d. She does not need to perform BSEs as long as a physician checks her breasts annually.

ANS: a. BSEs may detect lumps that appear between mammograms. The monthly practice of breast self-examination, along with clinical breast examination and mammograms are complementary screening measures. Mammography can reveal cancers too small to be detected by the woman or by the most experienced examiner. However, interval lumps may become palpable between mammograms.

In performing an assessment of a woman's axillary lymph system, the nurse should assess which of these nodes? a. Central, axillary, lateral, and sternal b. Pectoral, lateral, anterior, and sternal c. Central, lateral, pectoral, and subscapular d. Lateral, pectoral, axillary, and suprascapular

ANS: a. Central, lateral, pectoral, and subscapular nodes The breast has extensive lymphatic drainage. Four groups of axillary nodes are present: (1) central, (2) pectoral (anterior), (3) subscapular (posterior), and (4) lateral.

The nurse is palpating a female patient's breasts during an examination. Which of these positions is most likely to make significant lumps more distinct during breast palpation? a. Supine with the arms raised over her head b. Sitting with the arms relaxed at her sides c. Supine with the arms relaxed at her sides d. Sitting with the arms flexed and fingertips touching her shoulders

ANS: a. Supine with arms raised over her head The nurse should help the woman to a supine position, tuck a small pad under the side to be palpated, and help the woman raise her arm over her head. These maneuvers will flatten the breast tissue and displace it medially. Any significant lumps will then feel more distinct.

The nurse is assisting with a self-breast examination clinic. Which of these women reflect abnormal findings during the inspection phase of breast examination? a. Woman whose nipples are in different planes (deviated). b. Woman whose left breast is slightly larger than her right. c. Nonpregnant woman whose skin is marked with linear striae. d. Pregnant woman whose breasts have a fine blue network of veins visible under the skin.

ANS: a. Woman whose nipples are in different planes (deviated) The nipples should be symmetrically placed on the same plane on the two breasts. With deviation in pointing, an underlying cancer causes fibrosis in the mammary ducts, which pulls the nipple angle toward it. The other examples are normal findings. See Table 17-3.

A patient is newly diagnosed with benign breast disease. The nurse recognizes that which statement about benign breast disease is true? The presence of benign breast disease: a. Makes it hard to examine the breasts. b. Frequently turns into cancer in a woman's later years. c. Is easily reduced with hormone replacement therapy. d. Is usually diagnosed before a woman reaches childbearing age.

ANS: a. makes it harder to examine the breasts. The presence of benign breast disease (formerly fibrocystic breast disease) makes it harder to examine the breasts; the general lumpiness of the breast conceals a new lump. The other statements are not true.

An 80-year-old woman is visiting the clinic for a checkup. She states, "I can't walk as much as I used to." The nurse is observing for motor dysfunction in her hip and should have her: A) internally rotate her hip while she is sitting. B) abduct her hip while she is lying on her back. C) adduct her hip while she is lying on her back. D) externally rotate her hip while she is standing.

ANS: abduct her hip while she is lying on her back. Limitation of abduction of the hip while supine is the most common motion dysfunction found in hip disease. The other options are not correct.

A 40-year-old man has come into the clinic with complaints of "extreme tenderness in my toes." The nurse notices that his toes are slightly swollen, reddened, and warm to the touch. His complaints would suggest: A) osteoporosis. B) acute gout. C) ankylosing spondylitis. D) degenerative joint disease.

ANS: acute gout. Acute gout occurs primarily in men over 40 years of age. Clinical findings consist of redness, swelling, heat, and extreme tenderness. Gout is a metabolic disorder of disturbed purine metabolism, associated with elevated serum uric acid. See Table 22-1 for descriptions of the other terms.

A patient is being assessed for range of joint movement. The nurse asks him to move his arm in toward the center of his body. This movement is called: A) flexion. B) abduction. C) adduction. D) extension.

ANS: adduction. Moving a limb toward the midline of the body is called adduction; abduction is moving a limb away from the midline of the body. Flexion is bending a limb at a joint; extension is straightening a limb at a joint.

A young swimmer comes to the sports clinic complaining of a very sore shoulder. He was running at the pool, slipped on some wet concrete, and tried to catch himself with his outstretched hand. He landed on his outstretched hand and has not been able to move his shoulder since then. The nurse suspects: A) joint effusion. B) tear of rotator cuff. C) adhesive capsulitis. D) dislocated shoulder.

ANS: adhesive capsulitis. Dislocated shoulder occurs with trauma involving abduction, extension, and external rotation (e.g., falling on an outstretched arm or diving into a pool). See Table 22-2 for a description of the other conditions.

During a routine office visit, a patient takes off his shoes and shows the nurse "this awful sore that won't heal." On inspection, the nurse notes a 3-cm round ulcer on the left great toe, with a pale ischemic base, well-defined edges, and no drainage. The nurse should assess for other signs and symptoms of: a. Varicosities. b. Venous stasis ulcer. c. Arterial ischemic ulcer. d. Deep-vein thrombophlebitis.

ANS: an arterial ischemic ulcer. Arterial ischemic ulcers occur at toes, metatarsal heads, heels, and lateral ankle, and they are characterized by a pale ischemic base, well-defined edges, and no bleeding. See Table 20-5 for a description of varicose veins and deep vein thrombophlebitis. See Table 20-4 for a description of venous stasis ulcers.

To palpate the temporomandibular joint, the nurse's fingers should be placed in the depression _____ of the ear. A) distal to the helix B) proximal to the helix C) anterior to the tragus D) posterior to the tragus

ANS: anterior to the tragus The temporomandibular joint can be felt in the depression anterior to the tragus of the ear. The other locations are not correct.

An adult patient with a history of allergies comes to the clinic complaining of wheezing and difficulty in breathing when working in his yard. The assessment findings include tachypnea, use of accessory neck muscles, prolonged expiration, intercostal retractions, decreased breath sounds, and expiratory wheezes. The nurse interprets that these assessment findings are consistent with: a. Asthma. b. Atelectasis. c. Lobar pneumonia. d. Heart failure.

ANS: asthma. Asthma is allergic hypersensitivity to certain inhaled particles that produces inflammation and a reaction of bronchospasm, which increases airway resistance, especially during expiration. Increased respiratory rate, use of accessory muscles, retraction of intercostal muscles, prolonged expiration, decreased breath sounds, and expiratory wheezing are all characteristic of asthma. See Table 18-8 for descriptions of the other conditions.

The nurse is auscultating the lungs of a patient who had been sleeping and notices short, popping, crackling sounds that stop after a few breaths. The nurse recognizes that these breath sounds are: a. Atelectatic crackles that do not have a pathologic cause. b. Fine crackles and may be a sign of pneumonia. c. Vesicular breath sounds. d. Fine wheezes.

ANS: atelectatic crackles, and that they are not pathologic. One type of adventitious sound, atelectatic crackles, is not pathologic. They are short, popping, crackling sounds that sound like fine crackles but do not last beyond a few breaths. When sections of alveoli are not fully aerated (as in people who are asleep or in the elderly), they deflate slightly and accumulate secretions. Crackles are heard when these sections are expanded by a few deep breaths. Atelectatic crackles are heard only in the periphery, usually in dependent portions of the lungs, and disappear after the first few breaths or after a cough.

During a breast health interview, a patient states that she has noticed pain in her left breast. The nurse's most appropriate response to this would be: a. "Don't worry about the pain; breast cancer is not painful." b. "I would like some more information about the pain in your left breast." c. "Oh, I had pain like that after my son was born; it turned out to be a blocked milk duct." d. "Breast pain is almost always the result of benign breast disease."

ANS: b. "I would like some more information about the pain in your left breast." Breast pain occurs with trauma, inflammation, infection, or benign breast disease. The nurse will need to gather more information about the patient's pain rather than make statements that ignore the patient's concerns.

Which of the following statements is true regarding the internal structures of the breast? The breast is: a. Primarily muscle with very little fibrous tissue. b. Fibrous, glandular, and adipose tissues. c. Primarily milk ducts, known as lactiferous ducts. d. Glandular tissue, which supports the breast by attaching to the chest wall.

ANS: b. composed of fibrous, glandular, and adipose tissue. The breast is composed of glandular tissue, fibrous tissue (including the suspensory ligaments), and adipose tissue.

If a patient reports a recent breast infection, then the nurse should expect to find _____ node enlargement. a. Nonspecific b. Ipsilateral axillary c. Contralateral axillary d. Inguinal and cervical

ANS: b. ipsilateral axillary The breast has extensive lymphatic drainage. Most of the lymph, more than 75%, drains into the ipsilateral, or same side, axillary nodes.

A patient states during the interview that she noticed a new lump in the shower a few days ago. It was on her left breast near her axilla. The nurse should plan to: a. Palpate the lump first. b. Palpate the unaffected breast first. c. Avoid palpating the lump because it could be a cyst, which might rupture. d. Palpate the breast with the lump first but plan to palpate the axilla last.

ANS: b. palpate the unaffected breast first. If the woman mentions a breast lump she has discovered herself, the nurse should examine the unaffected breast first to learn a baseline of normal consistency for this individual.

When reviewing the musculoskeletal system, the nurse recalls that hematopoiesis takes place in the: A) liver. B) spleen. C) kidneys. D) bone marrow.

ANS: bone marrow. The musculoskeletal system functions to encase and protect inner vital organs, support the body, produce red blood cells in the bone marrow, and store minerals.

The nurse is assessing the pulses of a patient who has been admitted for untreated hyperthyroidism. The nurse should expect to find a(n) _____ pulse. a. Normal b. Absent c. Bounding d. Weak, thready

ANS: bounding A full, bounding pulse occurs with hyperkinetic states (such as exercise, anxiety, fever), anemia, and hyperthyroidism. Absent pulse occurs with occlusion. Weak, thready pulses occur with shock and peripheral artery disease.

The nurse is reviewing the blood supply to the arm. The major artery supplying the arm is the _____ artery. a. Ulnar b. Radial c. Brachial d. Deep palmar

ANS: brachial The major artery supplying the arm is the brachial artery. The brachial artery bifurcates into the ulnar and radial arteries immediately below the elbow. In the hand, the ulnar and radial arteries form two arches known as the superficial and deep palmar arches.

When auscultating over a patient's femoral arteries the nurse notices the presence of a bruit on the left side. The nurse knows that: a. Are often associated with venous disease. b. Occur in the presence of lymphadenopathy. c. In the femoral arteries are caused by hypermetabolic states. d. Occur with turbulent blood flow, indicating partial occlusion.

ANS: bruits occur with turbulent blood flow, indicating partial occlusion. A bruit occurs with turbulent blood flow and indicates partial occlusion of the artery. The other responses are not correct.

The nurse is preparing for a class in early detection of breast cancer. Which statement is true with regard to breast cancer in African-American women in the United States? a. Breast cancer is not a threat to black women. b. Black women have a lower incidence of regional or distant breast cancer than white women. c. Black women are more likely to die of breast cancer at any age. d. Breast cancer incidence in black women is higher than that of white women after age 45.

ANS: c. African-American women are more likely to die of breast cancer at any age. African-American women have a higher incidence of breast cancer before age 45 years than white women, and are more likely to die of their disease. In addition, African-American women are significantly more likely to be diagnosed with regional or distant breast cancer than are white women. This racial difference in mortality rates may be related to insufficient use of screening measures and lack of access to health care.

During a history interview, a female patient states that she has noticed a few drops of clear discharge from her right nipple. What should the nurse do next? a. Immediately contact the physician to report the discharge. b. Ask her if she is possibly pregnant. c. Ask the patient some additional questions about the medications she is taking. d. Immediately obtain a sample for culture and sensitivity testing.

ANS: c. Ask her some additional questions about the medications she is taking. The use of some medications, such as oral contraceptives, phenothiazines, diuretics, digitalis, steroids, methyldopa, and calcium channel blockers, may cause clear nipple discharge. Bloody or blood-tinged discharge from the nipple, not clear, is significant, especially if a lump is also present. In the pregnant female, colostrum would be a thick, yellowish liquid, and it would be expressed after the fourth month of pregnancy.

In examining a 70-year-old male patient, the nurse notices that he has bilateral gynecomastia. Which of the following describes the nurse's best course of action? a. Recommend that he make an appointment with his physician for a mammogram. b. Ignore it. Benign breast enlargement in men is not unusual. c. Explain that this condition may be the result of hormonal changes, and recommend that he see his physician. d. Explain that gynecomastia in men is usually associated with prostate enlargement and recommend that he be thoroughly screened.

ANS: c. Explain that this condition may be the result of hormonal changes and recommend that he see his physician. Gynecomastia may reappear in the aging male and may be due to testosterone deficiency.

During the physical examination, the nurse notices that a female patient has an inverted left nipple. Which statement regarding this is most accurate? a. Normal nipple inversion is usually bilateral. b. Unilateral inversion of a nipple is always a serious sign. c. Whether the inversion is a recent change should be determined. d. Nipple inversion is not significant unless accompanied by an underlying palpable mass.

ANS: c. It should be determined whether the inversion is a recent change. The nurse should distinguish a recently retracted nipple from one that has been inverted for many years or since puberty. Normal nipple inversion may be unilateral or bilateral and usually can be pulled out (i.e., it is not fixed). Recent nipple retraction signifies acquired disease. See Table 17-3.

A woman is in the family planning clinic seeking birth control information. She states that her breasts "change all month long" and that she is worried that this is unusual. What is the nurse's best response? a. Continual changes in her breasts are unusual. The breasts of nonpregnant women usually stay pretty much the same all month long. b. Breast changes in response to stress are very common and that she should assess her life for stressful events. c. Because of the changing hormones during the monthly menstrual cycle, cyclic breast changes are common. d. Breast changes normally occur only during pregnancy and that a pregnancy test is needed at this time.

ANS: c. Tell her that, because of the changing hormones during the monthly menstrual cycle, cyclic breast changes are common. Breasts of the nonpregnant woman change with the ebb and flow of hormones during the monthly menstrual cycle. During the 3 to 4 days before menstruation, the breasts feel full, tight, heavy, and occasionally sore. The breast volume is smallest on days 4 to 7 of the menstrual cycle.

During a physical examination, a 45-year-old woman states that she has had a crusty, itchy rash on her breast for about 2 weeks. In trying to find the cause of the rash, which of these would be important for the nurse to determine? a. "Is the rash raised and red?" b. "Does it appear to be cyclic?" c. "Where did the rash first appear—on the nipple, the areola, or the surrounding skin?" d. "What was she doing when she first noticed the rash, and do her actions make it worse?"

ANS: c. Where did it first appear—on the nipple, the areola, or the surrounding skin? It is important for the nurse to determine where the rash first appeared. Paget's disease starts with a small crust on the nipple apex and then spreads to the areola. Eczema or other dermatitis rarely starts at nipple unless it results from breastfeeding. It usually starts on the areola or surrounding skin and then spreads to the nipple. See Table 17-6.

The nurse notices that a woman in an exercise class is unable to jump rope. The nurse knows that to jump rope, one's shoulder has to be capable of: A) inversion. B) supination. C) protraction. D) circumduction.

ANS: circumduction. Circumduction is defined as moving the arm in a circle around the shoulder.

A 67-year-old patient states that he recently began to have pain in his left calf when climbing the 10 stairs to his apartment. This pain is relieved by sitting for about 2 minutes; then he is able to resume his activities. The nurse interprets that this patient is most likely experiencing: a. Claudication. b. Sore muscles. c. Muscle cramps. d. Venous insufficiency.

ANS: claudication. Intermittent claudication feels like a "cramp" and is usually relieved by rest within 2 minutes. The other responses are not correct.

While inspecting a patient's breasts, the nurse finds that the left breast is slightly larger than the right with the presence of Montgomery's glands bilaterally. The nurse should: a. Palpate over the Montgomery glands, checking for drainage. b. Consider these findings as normal, and proceed with the examination. c. Ask extensive health history questions regarding the woman's breast asymmetry. d. Continue with the examination, and then refer the patient for further evaluation of the Montgomery glands.

ANS: consider these normal findings and proceed with the examination. Normal findings of the breast include one breast (most often the left) slightly larger than the other and the presence of Montgomery's glands across the areola.

When performing a peripheral vascular assessment on a patient, the nurse is unable to palpate the ulnar pulses. The patient's skin is warm and capillary refill time is normal. The nurse should next: a. Check for the presence of claudication. b. Refer the individual for further evaluation. c. Consider this finding as normal, and proceed with the peripheral vascular evaluation. d. Ask the patient if he or she has experienced any unusual cramping or tingling in the arm.

ANS: consider this a normal finding and proceed with the peripheral vascular evaluation. It is not usually necessary to palpate the ulnar pulses. The ulnar pulses are often not palpable in the normal person. The other responses are not correct.

When assessing a patient's lungs, the nurse recalls that the left lung: a. Consists of two lobes. b. Is divided by the horizontal fissure. c. Primarily consists of an upper lobe on the posterior chest. d. Is shorter than the right lung because of the underlying stomach.

ANS: consists of two lobes. The left lung has two lobes, and the right lung has three lobes. The right lung is shorter than the left lung because of the underlying liver. The left lung is narrower than the right lung because the heart bulges to the left. The posterior chest is almost all lower lobe.

During palpation of the anterior chest wall, the nurse notices a coarse, crackling sensation over the skin surface. On the basis of these findings, the nurse suspects: a. Tactile fremitus. b. Crepitus. c. Friction rub. d. Adventitious sounds.

ANS: crepitus. Crepitus is a coarse, crackling sensation palpable over the skin surface. It occurs in subcutaneous emphysema when air escapes from the lung and enters the subcutaneous tissue, as after open thoracic injury or surgery.

The nurse is reviewing risk factors for breast cancer. Which of these women have risk factors that place them at a higher risk for breast cancer? a. 37 year old who is slightly overweight b. 42 year old who has had ovarian cancer c. 45 year old who has never been pregnant d. 65 year old whose mother had breast cancer

ANS: d. 65 year old whose mother had breast cancer Risk factors for breast cancer include having a first-degree relative with breast cancer (mother, sister, or daughter) and being older than 50 years. Refer to Table 17- 2 for other risk factors.

A 65-year-old patient remarks that she just can't believe that her breasts sag so much. She states it must be from lack of exercise. What explanation should the nurse offer her? a. Only women with large breasts experience sagging. b. Sagging is usually due to decreased muscle mass within the breast. c. A diet that is high in protein will help maintain muscle mass, which keeps the breasts from sagging. d. The glandular and fat tissue atrophies, causing breast size and elasticity to diminish, resulting in breasts that sag.

ANS: d. After menopause, the glandular and fat tissue atrophies, causing breast size and elasticity to diminish, resulting in breasts that sag. After menopause, the glandular tissue atrophies and is replaced with connective tissue. The fat envelope atrophies also, beginning in the middle years and becoming marked in the eighth and ninth decades. These changes decrease breast size and elasticity, so the breasts droop and sag, looking flattened and flabby.

The nurse is performing a breast examination. Which of these statements best describes the correct procedure to use when screening for nipple and skin retraction during a breast examination? Have the woman: a. Bend over and touch her toes. b. Lie down on her left side and notice any retraction. c. Shift from a supine position to a standing position, and note any lag or retraction. d. Slowly lift her arms above her head, and note any retraction or lag in movement.

ANS: d. slowly lift her arms above her head and note any retraction or lag in movement Direct the woman to change position while checking the breasts for skin retraction signs. First ask her to lift her arms slowly over her head. Both breasts should move up symmetrically. Retraction signs are due to fibrosis in the breast tissue, usually caused by growing neoplasms. The nurse should notice if there is a lag in movement of one breast.

The nurse knows that a normal finding when assessing the respiratory system of an elderly adult is: a. Increased thoracic expansion. b. Decreased mobility of the thorax. c. Decreased anteroposterior diameter. d. Bronchovesicular breath sounds throughout the lungs.

ANS: decreased mobility of the thorax. The costal cartilages become calcified with aging, resulting in a less mobile thorax. Chest expansion may be somewhat decreased, and the chest cage commonly shows an increased anteroposterior diameter.

A patient has been admitted to the hospital with vertebral fractures related to osteoporosis. She is in extreme pain. This type of pain would be classified as:

ANS: deep somatic. Deep somatic pain comes from sources such as the blood vessels, joints, tendons, muscles, and bone. Referred pain is felt at one site but originates from another location. Cutaneous pain is derived from the skin surface and subcutaneous tissues. Visceral pain originates from the larger, interior organs.

The primary muscles of respiration include the: a. Diaphragm and intercostals. b. Sternomastoids and scaleni. c. Trapezii and rectus abdominis. d. External obliques and pectoralis major.

ANS: diaphragm and intercostals. The major muscle of respiration is the diaphragm. The intercostal muscles lift the sternum and elevate the ribs during inspiration, increasing the anteroposterior diameter. Expiration is primarily passive. Forced inspiration involves the use of other muscles, such as the accessory neck muscles (sternomastoids, scalene, trapezii). Forced expiration involves the abdominal muscles.

The nurse is percussing over the lungs of a patient with pneumonia. The nurse knows that percussion over an area of atelectasis in the lungs would reveal: a. Dullness. b. Tympany. c. Resonance. d. Hyperresonance.

ANS: dullness. A dull percussion note signals an abnormal density in the lungs, as with pneumonia, pleural effusion, atelectasis, or tumor.

During an assessment, the nurse uses the "profile sign" to detect: a. Pitting edema. b. Early clubbing. c. Symmetry of the fingers. d. Insufficient capillary refill.

ANS: early clubbing. The nurse should use the profile sign (viewing the finger from the side) to detect early clubbing.

The nurse is reviewing the characteristics of breath sounds. Which statement about bronchovesicular breath sounds is true? They are: a. Musical in quality. b. Usually caused by a pathologic disease. c. Expected near the major airways. d. Similar to bronchial sounds except shorter in duration.

ANS: expected near the major airways. Bronchovesicular sounds are heard over major bronchi where fewer alveoli are located: posteriorly, between the scapulae, especially on the right; anteriorly, around the upper sternum in the first and second intercostal spaces. The other responses are not correct.

Which statement about the apices of the lungs is true? The apices of the lungs: a. Are at the level of the second rib anteriorly. b. Extend 3 to 4 cm above the inner third of the clavicles. c. Are located at the sixth rib anteriorly and the eighth rib laterally. d. Rest on the diaphragm at the fifth intercostal space in the midclavicular line (MCL).

ANS: extend 3 to 4 cm above the inner third of the clavicles. The apex of the lung on the anterior chest is 3 to 4 cm above the inner third of the clavicles. On the posterior chest, the apices are at the level of C7.

The nurse is assessing a patient's ischial tuberosity. To palpate the ischial tuberosity, the nurse knows that it is best to have the patient: A) stand. B) flex the hip. C) flex the knee. D) in the supine position.

ANS: flex the hip. The ischial tuberosity lies under the gluteus maximus muscle and is palpable when the hip is flexed.

A patient tells the nurse that she is having a hard time bringing her hand to her mouth when she eats or tries to brush her teeth. The nurse knows that for her to move her hand to her mouth, she must perform which movement? A) flexion. B) abduction. C) adduction. D) extension.

ANS: flexion. Flexion, or bending a limb at a joint, would be required to move the hand to the mouth. Extension is straightening a limb at a joint. Moving a limb toward the midline of the body is called adduction; abduction is moving a limb away from the midline of the body.

A patient's annual physical examination reveals a lateral curvature of the thoracic and lumbar segments of his spine; however, this curvature disappears with forward bending. The nurse knows that this abnormality of the spine is called: A) structural scoliosis. B) functional scoliosis. C) herniated nucleus pulposus. D) dislocated hip.

ANS: functional scoliosis. Functional scoliosis is flexible; it is apparent with standing and disappears with forward bending. Structural scoliosis is fixed; the curvature shows both when standing and when bending forward. See Table 22-7 for description of herniated nucleus pulposus. These findings are not indicative of a dislocated hip.

A patient tells the nurse that "all my life I've been called 'knock knees.'" The nurse knows that another term for "knock knees" is: A) genu varum. B) genu valgum. C) pes planus. D) metatarsus adductus.

ANS: genu valgum. Genu valgum is also known as "knock knees" and is present when there is more than 2.5 cm between the medial malleoli when the knees are together.

The nurse is providing patient education for a man who has been diagnosed with a rotator cuff injury. The nurse knows that a rotator cuff injury involves the: A) nucleus pulposus. B) articular process. C) medial epicondyle. D) glenohumeral joint.

ANS: glenohumeral joint. A rotator cuff injury involves the glenohumeral joint, which is enclosed by a group of four powerful muscles and tendons that support and stabilize it. The nucleus pulposus is located in the center of each intervertebral disk. The medial epicondyle is located at the elbow.

The nurse is examining the hip area of a patient and palpates a flat depression on the upper, lateral side of the thigh when the patient is standing. The nurse interprets this finding as the: A) ischial tuberosity. B) greater trochanter. C) iliac crest. D) gluteus maximus muscle.

ANS: greater trochanter. The greater trochanter of the femur is palpated when the person is standing, and it appears as a flat depression on the upper lateral side of the thigh. The iliac crest is the upper part of the hip bone; the ischial tuberosity lies under the gluteus maximus muscle and is palpable when the hip is flexed. The gluteus muscle is part of the buttocks.

During an examination, the nurse asks a patient to bend forward from the waist and notices that the patient has lateral tilting. When his leg is raised straight up, he complains of a pain going down his buttock into his leg. The nurse suspects: A) scoliosis. B) meniscus tear. C) herniated nucleus pulposus. D) spasm of paravertebral muscles.

ANS: herniated nucleus pulposus. Lateral tilting and sciatic pain with straight leg raising are findings that occur with a herniated nucleus pulposus. The other options are not correct.

The nurse suspects that a patient has carpal tunnel syndrome and wants to perform the Phalen's test. To perform this test, the nurse should instruct the patient to: A) dorsiflex the foot. B) plantarflex the foot. C) hold both hands back to back while flexing the wrists 90 degrees for 60 seconds. D) hyperextend the wrists with the palmar surface of both hands touching and wait for 60 seconds.

ANS: hold both hands back to back while flexing the wrists 90 degrees for 60 seconds. For the Phalen's test, the nurse should ask the person to hold both hands back to back while flexing the wrists 90 degrees. Acute flexion of the wrist for 60 seconds produces no symptoms in the normal hand. The Phalen's test reproduces numbness and burning in a person with carpal tunnel syndrome. The other actions are not correct for testing for carpal tunnel syndrome.

During percussion, the nurse knows that a dull percussion note elicited over a lung lobe most likely results from: a. Shallow breathing. b. Normal lung tissue. c. Decreased adipose tissue. d. Increased density of lung tissue.

ANS: increased density of lung tissue. A dull percussion note indicates an abnormal density in the lungs, as with pneumonia, pleural effusion, atelectasis, or tumor. Resonance is the expected finding in normal lung tissue.

The nurse is explaining to a patient that there are "shock absorbers" in his back to cushion the spine and to help it move. The nurse is referring to his: A) vertebral column. B) nucleus pulposus. C) vertebral foramen. D) intervertebral disks.

ANS: intervertebral disks. Intervertebral disks are elastic fibrocartilaginous plates that cushion the spine like shock absorbers and help it move. The vertebral column is the spinal column itself. The nucleus pulposus is located in the center of each disk. The vertebral foramen is the channel, or opening, for the spinal cord in the vertebrae.

The nurse is reviewing the technique of palpating for tactile fremitus with a new graduate. Which statement by the graduate nurse reflects a correct understanding of tactile fremitus? "Tactile remits: a. "Is caused by moisture in the alveoli." b. "Indicates that air is present in the subcutaneous tissues." c. "Is caused by sounds generated from the larynx." d. "Reflects the blood flow through the pulmonary arteries."

ANS: is caused by sounds generated from the larynx." Fremitus is a palpable vibration. Sounds generated from the larynx are transmitted through patent bronchi and the lung parenchyma to the chest wall where they are felt as vibrations. Crepitus is the term for air in the subcutaneous tissues.

A 65-year-old patient is experiencing pain in his left calf when he exercises that disappears after resting for a few minutes. The nurse recognizes that this description is most consistent with _________ the left leg. a. Venous obstruction of b. Claudication due to venous abnormalities in c. Ischemia caused by a partial blockage of an artery supplying d. Ischemia caused by the complete blockage of an artery supplying

ANS: ischemia caused by partial blockage of an artery supplying Ischemia is a deficient supply of oxygenated arterial blood to a tissue. A partial blockage creates an insufficient supply, and the ischemia may be apparent only during exercise when oxygen needs increase.

The functional units of the musculoskeletal system are the: A) joints. B) bones. C) muscles. D) tendons.

ANS: joints. Joints are the functional units of the musculoskeletal system because they permit the mobility needed for the activities of daily living. The skeleton (bones) is the framework of the body.

When evaluating a patient's pain, the nurse knows that an example of acute pain would be:

ANS: kidney stones. Acute pain is short-term and dissipates after an injury heals, such as with kidney stones. The other conditions are examples of chronic pain where the pain continues for 6 months or longer and does not stop when the injury heals.

A mother brings her newborn baby boy in for a checkup; she tells the nurse that he doesn't seem to be moving his right arm as much as his left and that he seems to have pain when she lifts him up under the arms. The nurse suspects a fractured clavicle and would observe for: A) a negative Allis test. B) a positive Ortolani's sign. C) limited range of motion during the Moro's reflex. D) limited range of motion during Lasègue's test

ANS: limited range of motion during the Moro's reflex. For a fractured clavicle, the nurse should observe for limited arm range of motion and unilateral response to the Moro's reflex. The other tests are not appropriate for this problem.

A woman who is 8 months pregnant comments that she has noticed a change in posture and is having lower back pain. The nurse tells her that during pregnancy women have a posture shift to compensate for the enlarging fetus. This shift in posture is known as: A) lordosis. B) scoliosis. C) ankylosis. D) kyphosis.

ANS: lordosis. Lordosis compensates for the enlarging fetus, which would shift the center of balance forward. This shift in balance in turn creates strain on the low back muscles, felt as low back pain during late pregnancy by some women. Scoliosis is lateral curvature of portions of the spine; ankylosis is extreme flexion of the wrist, as seen with severe rheumatoid arthritis; and kyphosis is an enhanced thoracic curvature of the spine.

A patient has been diagnosed with osteoporosis and asks the nurse, "What is osteoporosis?" The nurse explains to the patient that osteoporosis is defined as: A) increased bone matrix. B) loss of bone density. C) new, weaker bone growth. D) increased phagocytic activity.

ANS: loss of bone density. After age 40, loss of bone matrix (resorption) occurs more rapidly than new bone formation. The net effect is a gradual loss of bone density, or osteoporosis. The other options are not correct.

A professional tennis player comes into the clinic complaining of a sore elbow. The nurse will assess for tenderness at the: A) olecranon bursa. B) annular ligament. C) base of the radius. D) medial and lateral epicondyle.

ANS: medial and lateral epicondyle. The epicondyles, the head of radius, and tendons are common sites of inflammation and local tenderness, or "tennis elbow." The other locations are not affected.

The nurse is examining a 3-month-old infant. While holding the thumbs on the infant's inner mid thighs and the fingers outside on the hips, touching the greater trochanter, the nurse adducts the legs until the nurse's thumbs touch and then abducts the legs until the infant's knees touch the table. The nurse does not notice any "clunking" sounds and is confident to record a: A) positive Allis test. B) negative Allis test. C) positive Ortolani's sign. D) negative Ortolani's sign.

ANS: negative Ortolani's sign. Normally this maneuver feels smooth and has no sound. With a positive Ortolani's sign, the nurse will feel and hear a "clunk" as the head of the femur pops back into place. A positive Ortolani's sign reflects hip instability. The Allis test also tests for hip dislocation, but is done by comparing leg lengths.

A patient has hard, nonpitting edema of the left lower leg and ankle. The right leg has no edema. Based on these findings, the nurse recalls that: a. Nonpitting, hard edema occurs with lymphatic obstruction. b. Alterations in arterial function will cause edema. c. Phlebitis of a superficial vein will cause bilateral edema. d. Long-standing arterial obstruction will cause pitting edema.

ANS: nonpitting, hard edema occurs with lymphatic obstruction Unilateral edema occurs with occlusion of a deep vein and with unilateral lymphatic obstruction. With these factors, the edema is nonpitting and feels hard to the touch (brawny edema).

A teenage girl has arrived complaining of pain in her left wrist. She was playing basketball when she fell and landed on her left hand. The nurse examines her hand and would expect a fracture if the girl complains: A) of a dull ache. B) that the pain in her wrist is deep. C) of sharp pain that increases with movement. D) of dull throbbing pain that increases with rest.

ANS: of sharp pain that increases with movement A fracture causes sharp pain that increases with movement. The other pains do not occur with a fracture.

An 85-year-old patient comments during his annual physical that he seems to be getting shorter as he ages. The nurse should explain that decreased height occurs with aging because: A) long bones tend to shorten with age. B) of the shortening of the vertebral column. C) there is a significant loss of subcutaneous fat. D) there is a thickening of the intervertebral disks.

ANS: of the shortening of the vertebral column. Postural changes are evident with aging; decreased height is most noticeable and is due to shortening of the vertebral column. Long bones do not shorten with age. Intervertebral disks actually get thinner with age. Subcutaneous fat is not lost but is redistributed to the abdomen and hips.

A 68-year-old woman has come in for an assessment of her rheumatoid arthritis, and the nurse notices raised, firm, nontender nodules at the olecranon bursa and along the ulna. These nodules are most commonly diagnosed as: A) epicondylitis. B) gouty arthritis. C) olecranon bursitis. D) subcutaneous nodules.

ANS: olecranon bursitis. Subcutaneous nodules are raised, firm, and nontender and occur with rheumatoid arthritis in the olecranon bursa and along the extensor surface of the ulna. See Table 22-3 for a description of the other conditions.

When assessing a patient's pulse, the nurse notes that the amplitude is weaker during inspiration and stronger during expiration. When the nurse measures the blood pressure, the reading decreases 20 mm Hg during inspiration and increases with expiration. This patient is experiencing pulses: a. Alternans. b. Bisferiens. c. Bigeminus. d. Paradoxus.

ANS: paradoxus. In pulsus paradoxus, beats have a weaker amplitude with inspiration and a stronger amplitude with expiration. It is best determined during blood pressure measurement; reading decreases (>10 mm Hg) during inspiration and increases with expiration.

While examining a 75-year-old woman, the nurse notices that the skin over her right breast is thickened and the hair follicles are exaggerated. This condition is known as: a. Dimpling. b. Retraction. c. Peau d'orange. d. Benign breast disease.

ANS: peau d'orange. This condition is known as peau d'orange. Lymphatic obstruction produces edema, which thickens the skin and exaggerates the hair follicles. The skin has a pig-skin or orange-peel look, and this condition suggests cancer.

During a neonatal examination, the nurse notices that the newborn infant has six toes. This finding is documented as: A) unidactyly. B) syndactyly. C) polydactyly. D) multidactyly.

ANS: polydactyly. Polydactyly is the presence of extra fingers or toes. Syndactyly is webbing between adjacent fingers or toes. The other terms are not correct.

A patient comes to the clinic complaining of a cough that is worse at night but not as bad during the day. The nurse recognizes that this may indicate: a. Pneumonia. b. Postnasal drip or sinusitis. c. Exposure to irritants at work. d. Chronic bronchial irritation from smoking.

ANS: postnasal drip or sinusitis. A cough that occurs mainly at night may indicate postnasal drip or sinusitis. Exposure to irritants at work causes an afternoon or evening cough. Smokers experience early morning coughing. Coughing associated with acute illnesses such as pneumonia is continuous throughout the day.

A patient complains of leg pain that wakes him at night. He states that he "has been having problems" with his legs. He has pain in his legs when they are elevated that disappears when he dangles them. He recently noticed "a sore" on the inner aspect of the right ankle. On the basis of this history information, the nurse interprets that the patient is most likely experiencing: a. Pain related to lymphatic abnormalities. b. Problems related to arterial insufficiency. c. Problems related to venous insufficiency. d. Pain related to musculoskeletal abnormalities.

ANS: problems related to arterial insufficiency. Night leg pain is common in aging adults. It may indicate the ischemic rest pain of peripheral vascular disease. Alterations in arterial circulation cause pain that becomes worse with leg elevation and is eased when the extremity is dangled.

A mother brings her 3-month-old infant to the clinic for evaluation of a cold. She tells the nurse that he had "a runny nose for a week." When performing the physical assessment, the nurse notes that the child has nasal flaring and sternal and intercostal retractions. The nurse's next action should be to: a. Assure the mother that these signs are normal symptoms of a cold. b. Recognize that these are serious signs, and contact the physician. c. Ask the mother if the infant has had trouble with feedings. d. Perform a complete cardiac assessment because these signs are probably indicative of early heart failure.

ANS: recognize that these are serious signs and contact the physician. The infant is an obligatory nose breather until the age of 3 months. Normally there is no flaring of the nostrils and no sternal or intercostal retraction. Marked retractions of the sternum and intercostal muscles and nasal flaring indicate increased inspiratory effort, as in pneumonia, acute airway obstruction, asthma, and atelectasis; therefore, immediate referral to the physician is warranted. These signs do not indicate heart failure, and assessment of the infant's feeding is not a priority at this time.

A patient is able to flex his right arm forward without difficulty or pain but is unable to abduct his arm because of pain and muscle spasms; the nurse should suspect: A) crepitation. B) rotator cuff lesions. C) dislocated shoulder. D) rheumatoid arthritis.

ANS: rotator cuff lesions. Rotator cuff lesions may cause limited range of motion and pain and muscle spasm during abduction, whereas forward flexion stays fairly normal. The other options are not correct.

The nurse is observing the auscultation technique of another nurse. The correct method to use when progressing from one auscultatory site on the thorax to another is ____ comparison. a. Side-to-side b. Top-to-bottom c. Posterior-to-anterior d. Interspace-by-interspace

ANS: side-to-side Side-to-side comparison is most important when auscultating the chest. The nurse should listen to at least one full respiration in each location. The other techniques are incorrect.

The nurse is teaching a class on osteoporosis prevention to a group of postmenopausal woman. A participant shows that she needs more instruction when she states, "I will: A) start swimming to increase my weight-bearing exercise." B) try to stop smoking as soon as possible." C) check with my doctor about taking calcium supplements." D) get a bone-density test soon."

ANS: start swimming to increase my weight-bearing exercise." Weight-bearing exercises include walking, low-impact aerobics, dancing, or stationary cycling. Swimming is not considered a weight-bearing exercise. The other responses are correct.

During an examination of the anterior thorax, the nurse keeps in mind that the trachea bifurcates anteriorly at the: a. Costal angle. b. Sternal angle. c. Xiphoid process. d. Suprasternal notch.

ANS: sternal angle. The sternal angle marks the site of tracheal bifurcation into the right and left main bronchi; it corresponds with the upper border of the atria of the heart, and it lies above the fourth thoracic vertebra on the back.

The nurse is assessing a patient's pain. The nurse knows that the most reliable indicator of pain would be the:

ANS: subjective report. The subjective report is the most reliable indicator of pain. Physical examination findings can lend support, but the clinician cannot base the diagnosis of pain exclusively on physical assessment findings.

The nurse is assessing a 1-week-old infant and is testing his muscle strength. The nurse lifts the infant with hands under the axillae and notices that the infant starts to "slip" between the hands. The nurse should: A) suspect a fractured clavicle. B) suspect that the infant may have a deformity of the spine. C) suspect that the infant may have weakness of the shoulder muscles. D) consider this a normal finding because the musculature of an infant this age is undeveloped.

ANS: suspect that the infant may have weakness of the shoulder muscles. An infant who starts to "slip" between the nurse's hands shows weakness of the shoulder muscles. An infant with normal muscle strength wedges securely between the nurse's hands. The other responses are not correct.

The nurse has completed the musculoskeletal examination of a patient's knee and has found a positive bulge sign. The nurse interprets this finding to indicate: A) irregular bony margins. B) soft tissue swelling in the joint. C) swelling from fluid in the epicondyle. D) swelling from fluid in the suprapatellar pouch.

ANS: swelling from fluid in the suprapatellar pouch. For swelling in the suprapatellar pouch, the bulge sign confirms the presence of fluid. The other options are not correct.

The ankle joint is the articulation of the tibia, the fibula, and the: A) talus. B) cuboid. C) calcaneus. D) cuneiform bones.

ANS: talus. The ankle or tibiotalar joint is the articulation of the tibia, fibula, and talus. The other bones listed are foot bones, but not part of the ankle joint.

The articulation of the mandible and the temporal bone is known as the: A) intervertebral foramen. B) condyle of the mandible. C) temporomandibular joint. D) zygomatic arch of the temporal bone.

ANS: temporomandibular joint. The articulation of the mandible and the temporal bone is the temporomandibular joint. The other responses are not correct.

The nurse knows that auscultation of fine crackles would most likely be noticed in: a. A healthy 5-year-old child. b. A pregnant woman. c. The immediate newborn period. d. Association with a pneumothorax.

ANS: the immediate newborn period. Fine crackles are commonly heard in the immediate newborn period as a result of the opening of the airways and clearing of fluid. Persistent fine crackles would be noticed with pneumonia, bronchiolitis, or atelectasis.

The nurse has palpated a lump in a female patient's right breast. The nurse documents this as a small, round, firm, distinct, lump located at 2 o'clock, 2 cm from the nipple. It is nontender and fixed. There is no associated retraction of skin or nipple, no erythema, and no axillary lymphadenopathy. Which of these statements reveals the information that is missing from the documentation? It is missing information about: a. Shape of the lump b. Consistency of the lump c. Size of the lump d. Whether the lump is solitary or multiple

ANS: the size of the lump. If the nurse feels a lump or mass, he or she should note these characteristics: (1) location, (2) size—judge in centimeters in three dimensions: width length thickness, (3) shape, (4) consistency, (5) motility, (6) distinctness, (7) nipple, (8) the skin over the lump, (9) tenderness, and (10) lymphadenopathy.

A 60-year-old woman has developed reflexive sympathetic dystrophy after arthroscopic repair of her shoulder. A key feature of this condition is that:

ANS: the slightest touch, such as a sleeve brushing against her arm, causes severe, intense pain. A key feature of reflexive sympathetic dystrophy is that a typically innocuous stimulus can create a severe, intensely painful response. The affected extremity becomes less functional over time.

A man who has had gout for several years comes to the clinic with a problem with his toe. On examination, the nurse notices the presence of hard, painless nodules over the great toe; one has burst open with a chalky discharge. This finding is known as: A) a callus. B) a plantar wart. C) a bunion. D) tophi.

ANS: tophi. Tophi are collections of sodium urate crystals resulting from chronic gout in and around the joint that cause extreme swelling and joint deformity. They appear as hard, painless nodules (tophi) over the metatarsophalangeal joint of the first toe and they sometimes burst with a chalky discharge (See Table 22-6). See Table 22-6 for descriptions of the other conditions.

A 35-year-old recent immigrant is being seen in the clinic for complaints of a cough that is associated with rust-colored sputum, low-grade afternoon fevers, and night sweats for the past 2 months. The nurse's preliminary analysis, based on this history, is that this patient may be suffering from: a. Bronchitis. b. Pneumonia. c. Tuberculosis. d. Pulmonary edema.

ANS: tuberculosis. Sputum is not diagnostic alone, but some conditions have characteristic sputum production. Tuberculosis often produces rust-colored sputum in addition to other symptoms of night sweats and low-grade afternoon fevers. See Table 18-8.

A woman who has had rheumatoid arthritis for years is starting to notice that her fingers are drifting to the side. The nurse knows that this condition is commonly referred to as: A) radial drift. B) ulnar deviation. C) swan neck deformity. D) Dupuytren's contracture.

ANS: ulnar deviation. Fingers drift to the ulnar side because of stretching of the articular capsule and muscle imbalance caused by chronic rheumatoid arthritis. Radial drift is not seen. See Table 22-4 for descriptions of swan neck deformity and Dupuytren's contracture.

During an assessment the nurse has elevated a patient's legs 12 inches off the table and has had him wag his feet to drain off venous blood. After helping him to sit up and dangle his legs over the side of the table, the nurse should expect a normal finding at this point would be: a. Significant elevational pallor. b. Venous filling within 15 seconds. c. No change in the coloration of the skin. d. Color returning to the feet within 20 seconds of assuming a sitting position.

ANS: venous filling within 15 seconds. In this test it normally takes 10 seconds or less for the color to return to the feet and 15 seconds for the veins of the feet to fill. Marked elevational pallor as well as delayed venous filling occurs with arterial insufficiency.

In performing a breast examination, the nurse knows that it is especially important to examine the upper outer quadrant of the breast. The reason for this is that the upper outer quadrant is: a. The largest quadrant of the breast. b. The location of most breast tumors. c. Where most of the suspensory ligaments attach. d. More prone to injury and calcifications than other locations in the breast.

ANS:b. the location of most breast tumors. The upper outer quadrant is the site of most breast tumors. In the upper outer quadrant, the nurse should notice the axillary tail of Spence, the cone-shaped breast tissue that projects up into the axilla, close to the pectoral group of axillary lymph nodes.

A patient tells the nurse that he is very nervous, is nauseated, and feels hot. These types of data would be: a. Objective. b. Reflective. c. Subjective. d. Introspective.

C Subjective data are what the person says about him or herself during history taking. Objective data are what the health professional observes by inspecting, percussing, palpating, and auscultating during the physical examination. The terms reflective and introspective are not used to describe data.

The nurse is assessing the neurologic status of a patient who has a late-stage brain tumor. With the reflex hammer, the nurse draws a light stroke up the lateral side of the sole of the foot and inward, across the ball of the foot. In response, the patient's toes fan out, and the big toe shows dorsiflexion. The nurse interprets this result as: a.Negative Babinski sign, which is normal for adults. b.Positive Babinski sign, which is abnormal for adults. c.Clonus, which is a hyperactive response. d.Achilles reflex, which is an expected response.

B Dorsiflexion of the big toe and fanning of all toes is a positive Babinski sign, also calledup-going toes. This response occurs with upper motor neuron disease of the corticospinal (or pyramidal) tract and is an abnormal finding for adults.

The nurse is assessing a 16-year-old patient who has suffered head injuries from a recent motor vehicle accident. Which of these statements indicates the most important reason for assessing for any drainage from the ear canal? a. If the drum has ruptured, then purulent drainage will result. b. Bloody or clear watery drainage can indicate a basal skull fracture. c. The auditory canal many be occluded from increased cerumen. d. Foreign bodies from the accident may cause occlusion of the canal.

B Frank blood or clear watery drainage (cerebrospinal leak) after a trauma suggests a basal skull fracture and warrants immediate referral. Purulent drainage indicates otitis externa or otitis media.

The nurse is performing a functional assessment on an 82-year-old patient who recently had a stroke. Which of these questions would be most important to ask? a."Do you wear glasses?" b."Are you able to dress yourself?" c."Do you have any thyroid problems?" d."How many times a day do you have a bowel movement?"

B Functional assessment measures how a person manages day-to-day activities. For the older person, the meaning of health becomes those activities that they can or cannot do. The other responses do not relate to functional assessment.

A patient in her first trimester of pregnancy is diagnosed with rubella. Which of these statements is correct regarding the significance of this in relation to the infant's hearing? a. Rubella may affect the mother's hearing but not the infant's. b. Rubella can damage the infant's organ of Corti, which will impair hearing. c. Rubella is only dangerous to the infant in the second trimester of pregnancy. d. Rubella can impair the development of CN VIII and thus affect hearing.

B If maternal rubella infection occurs during the first trimester, then it can damage the organ of

The nurse is conducting a class for new graduate nurses. During the teaching session, the nurse should keep in mind that novice nurses, without a background of skills and experience from which to draw, are more likely to make their decisions using: a. Intuition. b. A set of rules. c. Articles in journals. d. Advice from supervisors.

B Novice nurses operate from a set of defined, structured rules. The expert practitioner uses intuitive links.

A 31-year-old patient tells the nurse that he has noticed pain in his left ear when people speak loudly to him. The nurse knows that this finding: a. Is normal for people of his age. b. Is a characteristic of recruitment. c. May indicate a middle ear infection. d. Indicates that the patient has a cerumen impaction.

B Recruitment is significant hearing loss occurring when speech is at low intensity, but sound actually becomes painful when the speaker repeats at a louder volume. The other responses are not correct.

A patient is brought by ambulance to the emergency department with multiple traumas received in an automobile accident. He is alert and cooperative, but his injuries are quite severe. How would the nurse proceed with data collection? a. Collect history information first, then perform the physical examination and institute life-saving measures. b. Simultaneously ask history questions while performing the examination and initiating life-saving measures. c. Collect all information on the history form, including social support patterns, strengths, and coping patterns. d. Perform life-saving measures and delay asking any history questions until the patient is transferred to the intensive care unit.

B The emergency data base calls for a rapid collection of the data base, often concurrently compiled with life-saving measures. The other responses are not appropriate for the situation.

When examining the ear with an otoscope, the nurse notes that the tympanic membrane should appear: a. Light pink with a slight bulge. b. Pearly gray and slightly concave. c. Pulled in at the base of the cone of light. d. Whitish with a small fleck of light in the superior portion.

B The tympanic membrane is a translucent membrane with a pearly gray color and a prominent cone of light in the anteroinferior quadrant, which is the reflection of the otoscope light. The tympanic membrane is oval and slightly concave, pulled in at its center by the malleus, which is one of the middle ear ossicles.

A 17-year-old student is a swimmer on her high school's swim team. She has had three bouts of otitis externa this season and wants to know what to do to prevent it. The nurse instructs her to: a. Use a cotton-tipped swab to dry the ear canals thoroughly after each swim. b. Use rubbing alcohol or 2% acetic acid eardrops after every swim. c. Irrigate the ears with warm water and a bulb syringe after each swim. d. Rinse the ears with a warmed solution of mineral oil and hydrogen peroxide.

B With otitis externa (swimmer's ear), swimming causes the external canal to become waterlogged and swell; skinfolds are set up for infection. Otitis externa can be prevented by using rubbing alcohol or 2% acetic acid eardrops after every swim.

During an assessment of a 22-year-old woman who sustained a head injury from an automobile accident 4 hours earlier, the nurse notices the following changes: pupils were equal, but now the right pupil is fully dilated and nonreactive, and the left pupil is 4 mm and reacts to light. What do these findings suggest? a.Injury to the right eye b.Increased intracranial pressure c.Test inaccurately performed d.Normal response after a head injury

B In a person with a brain injury, a sudden, unilateral, dilated, and nonreactive pupil is ominous. CN III runs parallel to the brainstem. When increasing intracranial pressure pushes down the brainstem (uncal herniation), it puts pressure on CN III, causing pupil dilation. The other responses are incorrect.

During an assessment of a patient's family history, the nurse constructs a genogram. Which statement best describes a genogram? a.List of diseases present in a person's near relatives *b.Graphic family tree that uses symbols to depict the gender, relationship, and age of immediate family members* c.Drawing that depicts the patient's family members up to five generations back d.Description of the health of a person's children and grandchildren

B A genogram (or pedigree) is a graphic family tree that uses symbols to depict the gender, relationship, and age of immediate blood relatives in at least three generations (parents, grandparents, siblings). The other options do not describe a genogram.

During an examination of a patient's abdomen, the nurse notes that the abdomen is rounded and firm to the touch. During percussion, the nurse notes a drumlike quality of the sounds across the quadrants. This type of sound indicates: a.Constipation. b.Air-filled areas. c.Presence of a tumor. d.Presence of dense organs.

B A musical or drumlike sound (tympany) is heard when percussion occurs over an air-filled viscus, such as the stomach or intestines.

The assessment of a 60-year-old patient has taken longer than anticipated. In testing his pain perception, the nurse decides to complete the test as quickly as possible. When the nurse applies the sharp point of the pin on his arm several times, he is only able to identify these as one "very sharp prick." What would be the most accurate explanation for this? a.The patient has hyperesthesia as a result of the aging process. b.This response is most likely the result of the summation effect. c.The nurse was probably not poking hard enough with the pin in the other areas. d.The patient most likely has analgesia in some areas of arm and hyperalgesia in others.

B At least 2 seconds should be allowed to elapse between each stimulus to avoid summation. With summation, frequent consecutive stimuli are perceived as one strong stimulus. The other responses are incorrect.

As part of the health history of a 6-year-old boy at a clinic for a sports physical examination, the nurse reviews his immunization record and notes that his last measles-mumps-rubella (MMR) vaccination was at 15 months of age. What recommendation should the nurse make? a.No further MMR immunizations are needed. *b.MMR vaccination needs to be repeated at 4 to 6 years of age.* c.MMR immunization needs to be repeated every 4 years until age 21 years. d.A recommendation cannot be made until the physician is consulted.

B Because of recent outbreaks of measles across the United States, the American Academy of Pediatrics (2006) recommends two doses of the MMR vaccine, one at 12 to 15 months of age and one at age 4 to 6 years.

The nurse would use bimanual palpation technique in which situation? a.Palpating the thorax of an infant b.Palpating the kidneys and uterus c.Assessing pulsations and vibrations d.Assessing the presence of tenderness and pain

B Bimanual palpation requires the use of both hands to envelop or capture certain body parts or organs such as the kidneys, uterus, or adnexa. The other situations are not appropriate for bimanual palpation.

A 40-year-old patient who has just finished chemotherapy for breast cancer tells the nurse that she is concerned about her mouth. During the assessment the nurse finds areas of buccal mucosa that are raw and red with some bleeding, as well as other areas that have a white, cheesy coating. The nurse recognizes that this abnormality is: a.Aphthous ulcers. b.Candidiasis. c.Leukoplakia. d.Koplik spots.

B Candidiasis is a white, cheesy, curdlike patch on the buccal mucosa and tongue. It scrapes off, leaving a raw, red surface that easily bleeds. It also occurs after the use of antibiotics or corticosteroids and in persons who are immunosuppressed. (See Table 16-4 for descriptions of the other lesions.)

During an examination, the nurse notices severe nystagmus in both eyes of a patient. Which conclusion by the nurse is correct? Severe nystagmus in both eyes: a.Is a normal occurrence. b.May indicate disease of the cerebellum or brainstem. c.Is a sign that the patient is nervous about the examination. d.Indicates a visual problem, and a referral to an ophthalmologist is indicated.

B End-point nystagmus at an extreme lateral gaze normally occurs; however, the nurse should carefully assess any other nystagmuses. Severe nystagmus occurs with disease of the vestibular system, cerebellum, or brainstem.

When examining the mouth of an older patient, the nurse recognizes which finding is due to the aging process? a.Teeth appearing shorter b.Tongue that looks smoother in appearance c.Buccal mucosa that is beefy red in appearance d.Small, painless lump on the dorsum of the tongue

B In the aging adult, the tongue looks smoother because of papillary atrophy. The teeth are slightly yellowed and appear longer because of the recession of gingival margins.

The nurse knows that testing kinesthesia is a test of a person's: a.Fine touch. b.Position sense. c.Motor coordination. d.Perception of vibration.

B Kinesthesia, or position sense, is the persons ability to perceive passive movements of the extremities. The other options are incorrect.

A woman who is in the second trimester of pregnancy mentions that she has had "more nosebleeds than ever" since she became pregnant. The nurse recognizes that this is a result of: a.A problem with the patient's coagulation system. b.Increased vascularity in the upper respiratory tract as a result of the pregnancy. c.Increased susceptibility to colds and nasal irritation. d.Inappropriate use of nasal sprays.

B Nasal stuffiness and epistaxis may occur during pregnancy as a result of increased vascularity in the upper respiratory tract.

In obtaining a health history on a 74-year-old patient, the nurse notes that he drinks alcohol daily and that he has noticed a tremor in his hands that affects his ability to hold things. With this information, what response should the nurse make? a."Does your family know you are drinking every day?" b."Does the tremor change when you drink alcohol?" c."We'll do some tests to see what is causing the tremor." d."You really shouldn't drink so much alcohol; it may be causing your tremor."

B Senile tremor is relieved by alcohol, although not a recommended treatment. The nurse should assess whether the person is abusing alcohol in an effort to relieve the tremor.

The nurse keeps in mind that the most important reason to share information and to offer brief teaching while performing the physical examination is to help the: a.Examiner feel more comfortable and to gain control of the situation. b.Examiner to build rapport and to increase the patient's confidence in him or her. c.Patient understand his or her disease process and treatment modalities. d.Patient identify questions about his or her disease and the potential areas of patient education.

B Sharing information builds rapport and increases the patient's confidence in the examiner. It also gives the patient a little more control in a situation during which feeling completely helpless is often present.

The nurse places a key in the hand of a patient and he identifies it as a penny. What term would the nurse use to describe this finding? a.Extinction b.Astereognosis c.Graphesthesia d.Tactile discrimination

B Stereognosis is the persons ability to recognize objects by feeling their forms, sizes, and weights. Astereognosis is an inability to identify objects correctly, and it occurs in sensory cortex lesions. Tactile discrimination tests fine touch. Extinction tests the persons ability to feel sensations on both sides of the body at the same point.

The nurse is incorporating a person's spiritual values into the health history. Which of these questions illustrates the "community" portion of the FICA (faith and belief, importance and influence, community, and addressing or applying in care) questions? a."Do you believe in God?" b."Are you a part of any religious or spiritual congregation?" c."Do you consider yourself to be a religious or spiritual person?" d."How does your religious faith influence the way you think about your health?"

B The "community" is assessed when the nurse asks whether a person is part of a religious or spiritual community or congregation. The other areas assessed are faith, influence, and addressing any religious or spiritual issues or concerns.

While auscultating heart sounds, the nurse hears a murmur. Which of these instruments should be used to assess this murmur? a.Electrocardiogram b.Bell of the stethoscope c.Diaphragm of the stethoscope d.Palpation with the nurse's palm of the hand

B The bell of the stethoscope is best for soft, low-pitched sounds such as extra heart sounds or murmurs. The diaphragm of the stethoscope is best used for high-pitched sounds such as breath, bowel, and normal heart sounds.

The nurse is using an otoscope to assess the nasal cavity. Which of these techniques is correct? a.Inserting the speculum at least 3 cm into the vestibule b.Avoiding touching the nasal septum with the speculum c.Gently displacing the nose to the side that is being examined d.Keeping the speculum tip medial to avoid touching the floor of the nares

B The correct technique for using an otoscope is to insert the apparatus into the nasal vestibule, avoiding pressure on the sensitive nasal septum. The tip of the nose should be lifted up before inserting the speculum.

The nurse is assessing a patient's skin during an office visit. What part of the hand and technique should be used to best assess the patient's skin temperature? a.Fingertips; they are more sensitive to small changes in temperature. b.Dorsal surface of the hand; the skin is thinner on this surface than on the palms. c.Ulnar portion of the hand; increased blood supply in this area enhances temperature sensitivity. d.Palmar surface of the hand; this surface is the most sensitive to temperature variations because of its increased nerve supply in this area.

B The dorsa (backs) of the hands and fingers are best for determining temperature because the skin is thinner on the dorsal surfaces than on the palms. Fingertips are best for fine, tactile discrimination. The other responses are not useful for palpation.

While performing an assessment of the mouth, the nurse notices that the patient has a 1-cm ulceration that is crusted with an elevated border and located on the outer third of the lower lip. What other information would be most important for the nurse to assess? a.Nutritional status b.When the patient first noticed the lesion c.Whether the patient has had a recent cold d.Whether the patient has had any recent exposure to sick animals

B With carcinoma, the initial lesion is round and indurated, but then it becomes crusted and ulcerated with an elevated border. Most cancers occur between the outer and middle thirds of the lip. Any lesion that is still unhealed after 2 weeks should be referred

Which statement concerning the areas of the brain is true? a.The cerebellum is the center for speech and emotions. b.The hypothalamus controls body temperature and regulates sleep. c.The basal ganglia are responsible for controlling voluntary movements. d.Motor pathways of the spinal cord and brainstem synapse in the thalamus.

B The hypothalamus is a vital area with many important functions: body temperature controller, sleep center, anterior and posterior pituitary gland regulator, and coordinator of autonomic nervous system activity and emotional status. The cerebellum controls motor coordination, equilibrium, and balance. The basal ganglia control autonomic movements of the body. The motor pathways of the spinal cord synapse in various areas of the spinal cord, not in the thalamus.

A mother of a 1-month-old infant asks the nurse why it takes so long for infants to learn to roll over. The nurse knows that the reason for this is: a.A demyelinating process must be occurring with her infant. b.Myelin is needed to conduct the impulses, and the neurons of a newborn are not yet myelinated. c.The cerebral cortex is not fully developed; therefore, control over motor function gradually occurs. d.The spinal cord is controlling the movement because the cerebellum is not yet fully developed.

B The infants sensory and motor development proceeds along with the gradual acquisition of myelin, which is needed to conduct most impulses. Very little cortical control exists, and the neurons are not yet myelinated. The other responses are not correct.

The primary purpose of the ciliated mucous membrane in the nose is to: a.Warm the inhaled air. b.Filter out dust and bacteria. c.Filter coarse particles from inhaled air. d.Facilitate the movement of air through the nares.

B The nasal hairs filter the coarsest matter from inhaled air, whereas the mucous blanket filters out dust and bacteria. The rich blood supply of the nasal mucosa warms the inhaled air.

The two parts of the nervous system are the: a.Motor and sensory. b.Central and peripheral. c.Peripheral and autonomic. d.Hypothalamus and cerebral.

B The nervous system can be divided into two partscentral and peripheral. The central nervous system includes the brain and spinal cord. The peripheral nervous system includes the 12 pairs of cranial nerves (CNs), the 31 pairs of spinal nerves, and all of their branches.

When performing a physical examination, safety must be considered to protect the examiner and the patient against the spread of infection. Which of these statements describes the most appropriate action the nurse should take when performing a physical examination? a.Washing one's hands after removing gloves is not necessary, as long as the gloves are still intact. b.Hands are washed before and after every physical patient encounter. c.Hands are washed before the examination of each body system to prevent the spread of bacteria from one part of the body to another. d.Gloves are worn throughout the entire examination to demonstrate to the patient concern regarding the spread of infectious diseases.

B The nurse should wash his or her hands before and after every physical patient encounter; after contact with blood, body fluids, secretions, and excretions; after contact with any equipment contaminated with body fluids; and after removing gloves. Hands should be washed after gloves have been removed, even if the gloves appear to be intact. Gloves should be worn when potential contact with any body fluids is present.

The review of systems provides the nurse with: a.Physical findings related to each system. *b.Information regarding health promotion practices.* c.An opportunity to teach the patient medical terms. d.Information necessary for the nurse to diagnose the patient's medical problem.

B The purposes of the review of systems are to: (1) evaluate the past and current health state of each body system, (2) double check facts in case any significant data were omitted in the present illness section, and (3) evaluate health promotion practices

When performing a physical assessment, the first technique the nurse will always use is: a.Palpation. b.Inspection. c.Percussion. d.Auscultation.

B The skills requisite for the physical examination are inspection, palpation, percussion, and auscultation. The skills are performed one at a time and in this order (with the exception of the abdominal assessment, during which auscultation takes place before palpation and percussion). The assessment of each body system begins with inspection. A focused inspection takes time and yields a surprising amount of information.

The nurse is teaching a class on basic assessment skills. Which of these statements is true regarding the stethoscope and its use? a.Slope of the earpieces should point posteriorly (toward the occiput). b.Although the stethoscope does not magnify sound, it does block out extraneous room noise. c.Fit and quality of the stethoscope are not as important as its ability to magnify sound. d.Ideal tubing length should be 22 inches to dampen the distortion of sound.

B The stethoscope does not magnify sound, but it does block out extraneous room sounds. The slope of the earpieces should point forward toward the examiner's nose. Long tubing will distort sound. The fit and quality of the stethoscope are both important

In assessing the tonsils of a 30 year old, the nurse notices that they are involuted, granular in appearance, and appear to have deep crypts. What is correct response to these findings? a.Refer the patient to a throat specialist. b.No response is needed; this appearance is normal for the tonsils. c.Continue with the assessment, looking for any other abnormal findings. d.Obtain a throat culture on the patient for possible streptococcal (strep) infection.

B The tonsils are the same color as the surrounding mucous membrane, although they look more granular and their surface shows deep crypts. Tonsillar tissue enlarges during childhood until puberty and then involutes.

With which of these patients would it be most appropriate for the nurse to use games during the assessment, such as having the patient "blow out" the light on the penlight? a.Infant b.Preschool child c.School-age child d.Adolescent

B When assessing preschool children, using games or allowing them to play with the equipment to reduce their fears can be helpful. Such games are not appropriate for the other age groups.

While obtaining a health history, a patient tells the nurse that he has frequent nosebleeds and asks the best way to get them to stop. What would be the nurse's best response? a."While sitting up, place a cold compress over your nose." b."Sit up with your head tilted forward and pinch your nose." c."Just allow the bleeding to stop on its own, but don't blow your nose." d."Lie on your back with your head tilted back and pinch your nose."

B With a nosebleed, the person should sit up with the head tilted forward and pinch the nose between the thumb and forefinger for 5 to 15 minutes.

The nurse is preparing to examine a 4-year-old child. Which action is appropriate for this age group? a.Explain the procedures in detail to alleviate the child's anxiety. b.Give the child feedback and reassurance during the examination. c.Do not ask the child to remove his or her clothes because children at this age are usually very private. d.Perform an examination of the ear, nose, and throat first, and then examine the thorax and abdomen.

B With preschool children, short, simple explanations should be used. Children at this age are usually willing to undress. An examination of the head should be performed last. During the examination, needed feedback and reassurance should be given to the preschooler.

During an assessment of the CNs, the nurse finds the following: asymmetry when the patient smiles or frowns, uneven lifting of the eyebrows, sagging of the lower eyelids, and escape of air when the nurse presses against the right puffed cheek. This would indicate dysfunction of which of these CNs? a.Motor component of CN IV b.Motor component of CN VII c.Motor and sensory components of CN XI d.Motor component of CN X and sensory component of CN VII

B findings listed reflect a dysfunction of the motor component of the facial nerve (CN VII).

31. A 40-year-old man states that his physician told him that he has a hernia. He asks the nurse to explain what a hernia is. Which response by the nurse is appropriate? a. "No need to worry. Most men your age develop hernias." b. "A hernia is a loop of bowel protruding through a weak spot in the abdominal muscles." c. "A hernia is the result of prenatal growth abnormalities that are just now causing problems." d. "I'll have to have your physician explain this to you."

B) "A hernia is a loop of bowel protruding through a weak spot in the abdominal muscles." The nurse should explain that a hernia is a protrusion of the abdominal viscera through an abnormal opening in the muscle wall.

11. The nurse is watching a new graduate nurse perform auscultation of a patient's abdomen. Which statement by the new graduate shows a correct understanding of the reason auscultation precedes percussion and palpation of the abdomen? a. "We need to determine the areas of tenderness before using percussion and palpation." b. "Auscultation prevents distortion of bowel sounds that might occur after percussion and palpation." c. "Auscultation allows the patient more time to relax and therefore be more comfortable with the physical examination." d. "Auscultation prevents distortion of vascular sounds, such as bruits and hums, that might occur after percussion and palpation."

B) "It prevents distortion of bowel sounds that might occur after percussion and palpation." Auscultation is performed first (after inspection) because percussion and palpation can increase peristalsis, which would give a false interpretation of bowel sounds.

27. During an abdominal assessment, the nurse is unable to hear bowel sounds in a patient's abdomen. Before reporting this finding as "silent bowel sounds" the nurse should listen for at least: a. 1 minute. b. 5 minutes. c. 10 minutes. d. 2 minutes in each quadrant.

B) 5 minutes. Absent bowel sounds are rare. The nurse must listen for 5 minutes before deciding that bowel sounds are completely absent.

14. During an abdominal assessment, the nurse would consider which of these findings as normal? a. Presence of a bruit in the femoral area b. Tympanic percussion note in the umbilical region c. Palpable spleen between the ninth and eleventh ribs in the left midaxillary line d. Dull percussion note in the left upper quadrant at the midclavicular line

B) A tympanic percussion note in the umbilical region Tympany should predominate in all four quadrants of the abdomen because air in the intestines rises to the surface when the person is supine. Vascular bruits are not usually present. Normally, the spleen is not palpable. Dullness would not be found in the area of lung resonance (left upper quadrant at the midclavicular line).

5. A mother and her 13-year-old daughter express their concern related to the daughter's recent weight gain and increase in appetite. Which of these statements represents information the nurse should discuss with them?

B) Snacks should be high in protein, iron, and calcium.

10. A patient has hypoactive bowel sounds. The nurse knows that a potential cause of hypoactive bowel sounds is: a. Diarrhea. b. Peritonitis. c. Laxative use. d. Gastroenteritis.

B) peritonitis. Diminished or absent bowel sounds signal decreased motility from inflammation as exhibited with peritonitis, with paralytic ileus after abdominal surgery, or with late bowel obstruction.

15. The nurse is assessing the abdomen of a pregnant woman who is complaining of having "acid indigestion" all the time. The nurse knows that esophageal reflux during pregnancy can cause: a. Diarrhea. b. Pyrosis. c. Dysphagia. d. Constipation.

B) pyrosis. Pyrosis, or heartburn, is caused by esophageal reflux during pregnancy. The other options are not correct.

41. During an assessment of a patient who has been homeless for several years, the nurse notices that his tongue is magenta in color. This is an indication of _____ deficiency.

B) riboflavin

The nurse is conducting a developmental history on a 5-year-old child. Which questions are appropriate to ask the parents for this part of the assessment? Select all that apply. a."How much junk food does your child eat?" b."How many teeth has he lost, and when did he lose them?" c."Is he able to tie his shoelaces?" d."Does he take a children's vitamin?" e."Can he tell time?" f."Does he have any food allergies?"

B, C, E Questions about tooth loss, ability to tell time, and ability to tie shoelaces are appropriate questions for a developmental assessment. Questions about junk food intake and vitamins are part of a nutritional history. Questions about food allergies are not part of a developmental history.

The nurse is testing the hearing of a 78-year-old man and is reminded of the changes in hearing that occur with aging that include which of the following? Select all that apply. a. Hearing loss related to aging begins in the mid 40s. b. Progression of hearing loss is slow. c. The aging person has low-frequency tone loss. d. The aging person may find it harder to hear consonants than vowels. e. Sounds may be garbled and difficult to localize. f. Hearing loss reflects nerve degeneration of the middle ear.

B, D, E Presbycusis is a type of hearing loss that occurs with aging and is found in 60% of those older than 65 years. It is a gradual sensorineural loss caused by nerve degeneration in the inner ear or auditory nerve, and it slowly progresses after the age of 50 years. The person first notices a high-frequency tone loss; it is harder to hear consonants (high-pitched components of speech) than vowels, which makes words sound garbled. The ability to localize sound is also impaired.

The nurse is teaching a health class to high-school boys. When discussing the topic of using smokeless tobacco (SLT), which of these statements are accurate? Select all that apply. a.One pinch of SLT in the mouth for 30 minutes delivers the equivalent of one cigarette. b.Using SLT has been associated with a greater risk of oral cancer than smoking. c.Pain is an early sign of oral cancer. d.Pain is rarely an early sign of oral cancer. e.Tooth decay is another risk of SLT because of the use of sugar as a sweetener. f.SLT is considered a healthy alternative to smoking.

B, D, E One pinch of SLT in the mouth for 30 minutes delivers the equivalent of three cigarettes. Pain is rarely an early sign of oral cancer. Many brands of SLT are sweetened with sugars, which promotes tooth decay. SLT is not considered a healthy alternative to smoking, and the use of SLT has been associated with a greater risk of oral cancer than smoking.

During an assessment, a patient mentions that "I just can't smell like I used to. I can barely smell the roses in my garden. Why is that?" For which possible causes of changes in the sense of smell will the nurse assess? Select all that apply. a.Chronic alcohol use b.Cigarette smoking c.Frequent episodes of strep throat d.Chronic allergies e.Aging f.Herpes simplex virus I

B, D, E The sense of smell diminishes with cigarette smoking, chronic allergies, and aging. Chronic alcohol use, a history of strep throat, and herpes simplex virus I are not associated with changes in the sense of smell.

A patient has been admitted after an accident at work. During the assessment, the patient is having trouble hearing and states, "I don't know what the matter is. All of a sudden, I can't hear you out of my left ear!" What should the nurse do next? a. Make note of this finding for the report to the next shift. b. Prepare to remove cerumen from the patient's ear. c. Notify the patient's health care provider. d. Irrigate the ear with rubbing alcohol.

C Any sudden loss of hearing in one or both ears that is not associated with an upper respiratory infection needs to be reported at once to the patient's health care provider. Hearing loss associated with trauma is often sudden. Irrigating the ear or removing cerumen is not appropriate at this time.

Which of these would be formulated by a nurse using diagnostic reasoning? a. Nursing diagnosis b. Medical diagnosis c. Diagnostic hypothesis d. Diagnostic assessment

C Diagnostic reasoning calls for the nurse to formulate a diagnostic hypothesis; the nursing process calls for a nursing diagnosis.

The nurse is preparing to perform an otoscopic examination of a newborn infant. Which statement is true regarding this examination? a. Immobility of the drum is a normal finding. b. An injected membrane would indicate an infection. c. The normal membrane may appear thick and opaque. d. The appearance of the membrane is identical to that of an adult.

C During the first few days after the birth, the tympanic membrane of a newborn often appears thickened and opaque. It may look injected and have a mild redness from increased vascularity. The other statements are not correct.

The nurse is reviewing information about evidence-based practice (EBP). Which statement best reflects EBP? a. EBP relies on tradition for support of best practices. b. EBP is simply the use of best practice techniques for the treatment of patients. c. EBP emphasizes the use of best evidence with the clinician's experience. d. The patients own preferences are not important with EBP.

C EBP is a systematic approach to practice that emphasizes the use of best evidence in combination with the clinicians experience, as well as patient preferences and values, when making decisions about care and treatment. EBP is more than simply using the best practice techniques to treat patients, and questioning tradition is important when no compelling and supportive research evidence exists.

The nurse has implemented several planned interventions to address the nursing diagnosis of acute pain. Which would be the next appropriate action? a. Establish priorities. b. Identify expected outcomes. c. Evaluate the individuals condition, and compare actual outcomes with expected outcomes. d. Interpret data, and then identify clusters of cues and make inferences.

C Evaluation is the next step after the implementation phase of the nursing process. During this step, the nurse evaluates the individuals condition and compares the actual outcomes with expected outcomes (See Figure 1-2).

A 70-year-old patient tells the nurse that he has noticed that he is having trouble hearing, especially in large groups. He says that he "can't always tell where the sound is coming from" and the words often sound "mixed up." What might the nurse suspect as the cause for this change? a. Atrophy of the apocrine glands b. Cilia becoming coarse and stiff c. Nerve degeneration in the inner ear d. Scarring of the tympanic membrane

C Presbycusis is a type of hearing loss that occurs in 60% of those older than 65 years of age, even in those living in a quiet environment. This sensorineural loss is gradual and caused by nerve degeneration in the inner ear. Words sound garbled, and the ability to localize sound is also impaired. This communication dysfunction is accentuated when background noise is present.

When considering priority setting of problems, the nurse keeps in mind that second-level priority problems include which of these aspects? a. Low self-esteem b. Lack of knowledge c. Abnormal laboratory values d. Severely abnormal vital signs

C Second-level priority problems are those that require prompt intervention to forestall further deterioration (e.g., mental status change, acute pain, abnormal laboratory values, risks to safety or security) (see Table 1-1).

A visiting nurse is making an initial home visit for a patient who has many chronic medical problems. Which type of data base is most appropriate to collect in this setting? a. A follow-up data base to evaluate changes at appropriate intervals b. An episodic data base because of the continuing, complex medical problems of this patient c. A complete health data base because of the nurses primary responsibility for monitoring the patients health d. An emergency data base because of the need to collect information and make accurate diagnoses rapidly

C The complete data base is collected in a primary care setting, such as a pediatric or family practice clinic, independent or group private practice, college health service, womens health care agency, visiting nurse agency, or community health agency. In these settings, the nurse is the first health professional to see the patient and has the primary responsibility for monitoring the persons health care.

The nurse is reviewing the function of the cranial nerves (CNs). Which CN is responsible for conducting nerve impulses to the brain from the organ of Corti? a. I b. III c. VIII d. XI

C The nerve impulses are conducted by the auditory portion of CN VIII to the brain.

The nurse is performing an otoscopic examination on an adult. Which of these actions is correct? a. Tilting the person's head forward during the examination b. Once the speculum is in the ear, releasing the traction c. Pulling the pinna up and back before inserting the speculum d. Using the smallest speculum to decrease the amount of discomfort

C The pinna is pulled up and back on an adult or older child, which helps straighten the S-shape of the canal. Traction should not be released on the ear until the examination is completed and the otoscope is removed.

During an examination, the patient states he is hearing a buzzing sound and says that it is "driving me crazy!" The nurse recognizes that this symptom indicates: a. Vertigo. b. Pruritus. c. Tinnitus. d. Cholesteatoma.

C Tinnitus is a sound that comes from within a person; it can be a ringing, crackling, or buzzing sound. It accompanies some hearing or ear disorders

When listening to a patients breath sounds, the nurse is unsure of a sound that is heard. The nurses next action should be to: a. Immediately notify the patients physician. b. Document the sound exactly as it was heard. c. Validate the data by asking a coworker to listen to the breath sounds. d. Assess again in 20 minutes to note whether the sound is still present.

C When unsure of a sound heard while listening to a patients breath sounds, the nurse validates the data to ensure accuracy. If the nurse has less experience in an area, then he or she asks an expert to listen.

The nurse is preparing to examine a 6-year-old child. Which action is most appropriate? a.The thorax, abdomen, and genitalia are examined before the head. b.Talking about the equipment being used is avoided because doing so may increase the child's anxiety. c.The nurse should keep in mind that a child at this age will have a sense of modesty. d.The child is asked to undress from the waist up.

C A 6-year-old child has a sense of modesty. The child should undress him or herself, leaving underpants on and using a gown or drape. A school-age child is curious to know how equipment works, and the sequence should progress from the child's head to the toes.

The nurse is assessing a patient in the hospital who has received numerous antibiotics and notices that his tongue appears to be black and hairy. In response to his concern, what would the nurse say? a."We will need to get a biopsy to determine the cause." b."This is an overgrowth of hair and will go away in a few days." c."Black, hairy tongue is a fungal infection caused by all the antibiotics you have received." d."This is probably caused by the same bacteria you had in your lungs."

C A black, hairy tongue is not really hair but the elongation of filiform papillae and painless overgrowth of mycelial threads of fungus infection on the tongue. It occurs after the use of antibiotics, which inhibit normal bacteria and allow a proliferation of fungus.

The nurse is assessing a 3 year old for "drainage from the nose." On assessment, a purulent drainage that has a very foul odor is noted from the left naris and no drainage is observed from the right naris. The child is afebrile with no other symptoms. What should the nurse do next? a.Refer to the physician for an antibiotic order. b.Have the mother bring the child back in 1 week. c.Perform an otoscopic examination of the left nares. d.Tell the mother that this drainage is normal for a child of this age.

C Children are prone to put an object up the nose, producing unilateral purulent drainage with a foul odor. Because some risk for aspiration exists, removal should be prompt.

During an assessment of a 26 year old at the clinic for "a spot on my lip I think is cancer," the nurse notices a group of clear vesicles with an erythematous base around them located at the lip-skin border. The patient mentions that she just returned from Hawaii. What would be the most appropriate response by the nurse? a.Tell the patient she needs to see a skin specialist. b.Discuss the benefits of having a biopsy performed on any unusual lesion. c.Tell the patient that these vesicles are indicative of herpes simplex I or cold sores and that they will heal in 4 to 10 days. d.Tell the patient that these vesicles are most likely the result of a riboflavin deficiency and discuss nutrition.

C Cold sores are groups of clear vesicles with a surrounding erythematous base. These evolve into pustules or crusts and heal in 4 to 10 days. The most likely site is the lip-skin junction. Infection often recurs in the same site. Recurrent herpes infections may be precipitated by sunlight, fever, colds, or allergy.

The nurse is unable to palpate the right radial pulse on a patient. The best action would be to: a.Auscultate over the area with a fetoscope. b.Use a goniometer to measure the pulsations. c.Use a Doppler device to check for pulsations over the area. d.Check for the presence of pulsations with a stethoscope.

C Doppler devices are used to augment pulse or blood pressure measurements. Goniometers measure joint range of motion. A fetoscope is used to auscultate fetal heart tones. Stethoscopes are used to auscultate breath, bowel, and heart sounds.

A man who was found wandering in a park at 2 AM has been brought to the emergency department for an examination; he said he fell and hit his head. During the examination, the nurse asks him to use his index finger to touch the nurse's finger, then his own nose, then the nurse's finger again (which has been moved to a different location). The patient is clumsy, unable to follow the instructions, and overshoots the mark, missing the finger. The nurse should suspect which of the following? a.Cerebral injury b.Cerebrovascular accident c.Acute alcohol intoxication d.Peripheral neuropathy

C During the finger-to-finger test, if the person has clumsy movement with overshooting the mark, either a cerebellar disorder or acute alcohol intoxication should be suspected. The persons movements should be smooth and accurate. The other options are not correct.

A 92-year-old patient has had a stroke. The right side of his face is drooping. The nurse might also suspect which of these assessment findings? a.Epistaxis b.Rhinorrhea c.Dysphagia d.Xerostomia

C Dysphagia is difficulty with swallowing and may occur with a variety of disorders, including stroke and other neurologic diseases. Rhinorrhea is a runny nose, epistaxis is a bloody nose, and xerostomia is a dry mouth.

When the nurse asks for a description of who lives with a child, the method of discipline, and the support system of the child, what part of the assessment is being performed? a.Family history b.Review of systems *c.Functional assessment* d.Reason for seeking care

C Functional assessment includes interpersonal relationships and home environment. Family history includes illnesses in family members; a review of systems includes questions about the various body systems; and the reason for seeking care is the rationale for requesting health care.

The nurse is performing a neurologic assessment on a 41-year-old woman with a history of diabetes. When testing her ability to feel the vibrations of a tuning fork, the nurse notices that the patient is unable to feel vibrations on the great toe or ankle bilaterally, but she is able to feel vibrations on both patellae. Given this information, what would the nurse suspect? a.Hyperalgesia b.Hyperesthesia c.Peripheral neuropathy d.Lesion of sensory cortex

C Loss of vibration sense occurs with peripheral neuropathy (e.g., diabetes and alcoholism). Peripheral neuropathy is worse at the feet and gradually improves as the examiner moves up the leg, as opposed to a specific nerve lesion, which has a clear zone of deficit for its dermatome. The other responses are incorrect.

In response to a question about stress, a 39-year-old woman tells the nurse that her husband and mother both died in the past year. Which response by the nurse is most appropriate? a."This has been a difficult year for you." b."I don't know how anyone could handle that much stress in 1 year!" *c."What did you do to cope with the loss of both your husband and mother?"* d."That is a lot of stress; now let's go on to the next section of your history."

C Questions about coping and stress management include questions regarding the kinds of stresses in one's life, especially in the last year, any changes in lifestyle or any current stress, methods tried to relieve stress, and whether these methods have been helpful.

When examining the nares of a 45-year-old patient who has complaints of rhinorrhea, itching of the nose and eyes, and sneezing, the nurse notices the following: pale turbinates, swelling of the turbinates, and clear rhinorrhea. Which of these conditions is most likely the cause? a.Nasal polyps b.Acute sinusitis c.Allergic rhinitis d.Acute rhinitis

C Rhinorrhea, itching of the nose and eyes, and sneezing are present with allergic rhinitis. On physical examination, serous edema is noted, and the turbinates usually appear pale with a smooth, glistening surface. (See Table 16-1 for descriptions of the other conditions.)

A patient comes into the clinic complaining of facial pain, fever, and malaise. On examination, the nurse notes swollen turbinates and purulent discharge from the nose. The patient also complains of a dull, throbbing pain in his cheeks and teeth on the right side and pain when the nurse palpates the areas. The nurse recognizes that this patient has: a.Posterior epistaxis. b.Frontal sinusitis. c.Maxillary sinusitis. d.Nasal polyps.

C Signs of maxillary sinusitis include facial pain after upper respiratory infection, red swollen nasal mucosa, swollen turbinates, and purulent discharge. The person also has fever, chills, and malaise. With maxillary sinusitis, dull throbbing pain occurs in the cheeks and teeth on the same side, and pain with palpation is present. With frontal sinusitis, pain is above the supraorbital ridge.

During an assessment of an 80-year-old patient, the nurse notices the following: an inability to identify vibrations at her ankle and to identify the position of her big toe, a slower and more deliberate gait, and a slightly impaired tactile sensation. All other neurologic findings are normal. The nurse should interpret that these findings indicate: a.CN dysfunction. b.Lesion in the cerebral cortex. c.Normal changes attributable to aging. d.Demyelination of nerves attributable to a lesion

C Some aging adults show a slower response to requests, especially for those calling for coordination of movements. The findings listed are normal in the absence of other significant abnormal findings. The other responses are incorrect.

The nurse is caring for a patient who has just had neurosurgery. To assess for increased intracranial pressure, what would the nurse include in the assessment? a.CNs, motor function, and sensory function b.Deep tendon reflexes, vital signs, and coordinated movements c.Level of consciousness, motor function, pupillary response, and vital signs d.Mental status, deep tendon reflexes, sensory function, and pupillary response

C Some hospitalized persons have head trauma or a neurologic deficit from a systemic disease process. These people must be closely monitored for any improvement or deterioration in neurologic status and for any indication of increasing intracranial pressure. The nurse should use an abbreviation of the neurologic examination in the following sequence: level of consciousness, motor function, pupillary response, and vital signs.

Which of these statements is true regarding the use of Standard Precautions in the health care setting? a.Standard Precautions apply to all body fluids, including sweat. b.Use alcohol-based hand rub if hands are visibly dirty. c.Standard Precautions are intended for use with all patients, regardless of their risk or presumed infection status. d.Standard Precautions are to be used only when nonintact skin, excretions containing visible blood, or expected contact with mucous membranes is present.

C Standard Precautions are designed to reduce the risk of transmission of microorganisms from both recognized and unrecognized sources and are intended for use for all patients, regardless of their risk or presumed infection status. Standard Precautions apply to blood and all other body fluids, secretions and excretions except sweat—regardless of whether they contain visible blood, nonintact skin, or mucous membranes. Hands should be washed with soap and water if visibly soiled with blood or body fluids. Alcohol-based hand rubs can be used if hands are not visibly soiled.

While assessing a 7-month-old infant, the nurse makes a loud noise and notices the following response: abduction and flexion of the arms and legs; fanning of the fingers, and curling of the index finger and thumb in a C position, followed by the infant bringing in the arms and legs to the body. What does the nurse know about this response? a.This response could indicate brachial nerve palsy. b.This reaction is an expected startle response at this age. c.This reflex should have disappeared between 1 and 4 months of age. d.This response is normal as long as the movements are bilaterally symmetric.

C The Moro reflex is present at birth and usually disappears at 1 to 4 months. Absence of the Moro reflex in the newborn or its persistence after 5 months of age indicates severe central nervous system injury. The other responses are incorrect.

A 2-year-old child has been brought to the clinic for a well-child checkup. The best way for the nurse to begin the assessment is to: a.Ask the parent to place the child on the examining table. b.Have the parent remove all of the child's clothing before the examination. c.Allow the child to keep a security object such as a toy or blanket during the examination. d.Initially focus the interactions on the child, essentially ignoring the parent until the child's trust has been obtained.

C The best place to examine the toddler is on the parent's lap. Toddlers understand symbols; therefore, a security object is helpful. Initially, the focus is more on the parent, which allows the child to adjust gradually and to become familiar with you. A 2-year-old child does not like to take off his or her clothes. Therefore, ask the parent to undress one body part at a time.

A 30-year-old woman tells the nurse that she has been very unsteady and has had difficulty in maintaining her balance. Which area of the brain that is related to these findings would concern the nurse? a.Thalamus b.Brainstem c.Cerebellum d.Extrapyramidal tract

C The cerebellar system coordinates movement, maintains equilibrium, and helps maintain posture. The thalamus is the primary relay station where sensory pathways of the spinal cord, cerebellum, and brainstem form synapses on their way to the cerebral cortex. The brainstem consists of the midbrain, pons, and medulla and has various functions, especially concerning autonomic centers. The extrapyramidal tract maintains muscle tone for gross automatic movements, such as walking.

The nurse is examining a patient's lower leg and notices a draining ulceration. Which of these actions is most appropriate in this situation? a.Washing hands, and contacting the physician b.Continuing to examine the ulceration, and then washing hands c.Washing hands, putting on gloves, and continuing with the examination of the ulceration d.Washing hands, proceeding with rest of the physical examination, and then continuing with the examination of the leg ulceration

C The examiner should wear gloves when the potential contact with any body fluids is present. In this situation, the nurse should wash his or her hands, put on gloves, and continue examining the ulceration.

A mother is concerned because her 18-month-old toddler has 12 teeth. She is wondering if this is normal for a child of this age. The nurse's best response would be: a."How many teeth did you have at this age?" b."All 20 deciduous teeth are expected to erupt by age 4 years." c."This is a normal number of teeth for an 18 month old." d."Normally, by age 2 years, 16 deciduous teeth are expected."

C The guidelines for the number of teeth for children younger than 2 years old are as follows: the child's age in months minus the number 6 should be equal to the expected number of deciduous teeth. Normally, all 20 teeth are in by 2 years old. In this instance, the child is 18 months old, minus 6, equals 12 deciduous teeth expected.

The nurse is performing a review of systems on a 76-year-old patient. Which of these statements is correct for this situation? a.The questions asked are identical for all ages. b.The interviewer will start incorporating different questions for patients 70 years of age and older. *c. Questions that are reflective of the normal effects of aging are added.* d.At this age, a review of systems is not necessary—the focus should be on current problems.

C The health history includes the same format as that described for the younger adult, as well as some additional questions. These additional questions address ways in which the activities of daily living may have been affected by the normal aging processes or by the effects of chronic illness or disability.

The nurse notices that the mother of a 2-year-old boy brings him into the clinic quite frequently for various injuries and suspects there may be some child abuse involved. During an inspection of his mouth, the nurse should look for: a.Swollen, red tonsils. b.Ulcerations on the hard palate. c.Bruising on the buccal mucosa or gums. d.Small yellow papules along the hard palate.

C The nurse should notice any bruising or laceration on the buccal mucosa or gums of an infant or young child. Trauma may indicate child abuse from a forced feeding of a bottle or spoon.

The nurse is assessing an 80-year-old patient. Which of these findings would be expected for this patient? a.Hypertrophy of the gums b.Increased production of saliva c.Decreased ability to identify odors d.Finer and less prominent nasal hair

C The sense of smell may be reduced because of a decrease in the number of olfactory nerve fibers. Nasal hairs grow coarser and stiffer with aging. The gums may recede with aging, not hypertrophy, and saliva production decreases.

A patient with a lack of oxygen to his heart will have pain in his chest and possibly in the shoulder, arms, or jaw. The nurse knows that the best explanation why this occurs is which one of these statements? a.A problem exists with the sensory cortex and its ability to discriminate the location. b.The lack of oxygen in his heart has resulted in decreased amount of oxygen to the areas experiencing the pain. c.The sensory cortex does not have the ability to localize pain in the heart; consequently, the pain is felt elsewhere. d.A lesion has developed in the dorsal root, which is preventing the sensation from being transmitted normally.

C The sensory cortex is arranged in a specific pattern, forming a corresponding map of the body. Pain in the right hand is perceived at a specific spot on the map. Some organs, such as the heart, liver, and spleen, are absent from the brain map. Pain originating in these organs is referred because no felt image exists in which to have pain. Pain is felt by proxy, that is, by another body part that does have a felt image. The other responses are not correct explanations.

During the examination, offering some brief teaching about the patient's body or the examiner's findings is often appropriate. Which one of these statements by the nurse is most appropriate? a."Your atrial dysrhythmias are under control." b."You have pitting edema and mild varicosities." c."Your pulse is 80 beats per minute, which is within the normal range." d."I'm using my stethoscope to listen for any crackles, wheezes, or rubs."

C The sharing of some information builds rapport, as long as the patient is able to understand the terminology.

The area of the nervous system that is responsible for mediating reflexes is the: a.Medulla. b.Cerebellum. c.Spinal cord. d.Cerebral cortex.

C The spinal cord is the main highway for ascending and descending fiber tracts that connect the brain to the spinal nerves; it is responsible for mediating reflexes

The nurse is unable to identify any changes in sound when percussing over the abdomen of an obese patient. What should the nurse do next? a.Ask the patient to take deep breaths to relax the abdominal musculature. b.Consider this finding as normal, and proceed with the abdominal assessment. c.Increase the amount of strength used when attempting to percuss over the abdomen. d.Decrease the amount of strength used when attempting to percuss over the abdomen.

C The thickness of the person's body wall will be a factor. The nurse needs a stronger percussion stroke for persons with obese or very muscular body walls. The force of the blow determines the loudness of the note. The other actions are not correct.

The nurse is performing an assessment on a 29-year-old woman who visits the clinic complaining of "always dropping things and falling down." While testing rapid alternating movements, the nurse notices that the woman is unable to pat both of her knees. Her response is extremely slow and she frequently misses. What should the nurse suspect? a.Vestibular disease b.Lesion of CN IX c.Dysfunction of the cerebellum d.Inability to understand directions

C When a person tries to perform rapid, alternating movements, responses that are slow, clumsy, and sloppy are indicative of cerebellar disease. The other responses are incorrect.

A 10 year old is at the clinic for "a sore throat that has lasted 6 days." Which of these findings would be consistent with an acute infection? a.Tonsils 1+/1-4+ and pink; the same color as the oral mucosa b.Tonsils 2+/1-4+ with small plugs of white debris c.Tonsils 3+/1-4+ with large white spots d.Tonsils 3+/1-4+ with pale coloring

C With an acute infection, tonsils are bright red and swollen and may have exudate or large white spots. Tonsils are enlarged to 2+, 3+, or 4+ with an acute infection.

The nurse is testing superficial reflexes on an adult patient. When stroking up the lateral side of the sole and across the ball of the foot, the nurse notices the plantar flexion of the toes. How should the nurse document this finding? a.Positive Babinski sign b.Plantar reflex abnormal c.Plantar reflex present d.Plantar reflex 2+ on a scale from "0 to 4+"

C With the same instrument, the nurse should draw a light stroke up the lateral side of the sole of the foot and across the ball of the foot, similar to an upside-down J. The normal response is plantar flexion of the toes and sometimes of the entire foot. A positive Babinski sign is abnormal and occurs with the response of dorsiflexion of the big toe and fanning of all toes. The plantar reflex is not graded on a 0 to 4+ scale.

33. Which of these interventions is most appropriate when the nurse is planning nutritional interventions for a healthy, active 74-year-old woman?

C) Decrease the number of calories she is eating because of the decrease in energy requirements from loss of lean body mass.

21. The nurse notices that a patient has had a black, tarry stool and recalls that a possible cause would be: a. Gallbladder disease. b. Overuse of laxatives. c. Gastrointestinal bleeding. d. Localized bleeding around the anus.

C) gastrointestinal bleeding. Black stools may be tarry as a result of occult blood (melena) from gastrointestinal bleeding. Red blood in stools occurs with localized bleeding around the anus.

18. A patient is complaining of a sharp pain along the costovertebral angles. The nurse knows that this symptom is most often indicative of: a. Ovary infection. b. Liver enlargement. c. Kidney inflammation. d. Spleen enlargement.

C) kidney inflammation. Sharp pain along the costovertebral angles occurs with inflammation of the kidney or paranephric area. The other options are not correct.

The nurse is performing a physical assessment on a newly admitted patient. An example of objective information obtained during the physical assessment includes the: a. Patients history of allergies. b. Patients use of medications at home. c. Last menstrual period 1 month ago. d. 2 5 cm scar on the right lower forearm.

D Objective data are the patients record, laboratory studies, and condition that the health professional observes by inspecting, percussing, palpating, and auscultating during the physical examination. The other responses reflect subjective data.

The mother of a 2-year-old toddler is concerned about the upcoming placement of tympanostomy tubes in her son's ears. The nurse would include which of these statements in the teaching plan? a. The tubes are placed in the inner ear. b. The tubes are used in children with sensorineural loss. c. The tubes are permanently inserted during a surgical procedure. d. The purpose of the tubes is to decrease the pressure and allow for drainage.

D Polyethylene tubes are surgically inserted into the eardrum to relieve middle ear pressure and to promote drainage of chronic or recurrent middle ear infections. Tubes spontaneously extrude in 6 months to 1 year.

The nurse is performing an assessment. Which of these findings would cause the greatest concern? a.Painful vesicle inside the cheek for 2 days b.Presence of moist, nontender Stensen's ducts c.Stippled gingival margins that snugly adhere to the teeth d.Ulceration on the side of the tongue with rolled edges

D Ulceration on the side or base of the tongue or under the tongue raises the suspicion of cancer and must be investigated. The risk of early metastasis is present because of rich lymphatic drainage. The vesicle may be an aphthous ulcer, which is painful but not dangerous. The other responses are normal findings.

The nurse is preparing to do a functional assessment. Which statement best describes the purpose of a functional assessment? a.The functional assessment assesses how the individual is coping with life at home. b.It determines how children are meeting developmental milestones. c.The functional assessment can identify any problems with memory the individual may be experiencing. d.It helps determine how a person is managing day-to-day activities.

D The functional assessment measures how a person manages day-to-day activities. The other answers do not reflect the purpose of a functional assessment.

The mother of a 2-year-old is concerned because her son has had three ear infections in the past year. What would be an appropriate response by the nurse? a. "It is unusual for a small child to have frequent ear infections unless something else is wrong." b. "We need to check the immune system of your son to determine why he is having so many ear infections." c. "Ear infections are not uncommon in infants and toddlers because they tend to have more cerumen in the external ear." d. "Your son's eustachian tube is shorter and wider than yours because of his age, which allows for infections to develop more easily."

D The infant's eustachian tube is relatively shorter and wider than the adult's eustachian tube, and its position is more horizontal; consequently, pathogens from the nasopharynx can more easily migrate through to the middle ear. The other responses are not appropriate.

In an individual with otitis externa, which of these signs would the nurse expect to find on assessment? a. Rhinorrhea b. Periorbital edema c. Pain over the maxillary sinuses d. Enlarged superficial cervical nodes

D The lymphatic drainage of the external ear flows to the parotid, mastoid, and superficial cervical nodes. The signs are severe swelling of the canal, inflammation, and tenderness. Rhinorrhea, periorbital edema, and pain over the maxillary sinuses do not occur with otitis externa.

The nursing process is a sequential method of problem solving that nurses use and includes which steps? a. Assessment, treatment, planning, evaluation, discharge, and follow-up b. Admission, assessment, diagnosis, treatment, and discharge planning c. Admission, diagnosis, treatment, evaluation, and discharge planning d. Assessment, diagnosis, outcome identification, planning, implementation, and evaluation

D The nursing process is a method of problem solving that includes assessment, diagnosis, outcome identification, planning, implementation, and evaluation.

The nurse is taking the history of a patient who may have a perforated eardrum. What would be an important question in this situation? a. "Do you ever notice ringing or crackling in your ears?" b. "When was the last time you had your hearing checked?" c. "Have you ever been told that you have any type of hearing loss?" d. "Is there any relationship between the ear pain and the discharge you mentioned?"

D Typically with perforation, ear pain occurs first, stopping with a popping sensation, and then drainage occurs.

A mother brings her 4-month-old infant to the clinic with concerns regarding a small pad in the middle of the upper lip that has been there since 1 month of age. The infant has no health problems. On physical examination, the nurse notices a 0.5-cm, fleshy, elevated area in the middle of the upper lip. No evidence of inflammation or drainage is observed. What would the nurse tell this mother? a."This area of irritation is caused from teething and is nothing to worry about." b."This finding is abnormal and should be evaluated by another health care provider." c."This area of irritation is the result of chronic drooling and should resolve within the next month or two." d."This elevated area is a sucking tubercle caused from the friction of breastfeeding or bottle-feeding and is normal."

D A normal finding in infants is the sucking tubercle, a small pad in the middle of the upper lip from the friction of breastfeeding or bottle-feeding. This condition is not caused by irritation, teething, or excessive drooling, and evaluation by another health care provider is not warranted.

When the nurse asks a 68-year-old patient to stand with his feet together and arms at his side with his eyes closed, he starts to sway and moves his feet farther apart. The nurse would document this finding as: a.Ataxia. b.Lack of coordination. c.Negative Homans sign. d.Positive Romberg sign.

D Abnormal findings for the Romberg test include swaying, falling, and a widening base of the feet to avoid falling. A positive Romberg sign is a loss of balance that is increased by the closing of the eyes. Ataxia is an uncoordinated or unsteady gait. Homans sign is used to test the legs for deep-vein thrombosis.

A 70-year-old woman tells the nurse that every time she gets up in the morning or after she's been sitting, she gets "really dizzy" and feels like she is going to fall over. The nurse's best response would be: a."Have you been extremely tired lately?" b."You probably just need to drink more liquids." c."I'll refer you for a complete neurologic examination." d."You need to get up slowly when you've been lying down or sitting."

D Aging is accompanied by a progressive decrease in cerebral blood flow. In some people, this decrease causes dizziness and a loss of balance with a position change. These individuals need to be taught to get up slowly. The other responses are incorrect.

The nurse is examining a 2-year-old child and asks, "May I listen to your heart now?" Which critique of the nurse's technique is most accurate? a.Asking questions enhances the child's autonomy b.Asking the child for permission helps develop a sense of trust c.This question is an appropriate statement because children at this age like to have choices d.Children at this age like to say, "No." The examiner should not offer a choice when no choice is available

D Children at this age like to say, "No." Choices should not be offered when no choice is really available. If the child says, "No" and the nurse does it anyway, then the nurse loses trust. Autonomy is enhanced by offering a limited option, "Shall I listen to your heart next or your tummy?"

Which of these responses might the nurse expect during a functional assessment of a patient whose leg is in a cast? a."I broke my right leg in a car accident 2 weeks ago." b."The pain is decreasing, but I still need to take acetaminophen." c."I check the color of my toes every evening just like I was taught." *d."I'm able to transfer myself from the wheelchair to the bed without help."*

D Functional assessment measures a person's self-care ability in the areas of general physical health or absence of illness. The other statements concern health or illness issues.

A pregnant woman states that she is concerned about her gums because she has noticed they are swollen and have started bleeding. What would be an appropriate response by the nurse? a."Your condition is probably due to a vitamin C deficiency." b."I'm not sure what causes swollen and bleeding gums, but let me know if it's not better in a few weeks." c."You need to make an appointment with your dentist as soon as possible to have this checked." d."Swollen and bleeding gums can be caused by the change in hormonal balance in your system during pregnancy."

D Gum margins are red and swollen and easily bleed with gingivitis. A changing hormonal balance may cause this condition to occur in pregnancy and puberty.

During auscultation of a patient's heart sounds, the nurse hears an unfamiliar sound. The nurse should: a.Document the findings in the patient's record. b.Wait 10 minutes, and auscultate the sound again. c.Ask the patient how he or she is feeling. d.Ask another nurse to double check the finding.

D If an abnormal finding is not familiar, then the nurse may ask another examiner to double check the finding. The other responses do not help identify the unfamiliar sound.

The nurse is obtaining a health history on a 3-month-old infant. During the interview, the mother states, "I think she is getting her first tooth because she has started drooling a lot." The nurse's best response would be: a."You're right, drooling is usually a sign of the first tooth." b."It would be unusual for a 3 month old to be getting her first tooth." c."This could be the sign of a problem with the salivary glands." d."She is just starting to salivate and hasn't learned to swallow the saliva."

D In the infant, salivation starts at 3 months. The baby will drool for a few months before learning to swallow the saliva. This drooling does not herald the eruption of the first tooth, although many parents think it does.

The nurse is preparing to assess a patient's abdomen by palpation. How should the nurse proceed? a.Palpation of reportedly "tender" areas are avoided because palpation in these areas may cause pain. b.Palpating a tender area is quickly performed to avoid any discomfort that the patient may experience. c.The assessment begins with deep palpation, while encouraging the patient to relax and to take deep breaths. d.The assessment begins with light palpation to detect surface characteristics and to accustom the patient to being touched.

D Light palpation is initially performed to detect any surface characteristics and to accustom the person to being touched. Tender areas should be palpated last, not first.

The nurse is reviewing the development of the newborn infant. Regarding the sinuses, which statement is true in relation to a newborn infant? a.Sphenoid sinuses are full size at birth. b.Maxillary sinuses reach full size after puberty. c.Frontal sinuses are fairly well developed at birth. d.Maxillary and ethmoid sinuses are the only sinuses present at birth.

D Only the maxillary and ethmoid sinuses are present at birth. The sphenoid sinuses are minute at birth and develop after puberty. The frontal sinuses are absent at birth, are fairly well developed at age 7 to 8 years, and reach full size after puberty.

The nurse hears bilateral loud, long, and low tones when percussing over the lungs of a 4-year-old child. The nurse should: a.Palpate over the area for increased pain and tenderness. b.Ask the child to take shallow breaths, and percuss over the area again. c.Immediately refer the child because of an increased amount of air in the lungs. d.Consider this finding as normal for a child this age, and proceed with the examination

D Percussion notes that are loud in amplitude, low in pitch, of a booming quality, and long in duration are normal over a child's lung.

While obtaining a health history from the mother of a 1-year-old child, the nurse notices that the baby has had a bottle in his mouth the entire time. The mother states, "It makes a great pacifier." The best response by the nurse would be: a."You're right. Bottles make very good pacifiers." b."Using a bottle as a pacifier is better for the teeth than thumb-sucking." c."It's okay to use a bottle as long as it contains milk and not juice." d."Prolonged use of a bottle can increase the risk for tooth decay and ear infections."

D Prolonged bottle use during the day or when going to sleep places the infant at risk for tooth decay and middle ear infections.

During an assessment, the nurse uses the CAGE test. The patient answers "yes" to two of the questions. What could this be indicating? a.The patient is an alcoholic. b.The patient is annoyed at the questions. c.The patient should be thoroughly examined for possible alcohol withdrawal symptoms. d.The nurse should suspect alcohol abuse and continue with a more thorough substance abuse assessment.

D The CAGE test is known as the "cut down, annoyed, guilty, and eye-opener" test. If a person answers "yes" to two or more of the four CAGE questions, then the nurse should suspect alcohol abuse and continue with a more complete substance abuse assessment.

Before auscultating the abdomen for the presence of bowel sounds on a patient, the nurse should: a.Warm the endpiece of the stethoscope by placing it in warm water. b.Leave the gown on the patient to ensure that he or she does not get chilled during the examination. c.Ensure that the bell side of the stethoscope is turned to the "on" position. d.Check the temperature of the room, and offer blankets to the patient if he or she feels cold.

D The examination room should be warm. If the patient shivers, then the involuntary muscle contractions can make it difficult to hear the underlying sounds. The end of the stethoscope should be warmed between the examiner's hands, not with water. The nurse should never listen through a gown. The diaphragm of the stethoscope should be used to auscultate for bowel sounds

The nurse is testing the function of CN XI. Which statement best describes the response the nurse should expect if this nerve is intact? The patient: a.Demonstrates the ability to hear normal conversation. b.Sticks out the tongue midline without tremors or deviation. c.Follows an object with his or her eyes without nystagmus or strabismus. d.Moves the head and shoulders against resistance with equal strength.

D The following normal findings are expected when testing the spinal accessory nerve (CN XI): The patients sternomastoid and trapezius muscles are equal in size; the person can forcibly rotate the head both ways against resistance applied to the side of the chin with equal strength; and the patient can shrug the shoulders against resistance with equal strength on both sides. Checking the patients ability to hear normal conversation checks the function of CN VIII. Having the patient stick out the tongue checks the function of CN XII. Testing the eyes for nystagmus or strabismus is performed to check CNs III, IV, and VI.

The nurse is preparing to complete a health assessment on a 16-year-old girl whose parents have brought her to the clinic. Which instruction would be appropriate for the parents before the interview begins? a."Please stay during the interview; you can answer for her if she does not know the answer." b."It would help to interview the three of you together." c."While I interview your daughter, will you please stay in the room and complete these family health history questionnaires?" d."While I interview your daughter, will you step out to the waiting room and complete these family health history questionnaires?"

D The girl should be interviewed alone. The parents can wait outside and fill out the family health history questionnaires.

During a checkup, a 22-year-old woman tells the nurse that she uses an over-the-counter nasal spray because of her allergies. She also states that it does not work as well as it used to when she first started using it. The best response by the nurse would be: a."You should never use over-the-counter nasal sprays because of the risk of addiction." b."You should try switching to another brand of medication to prevent this problem." c."Continuing to use this spray is important to keep your allergies under control." d."Using these nasal medications irritates the lining of the nose and may cause rebound swelling."

D The misuse of over-the-counter nasal medications irritates the mucosa, causing rebound swelling, which is a common problem.

During the taking of the health history of a 78-year-old man, his wife states that he occasionally has problems with short-term memory loss and confusion: "He can't even remember how to button his shirt." When assessing his sensory system, which action by the nurse is most appropriate? a.The nurse would not test the sensory system as part of the examination because the results would not be valid. b.The nurse would perform the tests, knowing that mental status does not affect sensory ability. c.The nurse would proceed with an explanation of each test, making certain that the wife understands. d.Before testing, the nurse would assess the patient's mental status and ability to follow directions.

D The nurse should ensure the validity of the sensory system testing by making certain that the patient is alert, cooperative, comfortable, and has an adequate attention span. Otherwise, the nurse may obtain misleading and invalid results.

A 50-year-old woman is in the clinic for weakness in her left arm and leg that she has noticed for the past week. The nurse should perform which type of neurologic examination? a.Glasgow Coma Scale b.Neurologic recheck examination c.Screening neurologic examination d.Complete neurologic examination

D The nurse should perform a complete neurologic examination on an individual who has neurologic concerns (e.g., headache, weakness, loss of coordination) or who is showing signs of neurologic dysfunction. The Glasgow Coma Scale is used to define a persons level of consciousness. The neurologic recheck examination is appropriate for those who are demonstrating neurologic deficits. The screening neurologic examination is performed on seemingly well individuals who have no significant subjective findings from the health history.

An examiner is using an ophthalmoscope to examine a patient's eyes. The patient has astigmatism and is nearsighted. The use of which of these techniques would indicate that the examination is being correctly performed? a.Using the large full circle of light when assessing pupils that are not dilated b.Rotating the lens selector dial to the black numbers to compensate for astigmatism c.Using the grid on the lens aperture dial to visualize the external structures of the eye d.Rotating the lens selector dial to bring the object into focus

D The ophthalmoscope is used to examine the internal eye structures. It can compensate for nearsightedness or farsightedness, but it will not correct for astigmatism. The grid is used to assess size and location of lesions on the fundus. The large full spot of light is used to assess dilated pupils. Rotating the lens selector dial brings the object into focus.

The nurse is preparing to perform a physical assessment. The correct action by the nurse is reflected by which statement? The nurse: a.Performs the examination from the left side of the bed. b.Examines tender or painful areas first to help relieve the patient's anxiety. c.Follows the same examination sequence, regardless of the patient's age or condition. d.Organizes the assessment to ensure that the patient does not change positions too often.

D The steps of the assessment should be organized to ensure that the patient does not change positions too often. The sequence of the steps of the assessment may differ, depending on the age of the person and the examiner's preference. Tender or painful areas should be assessed last.

A 78-year-old man has a history of a cerebrovascular accident. The nurse notes that when he walks, his left arm is immobile against the body with flexion of the shoulder, elbow, wrist, and fingers and adduction of the shoulder. His left leg is stiff and extended and circumducts with each step. What type of gait disturbance is this individual experiencing? a.Scissors gait b.Cerebellar ataxia c.Parkinsonian gait d.Spastic hemiparesis

D With spastic hemiparesis, the arm is immobile against the body. Flexion of the shoulder, elbow, wrist, and fingers occurs, and adduction of the shoulder, which does not swing freely, is observed. The leg is stiff and extended and circumducts with each step. Causes of this type of gait include cerebrovascular accident.

A 72-year-old patient has a history of hypertension and chronic lung disease. An important question for the nurse to include in the health history would be: a."Do you use a fluoride supplement?" b."Have you had tonsillitis in the last year?" c."At what age did you get your first tooth?" d."Have you noticed any dryness in your mouth?"

D Xerostomia (dry mouth) is a side effect of many drugs taken by older people, including antidepressants, anticholinergics, antispasmodics, antihypertensives, antipsychotics, and bronchodilators.

6. A 22-year-old man comes to the clinic for an examination after falling off his motorcycle and landing on his left side on the handlebars. The nurse suspects that he may have injured his spleen. Which of these statements is true regarding assessment of the spleen in this situation? a. The spleen can be enlarged as a result of trauma. b. The spleen is normally felt on routine palpation. c. If an enlarged spleen is noted, then the nurse should thoroughly palpate to determine its size. d. An enlarged spleen should not be palpated because it can easily rupture.

D) An enlarged spleen should not be palpated because it can rupture easily. If an enlarged spleen is felt, then the nurse should refer the person and should not continue to palpate it. An enlarged spleen is friable and can easily rupture with overpalpation.

4. A pregnant woman is interested in breastfeeding her baby, and asks several questions about the topic. Which information is appropriate for the nurse to share with her?

D) Breast milk provides the nutrients necessary for growth as well as natural immunity.

36. The nurse is preparing to measure fat and lean body mass and bone mineral density. Which tool is appropriate?

D) Dual-energy x-ray absorptiometry

15. The nurse is discussing appropriate foods with the mother of a 3-year-old child. Which of these foods are recommended?

D) Finger foods and nutritious snacks that can't cause choking

2. Which structure is located in the left lower quadrant of the abdomen? a. Liver b. Duodenum c. Gallbladder d. Sigmoid colon

D) Sigmoid colon The sigmoid colon is located in the left lower quadrant of the abdomen

43. An elderly patient in a nursing home has been receiving tube feedings for several months. During an oral examination, the nurse notes that patient's gums are swollen, ulcerated, and bleeding in some areas. The nurse suspects that the patient has what condition?

D) Vitamin C deficiency

38. During an abdominal assessment, the nurse tests for a fluid wave. A positive fluid wave test occurs with: a. Splenomegaly. b. Distended bladder. c. Constipation. d. Ascites.

D) ascites. If ascites (fluid in the abdomen) is present, then the examiner will feel a fluid wave when assessing the abdomen. A fluid wave is not present with splenomegaly, a distended bladder, or constipation.

8. The nurse is describing a scaphoid abdomen. To the horizontal plane, a scaphoid contour of the abdomen depicts a _____ profile. a. Flat b. Convex c. Bulging d. Concave

D) concave Contour describes the profile of the abdomen from the rib margin to the pubic bone; a scaphoid contour is one that is concave from a horizontal plane (see Figure 21-7)

32. A 45-year-old man is in the clinic for a physical examination. During the abdominal assessment, the nurse percusses the abdomen and notices an area of dullness above the right costal margin of about 10 cm. The nurse should: a. Document the presence of hepatomegaly. b. Ask additional health history questions regarding his alcohol intake. c. Describe this dullness as indicative of an enlarged liver, and refer him to a physician. d. Consider this finding as normal, and proceed with the examination.

D) consider this a normal finding and proceed with the examination. A liver span of 10.5 cm is the mean for males and 7 cm for females. Men and taller individuals are at the upper end of this range. Women and shorter individuals are at the lower end of this range. A liver span of 11 cm is within normal limits for this individual.

13. The physician comments that a patient has abdominal borborygmi. The nurse knows that this term refers to: a. Loud continual hum. b. Peritoneal friction rub. c. Hypoactive bowel sounds. d. Hyperactive bowel sounds.

D) hyperactive bowel sounds. Borborygmi is the term used for hyperperistalsis when the person actually feels his or her stomach growling.


Related study sets

BIBL 104 Quiz 2 Liberty University

View Set

Chapter 14 Smartbook (Bonds and Long-Term Notes)

View Set

Therapeutic Relationships Ch8 (for more ?'s see p. 135)

View Set

Ch 4-6 Exam: Questions and Anwsers

View Set

Chapter 16 Respiratory Emergencies Quiz and Test

View Set

CompTIA Security+ (SY0-401) Multiple Choice Sample Questions

View Set